AeroElectric-Archive.digest.vol-mh

March 12, 2014 - May 03, 2014



      
         We had this conversation many moons ago after somebody
         noticed that some high wing airplanes offered under-wing
         courtesy lights. Then some other readers hypothesized about
         baggage compartment lights on low wing airplanes that
         could be exercised even with the master switch off.
      
         The on-limit timer allows one to park the airplane
         and perhaps even button up the cockpit while using
         lights on the ship's battery to take care of final
         chores outside.
      
         I sketched an on-limit timer for somebody who was
         interested in adding the feature his airplane but
         I no longer recall his exact application.
      
         The notion of an on-limit timer was never to
         replace the forgetful pilot's remembering to
         turn it off . . . but to be able to leave the
         light on for convenience and have it go off by
         itself after the task needing illumination
         was completed.
      
      
         Bob . . . 
      
      
________________________________________________________________________________
Date: Mar 12, 2014
Subject: Re: Timer circuit for led array
From: David Duperron <davedup10(at)gmail.com>
Thanks for the reply Jeff. Do you have any more info on the automotive light delay relay? Dave Duperron (davedup10(at)gmail.com. On Wed, Mar 12, 2014 at 1:06 PM, Jeff Luckey wrote: > True but ... > > that overlooks the convenience of always-available cabin lighting without > having the Master on. > > The post earlier today that referenced an existing automotive delay relay > looks like a great solution - cheap, designed for the purpose, plug-n-play, > no re-inventing the wheel > > - what don't we like about that > > -Jeff > > ------------------------------ > *From:* Eric M. Jones > *To:* aeroelectric-list(at)matronics.com > *Sent:* Wednesday, March 12, 2014 9:42 AM > *Subject:* AeroElectric-List: Re: Timer circuit for led array > > emjones(at)charter.net> > > > > Timer circuit for led array Reply with quote > > I'm looking for a timer to shut off my cabin overhead utility light > after about 10 minutes of operation. It's fed directly from the main > battery buss and I would like to know that if I forgot to turn it off the > timer would do it for me before draining the battery. Any ideas? David > Duperron > > > I learned to fly in Cessnas, and as I recall leaving the beacon on was a > certain way to avoid walking away from the airplane with something still on. > Perhaps the solution to your problem is just to have the cabin light on > the same bus as the beacon. > > Adding an extra gizmo is not the best way to go. > > -------- > Eric M. Jones > www.PerihelionDesign.com <http://www.periheliondesign.com/> > 113 Brentwood Drive > Southbridge, MA 01550 > (508) 764-2072 > emjones(at)charter.net > > > Read this topic online here: > > http://forums.matronics.comnbsp; > > > * > > > * > > ________________________________________________________________________________
Date: Mar 12, 2014
Subject: Re: Timer circuit for led array
From: David Duperron <davedup10(at)gmail.com>
Thanks for the tip DL, I will investigate further. Dave On Wed, Mar 12, 2014 at 11:11 AM, D L Josephson wrote: > dlj04(at)josephson.com> > > There are dome light timer relays for cars > http://www.delcity.net/store/Time-Delay-Relay/p_804415.h_ > 804416.t_1.r_IF1003?gclid=COb_ieijjb0CFc1afgodbl0AUA > > ________________________________________________________________________________
Date: Mar 12, 2014
From: Jeff Luckey <jluckey(at)pacbell.net>
Subject: Re: Timer circuit for led array
Dave,=0A=0AIn case you did not see DL Josepheson's earlier post, here is th e link he provided:=0A=0Ahttp://www.delcity.net/store/Time-Delay-Relay/p_80 4415.h_804416.t_1.r_IF1003?gclid=COb_ieijjb0CFc1afgodbl0AUA=0A=0A=0AI don 't know anything more about the relay than what's shown at that link, but I imagine using the relay is pretty straightforward.- If you have specific questions, I and other listers will be happy to try and answer them.=0A=0A -Jeff=0A=0A=0A=0A________________________________=0A From: David Duperron < davedup10(at)gmail.com>=0ATo: aeroelectric-list(at)matronics.com =0ASent: Wednesd ay, March 12, 2014 6:21 PM=0ASubject: Re: AeroElectric-List: Re: Timer circ uit for led array=0A =0A=0A=0AThanks for the reply Jeff. Do you have any mo re info on the automotive light delay relay? Dave Duperron -(davedup10@gm ail.com.=0A=0A=0A=0AOn Wed, Mar 12, 2014 at 1:06 PM, Jeff Luckey wrote:=0A=0ATrue but ... =0A>=0A>that overlooks the convenience of always-available cabin lighting without having the Master on.- =0A> =0A>The post earlier today that referenced an existing automotive delay rel ay looks like a great solution - cheap, designed for the purpose, plug-n-pl ay, no re-inventing the wheel =0A>=0A>- what don't we like about that=0A> =0A>=0A>=0A>-Jeff=0A>=0A>=0A>=0A>________________________________=0A> From: Eric M. Jones =0A>To: aeroelectric-list(at)matronics.com =0A>Sent: Wednesday, March 12, 2014 9:42 AM=0A>Subject: AeroElectric-List: Re: Timer circuit for led array=0A> =0A>=0A>--> AeroElectric-List message posted by: "Eric M. Jones" =0A>=0A>=0A>>- Timer circ uit for led array--- Reply with quote=0A>> I'm looking for a timer to shut off my cabin overhead utility light after about 10 minutes of operati on. It's fed directly from the main battery buss and I would like to know t hat if I forgot to turn it off the timer would do it for me before draining the battery. Any ideas? David Duperron =0A>=0A>=0A>I learned to fly in Ces snas, and as I recall leaving the beacon on was a certain way to avoid walk ing away from the airplane with something still on.=0A>Perhaps the solution to your problem is just to have the cabin light on the same bus as the bea con.=0A>=0A>Adding an extra gizmo is not the best way to go.=0A>=0A>------- -=0A>Eric M. Jones=0A>www.PerihelionDesign.com=0A>113 Brentwood Drive=0A>So uthbridge, MA 01550=0A>(508) 764-2072=0A>emjones(at)charter.net=0A>=0A>=0A> =0A>=0A>Read this topic online here:=0A>=0A>http://forums.matronics.comnbsp ; - - - - - - - - - -==0A>=0A>=0A>=0A>=0A>=0A>=0A>ist " target="_blank">http://www.matronics.com/Navigator?AeroElectric-List=0A tp://forums.matronics.com=0A_blank">http://www.matronics.com/contribution ===================== ________________________________________________________________________________
From: "Roger & Jean" <mrspudandcompany(at)verizon.net>
Subject: Re: Timer circuit for led array
Date: Mar 13, 2014
I haven't done a survey of the entire automotive world, but it seems as if every car today has convenience lighting that turns on the interior lights when the doors are unlocked and again when the car is turned off. On my 2006 Toyota truck, they turn off when the doors are locked, either with the switch on the door or with the remote. Perhaps a You Pick car wrecker would be a good place to grab a circuit whole for a relative cheap price. I believe that you will find that most of the delays etc are controlled through the car's computer and not with individual delay circuits. I could be wrong, have been on occasion. Roger ________________________________________________________________________________
From: Craig Reding <clr(at)redingaviation.com>
Subject: spliting 2 wires at a dsub connector
Date: Mar 13, 2014
In the sprit of the comic books that Bob has put together which have proven so valuable I need to split a shielded wire into 2 pins on on a DSUB. I am sure that what I did would be an acceptable approach but is there a better way? this is a 2 conductor shielded wire. I have already added a pigtail using a solder sleeve thanks for the help I striped off a 1/4 inch strip of insulation about 1 1/2 inches back from the pin I following Bob=92s comic book on splicing wires i teased out a couple of strands of the wire to be joined Wrapped and soldered added shrink wrap and crimped on the pin ________________________________________________________________________________
From: "Jay Hyde" <jay(at)horriblehyde.com>
Subject: spliting 2 wires at a dsub connector
Date: Mar 15, 2014
That's a perfectly acceptable way to do it; I have used that method many times without any problems. Jay From: owner-aeroelectric-list-server(at)matronics.com [mailto:owner-aeroelectric-list-server(at)matronics.com] On Behalf Of Craig Reding Sent: 13 March 2014 11:46 PM Subject: AeroElectric-List: spliting 2 wires at a dsub connector In the sprit of the comic books that Bob has put together which have proven so valuable I need to split a shielded wire into 2 pins on on a DSUB. I am sure that what I did would be an acceptable approach but is there a better way? this is a 2 conductor shielded wire. I have already added a pigtail using a solder sleeve thanks for the help I striped off a 1/4 inch strip of insulation about 1 1/2 inches back from the pin I following Bob's comic book on splicing wires i teased out a couple of strands of the wire to be joined Wrapped and soldered added shrink wrap and crimped on the pin ________________________________________________________________________________
Subject: Re: spliting 2 wires at a dsub connector
From: "Craig L. Reding" <clr(at)redingaviation.com>
Date: Mar 15, 2014
Jay Thanks > On Mar 15, 2014, at 4:47 AM, "Jay Hyde" wrote: > > That=99s a perfectly acceptable way to do it; I have used that metho d many times without any problems. > > Jay > > > From: owner-aeroelectric-list-server(at)matronics.com [mailto:owner-aeroelect ric-list-server(at)matronics.com] On Behalf Of Craig Reding > Sent: 13 March 2014 11:46 PM > To: aeroelectric-list(at)matronics.com > Subject: AeroElectric-List: spliting 2 wires at a dsub connector > > > In the sprit of the comic books that Bob has put together which have prov en so valuable > > I need to split a shielded wire into 2 pins on on a DSUB. I am sure that w hat I did would be an acceptable approach but is there a better way? > this is a 2 conductor shielded wire. I have already added a pigtail using a solder sleeve > > thanks for the help > > > I striped off a 1/4 inch strip of insulation about 1 1/2 inches back from t he pin > I > > following Bob=99s comic book on splicing wires i teased out a couple of strands of the wire to be joined > > > Wrapped and soldered > > > added shrink wrap and crimped on the pin > > > ________________________________________________________________________________
Subject: Lithium batteries
From: Thomas E Blejwas <tomblejwas(at)yahoo.com>
Date: Mar 15, 2014
Bob, Don't know which message it was in, but you mentioned that you we were working on the last battery article for Kitplanes that would include lithium batteries. I hope you will look at the EarthX batteries. They are now recommended on the Viking website, so I've read their promotions. They actually have a section on their website for experimental aircraft applications and they claim to deal with charging/discharging issues that other lithium-iron-phosphate batteries don't. I learned the hard way (had a meter lead plugged into the wrong socket) that a brief inadvertent short of a Shorai battery will ruin it. Still haven't heard of any explosive failures of batteries with this chemistry. I look forward to your article. Tom Sent from my iPad ________________________________________________________________________________
Subject: Re: spliting 2 wires at a dsub connector
From: "Eric M. Jones" <emjones(at)charter.net>
Date: Mar 15, 2014
For frequencies above about a MHz, this shouldn't be done, since the insulation itself carries the signal. See attached. By the way. I ran across a paper that recommended wire-nuts for connecting multiple wires. Is there some reason not to do this? -------- Eric M. Jones www.PerihelionDesign.com 113 Brentwood Drive Southbridge, MA 01550 (508) 764-2072 emjones(at)charter.net Read this topic online here: http://forums.matronics.com/viewtopic.php?p=420383#420383 Attachments: http://forums.matronics.com//files/dabbling_with_electricity_565.pdf ________________________________________________________________________________
Date: Mar 15, 2014
From: "Robert L. Nuckolls, III" <nuckolls.bob(at)aeroelectric.com>
Subject: Re: Lithium batteries
At 12:35 PM 3/15/2014, you wrote: > > >Bob, > >Don't know which message it was in, but you mentioned that you we >were working on the last battery article for Kitplanes that would >include lithium batteries. I hope you will look at the EarthX >batteries. They are now recommended on the Viking website, so I've >read their promotions. They actually have a section on their >website for experimental aircraft applications and they claim to >deal with charging/discharging issues that other >lithium-iron-phosphate batteries don't. I learned the hard way (had >a meter lead plugged into the wrong socket) that a brief inadvertent >short of a Shorai battery will ruin it. Still haven't heard of any >explosive failures of batteries with this chemistry. I look forward >to your article. > >Tom Excellent input . . . thanks. I'll add them to the 'research'. So far I've submitted three articles that speak to battery selection and operations in general. The 4th is taking longer than I thought . . . input from folks like yourself have been helping me peel away the layers of the onion . . . In the mean time, I'm submitting an article on mean time between failure and how it has little or nothing to do with system reliability. The article will be a prelude to chapter 4 on lithium batteries where I'll remind readers that irrespective of their battery choice, the electrically dependent engine places new requirements on batteries. Builders they need to look past the marketing rhetoric for weight, cranking ability and any mumbling that speaks to 'safety'. It's unfortunate that all the smoke and fire (no pun intended) has pushed performance issues to the back of the bus. The article are to remind builders that they need to purchase to design goals then test and maintain to those goals. A Kitplanes reader wrote Paul Dye to take issue with a statement I made to the effect that I'd never read of an accident where a backup system would have made a difference. The "box" into which the pilot driven was profound and in-escapable. He related the fact that during a flight behind his electrically dependent engine, a wire came loose ad "took down the main bus". He said that the presence of a back-up battery and a constellation of switches to access it saved the day. Through a couple of exchanges I discovered that he had a hefty alternator and a 34 a.h. battery . . . both of which became unavailable to him thus forcing dependance upon a small 'backup' battery. I suggested that while his particular system WAS tolerant of that particular failure, it was not very robust. I hoped to engage him in conversation that would reduce his reliance on a backup battery . . . but he was rather unhappy about my assessment. The point to be pondered is that no matter what kind of battery you have on board, failure tolerance has little to do with choice of parts, their MTBF numbers or their marketing position amongst the stars of products. I'm getting no outpouring of data to support anyone's marketing hype. E-mails are brushed aside with an assertion of no comment, use on airplanes is not supported, or simply ignored. This has been an interesting exercise . . . Bob . . . ________________________________________________________________________________
From: "Roger & Jean" <mrspudandcompany(at)verizon.net>
Subject: Re: spliting 2 wires at a dsub connector
Date: Mar 15, 2014
> > By the way. I ran across a paper that recommended wire-nuts for connecting > multiple wires. Is there some reason not to do this? The main reason that I could see is the added bulk of the wire nuts might make it difficult to impossible to add a cover to the connector. Roger ________________________________________________________________________________
From: "B Tomm" <fvalarm(at)rapidnet.net>
Subject: Lithium batteries
Date: Mar 15, 2014
Bob, I agree that EarthX, based on their forthcoming dialog, substance in their answers, and willingness to support the amateur aircraft builders (even encourage the use of their batteries in our aircraft, and why) has them on my short list for when I finally need a battery for the RV7A. I strongly suggest that you get in touch with them for your article. It will be very easy to have an audience there. Bevan -----Original Message----- From: owner-aeroelectric-list-server(at)matronics.com [mailto:owner-aeroelectric-list-server(at)matronics.com] On Behalf Of Robert L. Nuckolls, III Sent: Saturday, March 15, 2014 12:10 PM Subject: Re: AeroElectric-List: Lithium batteries --> At 12:35 PM 3/15/2014, you wrote: > > >Bob, > >Don't know which message it was in, but you mentioned that you we were >working on the last battery article for Kitplanes that would include >lithium batteries. I hope you will look at the EarthX batteries. They >are now recommended on the Viking website, so I've read their >promotions. They actually have a section on their website for >experimental aircraft applications and they claim to deal with >charging/discharging issues that other lithium-iron-phosphate batteries >don't. I learned the hard way (had a meter lead plugged into the wrong >socket) that a brief inadvertent short of a Shorai battery will ruin >it. Still haven't heard of any explosive failures of batteries with >this chemistry. I look forward to your article. > >Tom Excellent input . . . thanks. I'll add them to the 'research'. So far I've submitted three articles that speak to battery selection and operations in general. The 4th is taking longer than I thought . . . input from folks like yourself have been helping me peel away the layers of the onion . . . In the mean time, I'm submitting an article on mean time between failure and how it has little or nothing to do with system reliability. The article will be a prelude to chapter 4 on lithium batteries where I'll remind readers that irrespective of their battery choice, the electrically dependent engine places new requirements on batteries. Builders they need to look past the marketing rhetoric for weight, cranking ability and any mumbling that speaks to 'safety'. It's unfortunate that all the smoke and fire (no pun intended) has pushed performance issues to the back of the bus. The article are to remind builders that they need to purchase to design goals then test and maintain to those goals. A Kitplanes reader wrote Paul Dye to take issue with a statement I made to the effect that I'd never read of an accident where a backup system would have made a difference. The "box" into which the pilot driven was profound and in-escapable. He related the fact that during a flight behind his electrically dependent engine, a wire came loose ad "took down the main bus". He said that the presence of a back-up battery and a constellation of switches to access it saved the day. Through a couple of exchanges I discovered that he had a hefty alternator and a 34 a.h. battery . . . both of which became unavailable to him thus forcing dependance upon a small 'backup' battery. I suggested that while his particular system WAS tolerant of that particular failure, it was not very robust. I hoped to engage him in conversation that would reduce his reliance on a backup battery . . . but he was rather unhappy about my assessment. The point to be pondered is that no matter what kind of battery you have on board, failure tolerance has little to do with choice of parts, their MTBF numbers or their marketing position amongst the stars of products. I'm getting no outpouring of data to support anyone's marketing hype. E-mails are brushed aside with an assertion of no comment, use on airplanes is not supported, or simply ignored. This has been an interesting exercise . . . Bob . . . ________________________________________________________________________________
From: "Michael McMahon" <mike(at)aeromotogroup.com>
Subject: Lithium batteries
Date: Mar 15, 2014
Mr. Brent Regan is apparently an expert on the use of Lithium batteries in aircraft. I attempted to start a discussion on the LML list a few years ago and he shut me down with his expertise. You may want to tap into his exhaustive research on the subject: http://lancair.net/lists/lml/Message/57030-P.txt Michael -----Original Message----- From: owner-aeroelectric-list-server(at)matronics.com [mailto:owner-aeroelectric-list-server(at)matronics.com] On Behalf Of B Tomm Sent: Saturday, March 15, 2014 6:43 PM Subject: RE: AeroElectric-List: Lithium batteries Bob, I agree that EarthX, based on their forthcoming dialog, substance in their answers, and willingness to support the amateur aircraft builders (even encourage the use of their batteries in our aircraft, and why) has them on my short list for when I finally need a battery for the RV7A. I strongly suggest that you get in touch with them for your article. It will be very easy to have an audience there. Bevan -----Original Message----- From: owner-aeroelectric-list-server(at)matronics.com [mailto:owner-aeroelectric-list-server(at)matronics.com] On Behalf Of Robert L. Nuckolls, III Sent: Saturday, March 15, 2014 12:10 PM Subject: Re: AeroElectric-List: Lithium batteries --> At 12:35 PM 3/15/2014, you wrote: > > >Bob, > >Don't know which message it was in, but you mentioned that you we were >working on the last battery article for Kitplanes that would include >lithium batteries. I hope you will look at the EarthX batteries. They >are now recommended on the Viking website, so I've read their >promotions. They actually have a section on their website for >experimental aircraft applications and they claim to deal with >charging/discharging issues that other lithium-iron-phosphate batteries >don't. I learned the hard way (had a meter lead plugged into the wrong >socket) that a brief inadvertent short of a Shorai battery will ruin >it. Still haven't heard of any explosive failures of batteries with >this chemistry. I look forward to your article. > >Tom Excellent input . . . thanks. I'll add them to the 'research'. So far I've submitted three articles that speak to battery selection and operations in general. The 4th is taking longer than I thought . . . input from folks like yourself have been helping me peel away the layers of the onion . . . In the mean time, I'm submitting an article on mean time between failure and how it has little or nothing to do with system reliability. The article will be a prelude to chapter 4 on lithium batteries where I'll remind readers that irrespective of their battery choice, the electrically dependent engine places new requirements on batteries. Builders they need to look past the marketing rhetoric for weight, cranking ability and any mumbling that speaks to 'safety'. It's unfortunate that all the smoke and fire (no pun intended) has pushed performance issues to the back of the bus. The article are to remind builders that they need to purchase to design goals then test and maintain to those goals. A Kitplanes reader wrote Paul Dye to take issue with a statement I made to the effect that I'd never read of an accident where a backup system would have made a difference. The "box" into which the pilot driven was profound and in-escapable. He related the fact that during a flight behind his electrically dependent engine, a wire came loose ad "took down the main bus". He said that the presence of a back-up battery and a constellation of switches to access it saved the day. Through a couple of exchanges I discovered that he had a hefty alternator and a 34 a.h. battery . . . both of which became unavailable to him thus forcing dependance upon a small 'backup' battery. I suggested that while his particular system WAS tolerant of that particular failure, it was not very robust. I hoped to engage him in conversation that would reduce his reliance on a backup battery . . . but he was rather unhappy about my assessment. The point to be pondered is that no matter what kind of battery you have on board, failure tolerance has little to do with choice of parts, their MTBF numbers or their marketing position amongst the stars of products. I'm getting no outpouring of data to support anyone's marketing hype. E-mails are brushed aside with an assertion of no comment, use on airplanes is not supported, or simply ignored. This has been an interesting exercise . . . Bob . . . ________________________________________________________________________________
Date: Mar 15, 2014
From: "Robert L. Nuckolls, III" <nuckolls.bob(at)aeroelectric.com>
Subject: Re: spliting 2 wires at a dsub connector
At 01:27 PM 3/15/2014, you wrote: > >For frequencies above about a MHz, this shouldn't be done, since the >insulation itself carries the signal. See attached. I'd like to see the physics of this explained. Yes, there is a skin-effect that pushes the current flow to the surface of the conductor http://tinyurl.com/n2h9ljy . . . but carried on the insulation? >By the way. I ran across a paper that recommended wire-nuts for >connecting multiple wires. Is there some reason not to do this? Wire nuts? Like this? [] Not recommended for aircraft Bob . . . ________________________________________________________________________________
Date: Mar 15, 2014
From: "Robert L. Nuckolls, III" <nuckolls.bob(at)aeroelectric.com>
Subject: spliting 2 wires at a dsub connector
> > >I need to split a shielded wire into 2 pins on on a DSUB. I am sure >that what I did would be an acceptable approach but is there a better way? >this is a 2 conductor shielded wire. I have already added a pigtail >using a solder sleeve > >thanks for the help Works good, lasts a long time . . . Bob . . . ________________________________________________________________________________
Subject: Re: spliting 2 wires at a dsub connector
From: Craig Reding <clr(at)redingaviation.com>
Date: Mar 16, 2014
Bob, Thank you very much for all of your lessons! On Mar 15, 2014, at 10:59 PM, Robert L. Nuckolls, III wrote: > > >> >> >> I need to split a shielded wire into 2 pins on on a DSUB. I am sure that what I did would be an acceptable approach but is there a better way? >> this is a 2 conductor shielded wire. I have already added a pigtail using a solder sleeve >> >> thanks for the help > > > > > > Works good, lasts a long time . . . > > > Bob . . . > > > > ________________________________________________________________________________
Date: Mar 17, 2014
From: "Robert L. Nuckolls, III" <nuckolls.bob(at)aeroelectric.com>
Subject: Lithium batteries
At 08:53 PM 3/15/2014, you wrote: > > >Mr. Brent Regan is apparently an expert on the use of Lithium batteries in >aircraft. I attempted to start a discussion on the LML list a few years ago >and he shut me down with his expertise. You may want to tap into his >exhaustive research on the subject: >http://lancair.net/lists/lml/Message/57030-P.txt Brent and I have crossed paths in the past but I'm having trouble recalling now when, where and in what context. I'll drop him a note and see if he would be willing to collaborate on the last installment of the series on batteries. Thanks for the heads-up! Bob . . . ________________________________________________________________________________
Subject: New Over-Voltage Protection Architecture
From: "user9253" <fransew(at)gmail.com>
Date: Mar 17, 2014
Suppose that it is not desired that a crowbar type over-voltage protection device to have the ability to disable the alternator on an aircraft with an electrically dependent engine or electrically dependent instruments. In the event that an externally regulated alternator puts out too high voltage, is it feasible for the O.V. protection device to insert a series resistance into the alternator field circuit instead of shorting it out? The amount of resistance could be determined through trial and error to give 12-13 volts with normal aircraft loads. Or a resistor could be permanently installed in the alternator field circuit, but normally shorted out by the O.V. protection device. In case of high voltage or O.V. protection malfunction, the short across the resistor will be opened. The alternator will then operate at reduced output determined by the load. A low voltage warning will be indicated to the pilot who can increase or decrease aircraft loads to fine tune the system voltage if desired. No circuit breaker will pop nor will a fuse blow. The pilot or mechanic can troubleshoot the low voltage condition after landing. The advantage of this method of over-voltage protection is that the alternator output will be reduced instead of completely disabled. Question is, is it practical and feasible? Joe -------- Joe Gores Read this topic online here: http://forums.matronics.com/viewtopic.php?p=420469#420469 ________________________________________________________________________________
Date: Mar 17, 2014
From: "Robert L. Nuckolls, III" <nuckolls.bob(at)aeroelectric.com>
Subject: EarthX lithium
I just became aware of EarthX batteries based on comments here on the list. I visited their website and studied a variety of assertions and comparisons designed to promote sales. At first blush, these folks are hanging their marketing hats on the same philosophy as other suppliers of lithium technologies. See attached document . . . The DEARTH of good engineering data for these products makes it difficult if not impossible to craft a well considered integration of the current COTS (commericial off the shelf) offerings onto airplanes. I had to delay submission of the last of four Kitplanes articles for a month . . . not because theres a paucity of choices . . . but because the folks selling them can't define performance in rational numbers. Still digging . . . Bob . . . ________________________________________________________________________________
Date: Mar 17, 2014
From: "Robert L. Nuckolls, III" <nuckolls.bob(at)aeroelectric.com>
Subject: Re: New Over-Voltage Protection Architecture
At 11:26 AM 3/17/2014, you wrote: Suppose that it is not desired that a crowbar type over-voltage protection device to have the ability to disable the alternator on an aircraft with an electrically dependent engine or electrically dependent instruments. In the event that an externally regulated alternator puts out too high voltage, is it feasible for the O.V. protection device to insert a series resistance into the alternator field circuit instead of shorting it out? The amount of resistance could be determined through trial and error to give 12-13 volts with normal aircraft loads. Why not just switch in a stand-by regulator? http://tinyurl.com/npya5l7 . . .they're really cheap . . . In the early days of the alternator fitted C337 and Barons, controlled stand-by regulators were part of the system . . . Or a resistor could be permanently installed in the alternator field circuit, but normally shorted out by the O.V. protection device. In case of high voltage or O.V. protection malfunction, the short across the resistor will be opened. The alternator will then operate at reduced output determined by the load. A low voltage warning will be indicated to the pilot who can increase or decrease aircraft loads to fine tune the system voltage if desired. No circuit breaker will pop nor will a fuse blow. The pilot or mechanic can troubleshoot the low voltage condition after landing. The advantage of this method of over-voltage protection is that the alternator output will be reduced instead of completely disabled. Question is, is it practical and feasible? Pretty tough to sell . . . a voltage regulator is an exceedingly agile device that will willingly produce any field excitation level from near zero to full bus voltage depending on loads and rpm. You could conduct some experiments on your airplane to deduce the size of such a resistor but . . . but be sure to explore the "non nominal" conditions to assess performance. Bob . . . ________________________________________________________________________________
Date: Mar 17, 2014
From: "Robert L. Nuckolls, III" <nuckolls.bob(at)aeroelectric.com>
Subject: EarthX lithium
Skip, Had a reader recommend EarthX lithium batteries for airplanes. Seems they're courting Van's and some other suppliers of kit aircraft and parts. Downloaded a page of their marketing pitch. I commented on it and published it back to my forum. I'm going to contact EarthX for more information on ENERGY numbers and a functional description of their 'battery management system'. See attached . . . Bob . . . ________________________________________________________________________________
Subject: Re: New Over-Voltage Protection Architecture
From: "user9253" <fransew(at)gmail.com>
Date: Mar 17, 2014
> Why not just switch in a stand-by regulator? > http://tinyurl.com/npya5l7 > . . .they're really cheap . . . > Great idea. Thanks Bob My RV-12 has a permanent magnet alternator. So I can not try out the series resistor idea. I like to think about alternative ways of wiring an airplane. Most of the time, the old way is the best way. :-) Joe -------- Joe Gores Read this topic online here: http://forums.matronics.com/viewtopic.php?p=420475#420475 ________________________________________________________________________________
Subject: Re: EarthX lithium
From: Robert Borger <rlborger(at)mac.com>
Date: Mar 17, 2014
Bob N, In your commentary on the FAQ. At the end there is a question about deeply discharging the battery. Your comment is So if I go off and leave the master switch ON, odds are that by the time I get back to fly again, my battery is trash? I can answer that in one word - YES. Your battery is trash. If this battery is like the one I purchased to test, it does not have a BMS which auto-disconnects to protect cells from over-discharge. I flew with this battery for several months and did some simple ground testing. Tested things like - How long would it last running all the electrics (EIS, EFIS, radio & transponder), how long just running the EIS, radio & transponder, how long just running the EIS & radio and how long just running the EIS & transponder? After each test I put the high-tech charger on and recharged with the cell balance in effect. I didnt get to the last of the questions because after the EIS, radio & transponder test I managed to put the charger on but left the Master on as well. When I came back a couple days later to continue the testing the battery was fully discharged and would not accept recharging. It was trashed. FYI, in testing the battery I found that it would run the whole electrical system about 5 minutes. It would run the EIS, radio & transponder about 7 minutes. Overall, I was not impressed with the LiFePo battery. It was extremely light and it started my Rotax 914 quickly and easily. After starting the engine and flying for a while, it would be properly and fully charged when back on the ground. Unfortunately, from my limited testing, it didnt have any real legs to run things on its own and it requires a lot of fancy, high-tech, gear to keep it happy. And take great care not discharge it fully. Just my 2 from my very limited experience. Blue skies & tailwinds, Bob Borger Europa XS Tri, Rotax 914, Airmaster C/S Prop (50 hrs). Little Toot Sport Biplane, Lycoming Thunderbolt AEIO-320 EXP 3705 Lynchburg Dr. Corinth, TX 76208-5331 Cel: 817-992-1117 rlborger(at)mac.com On Mar 17, 2014, at 11:40 AM, Robert L. Nuckolls, III wrote: I just became aware of EarthX batteries based on comments here on the list. I visited their website and studied a variety of assertions and comparisons designed to promote sales. At first blush, these folks are hanging their marketing hats on the same philosophy as other suppliers of lithium technologies. See attached document . . . The DEARTH of good engineering data for these products makes it difficult if not impossible to craft a well considered integration of the current COTS (commericial off the shelf) offerings onto airplanes. I had to delay submission of the last of four Kitplanes articles for a month . . . not because theres a paucity of choices . . . but because the folks selling them can't define performance in rational numbers. Still digging . . . Bob . . . ________________________________________________________________________________
Date: Mar 17, 2014
From: "Robert L. Nuckolls, III" <nuckolls.bob(at)aeroelectric.com>
Subject: Re: EarthX lithium
At 12:46 PM 3/17/2014, you wrote: Bob N, In your commentary on the FAQ. At the end there is a question about deeply discharging the battery. Your comment is =93So if I go off and leave the master switch ON, odds are that by the time I get back to fly again, my battery is trash? I can answer that in one word - YES. Your battery is trash. If this battery is like the one I purchased to test, it does not have a =93BMS=94 which auto-disconnects to protect cells from over-discharge. Which part number of battery did you test? I flew with this battery for several months and did some simple ground testing. Tested things like - How long would it last running all the electrics (EIS, EFIS, radio & transponder), how long just running the EIS, radio & transponder, how long just running the EIS & radio and how long just running the EIS & transponder? After each test I put the high-tech charger on and recharged with the cell balance in effect. I didn=92t get to the last of the questions because after the EIS, radio & transponder test I managed to put the charger on but left the Master on as well. When I came back a couple days later to continue the testing the battery was fully discharged and would not accept recharging. It was trashed. Good data point . . . FYI, in testing the battery I found that it would run the whole electrical system about 5 minutes. It would run the EIS, radio & transponder about 7 minutes. Overall, I was not impressed with the LiFePo battery. It was extremely light and it started my Rotax 914 quickly and easily. After starting the engine and flying for a while, it would be properly and fully charged when back on the ground. Unfortunately, from my limited testing, it didn=92t have any real legs to run things on its own and it requires a lot of fancy, high-tech, gear to keep it happy. And take great care not discharge it fully. Just my 2=A2 from my very limited experience. From your limited observations, what would you estimate the energy content to be for the model you tested? In other words, what was the demand for the EIS, radio and transponder? Apparently it would satisfactorily crank an engine for some time. I have emailed EarthX requesting the name and address for an individual who can supply technical date in detail sufficient to make good system integration choices. Thanks for the data points! Bob . . . ________________________________________________________________________________
Date: Mar 17, 2014
From: "Robert L. Nuckolls, III" <nuckolls.bob(at)aeroelectric.com>
Subject: Re: New Over-Voltage Protection Architecture
>Great idea. Thanks Bob >My RV-12 has a permanent magnet alternator. So I can not try out >the series resistor idea. I like to think about alternative ways of >wiring an airplane. Most of the time, the old way is the best way. ALL ways are worthy of considered thought. It's just as valuable to know what DOES work as those things which are QUESTIONABLE. I've often asked builders to get on the List and tell us what was tried that didn't work . . . like Bob's posting on EarthX . . . It helps avoid discovering a bad idea over and over again! Bob . . . ________________________________________________________________________________
Subject: Re: EarthX lithium
From: Robert Borger <rlborger(at)mac.com>
Date: Mar 17, 2014
Bob N, LiFePo battery - AeroVoltz 8 Cell Lithium Battery. http://aerovoltz.net/en/ The Sport EFIS consumes about 1 amp according to the documentation. I don=92t have good numbers on the GRT EIS4000 but I would allow about 1 amp with all the sensors, back light, display, etc. I may be able to get a better number next trip to the hanger. The radio and transponder are MicroAir M760Q & T2000 SFL. =46rom the documentation, I=92d say they each use about 250-300ma with the backlight on and not transmitting. The battery turned the 914 over very smartly. The engine is easy to start in any case so it wasn=92t really working that hard. Three or 4 prop blades and it starts. That=92s 2 or 3 rotations of the engine. Blue skies & tailwinds, Bob Borger Europa XS Tri, Rotax 914, Airmaster C/S Prop (50 hrs). Little Toot Sport Biplane, Lycoming Thunderbolt AEIO-320 EXP 3705 Lynchburg Dr. Corinth, TX 76208-5331 Cel: 817-992-1117 rlborger(at)mac.com On Mar 17, 2014, at 1:17 PM, Robert L. Nuckolls, III wrote: At 12:46 PM 3/17/2014, you wrote: Bob N, In your commentary on the FAQ. At the end there is a question about deeply discharging the battery. Your comment is =93So if I go off and leave the master switch ON, odds are that by the time I get back to fly again, my battery is trash? I can answer that in one word - YES. Your battery is trash. If this battery is like the one I purchased to test, it does not have a =93BMS=94 which auto-disconnects to protect cells from over-discharge. Which part number of battery did you test? I flew with this battery for several months and did some simple ground testing. Tested things like - How long would it last running all the electrics (EIS, EFIS, radio & transponder), how long just running the EIS, radio & transponder, how long just running the EIS & radio and how long just running the EIS & transponder? After each test I put the high-tech charger on and recharged with the cell balance in effect. I didn=92t get to the last of the questions because after the EIS, radio & transponder test I managed to put the charger on but left the Master on as well. When I came back a couple days later to continue the testing the battery was fully discharged and would not accept recharging. It was trashed. Good data point . . . FYI, in testing the battery I found that it would run the whole electrical system about 5 minutes. It would run the EIS, radio & transponder about 7 minutes. Overall, I was not impressed with the LiFePo battery. It was extremely light and it started my Rotax 914 quickly and easily. After starting the engine and flying for a while, it would be properly and fully charged when back on the ground. Unfortunately, from my limited testing, it didn=92t have any real legs to run things on its own and it requires a lot of fancy, high-tech, gear to keep it happy. And take great care not discharge it fully. Just my 2=A2 from my very limited experience. =46rom your limited observations, what would you estimate the energy content to be for the model you tested? In other words, what was the demand for the EIS, radio and transponder? Apparently it would satisfactorily crank an engine for some time. I have emailed EarthX requesting the name and address for an individual who can supply technical date in detail sufficient to make good system integration choices. Thanks for the data points! Bob . . . ________________________________________________________________________________
Date: Mar 18, 2014
From: "Robert L. Nuckolls, III" <nuckolls.bob(at)aeroelectric.com>
Subject: RE: EarthX battery data
At 01:17 PM 3/17/2014, you wrote: >Dear Bob, > >I will forward this email to our engineering department so they can answer >your questions on a more technical level. I did review your question about >the conflicting statements and I am not sure I understand what is >conflicting. But maybe this will clarify....you can put gasoline in a fire >proof container, but if it is unfortunate enough to be involved in a fire, >it can still catch fire. We have a BMS designed to protect from overcharge >in the applications it is designed for, but if someone uses something that >is in absolute conflict to how our product is designed, it could be a >problem. Understand. Where in your marketing literature are there defined limits beyond which the BMS is unable to prevent catastrophic failure? In one document I am warned that inappropriate treatment of EarthX batteries raises risks of "rupture and/or fire" but there are no qualified or quantified limits to what that treatment might be. As a designer tasked with evaluating performance, cost of ownership and risks associated with incorporation of your offerings into my client's system. When I compare this page from your website . . . Emacs! . . . with this data published elsewhere Emacs! I perceive a disconnect between EarthX claim for an 80% lighter battery (1/5th the weight of a lead-acid). Technical literature for comparative studies of lead-acid vs. lithium suggest that weigh savings is on the order of 66 percent weight reduction, not 80 percent. Is EarthX claiming some breakthrough in performance that offers a still lighter battery? Further, the top bubble in the image from your website speaks to a BMS in terms that seems to imply a bullet-proof design . . . a feature that would be very attractive to my client . . . as long as the limits to external abuse are quantified and the behaviors of the BMS are known. >Not sure if that answers your question but I have forwarded this on. >Sincerely, > >Kathy Nicoson >Global Sales Director >(970) 301-6064 >Fax: (970) 674-9544 >www. Earthxmotorsports.com >sales(at)earthxmotorsports.com I've worked with lead-acid, ni-cad and ni-mh batteries for decades and there are few if any features of performance that are not well known for these chemistries. Much is known about lithium chemistries but the numbers and bullet-points from literature I've read so far on EarthX products leaves many questions unanswered. I'd really need to understand more about the physics and design of your products before I can recommend them into my client's application. I appreciate your willingness to be of assistance. Kindest regards, Bob Nuckolls AeroElectric Connection Medicine Lodge, Kansas >-----Original Message----- >From: Robert L. Nuckolls, III [mailto:nuckolls.bob(at)aeroelectric.com] >Sent: Monday, March 17, 2014 11:54 AM >To: sales(at)earthxmotorsports.com >Subject: EarthX battery data > >Good afternoon, > >By way of introduction I am a consulting engineer with a private practice in >a variety of markets. > >I retired out of Beech Aircraft in 2007 after 45+ years experience in DC >vehicular power systems, communications, failure analysis and system >integration. > >I have a client who has asked me to explore your products suitability to >task in a new program. > >I need to understand how your battery management system functions. Does it >disconnect the battery from the system when in danger of over charging or >excessive discharging? At what voltage levels to these features assert >themselves? > >I am confused by what appears to be conflicting statements on your website. >For example: On these two pages > >http://tinyurl.com/qex9gbg > >http://tinyurl.com/nxzf7r2 > >I see statements that imply management of battery overcharge events by the >BMS . . . but on the other page, we are cautioned about mis-applied chargers >offer risks for battery rupture and/or fire. > >If an EarthX battery is inadvertently discharged too deeply, I presume the >BMS will shut off current flow below some level . . . what are the limits >for remaining in this discharged state in terms of recovering battery >performance? Your website literature speaks to 4000 cycles . . . what depth >of discharge is tolerated by EarthX batteries to meet the 4000 number? What >capacity fraction used to benchmark end of life after 4000 cycles? > >I'm seeking engineering test data on energy content and internal impedance >of the various EarthX products. In particular, a data sheet not unlike this >document describing performance for a Panasonic SVLA battery . . . > >http://tinyurl.com/pjgzewx > >This format and detail of data will allow me to service my customer with a >well considered decision on EarthX products for suitability to task. > >It would be very useful to have a contact name and email for an individual >at EarthX who can supply needed data and carry on a dialog for any future >questions. > >Kindest regards, Bob . . . ________________________________________________________________________________
Date: Mar 18, 2014
From: "Robert L. Nuckolls, III" <nuckolls.bob(at)aeroelectric.com>
Subject: Re: EarthX lithium
At 02:01 PM 3/17/2014, you wrote: Bob N, LiFePo battery - AeroVoltz 8 Cell Lithium Battery. http://aerovoltz.net/en/ The Sport EFIS consumes about 1 amp according to the documentation. I don't have good numbers on the GRT EIS4000 but I would allow about 1 amp with all the sensors, back light, display, etc. I may be able to get a better number next trip to the hanger. The radio and transponder are MicroAir M760Q & T2000 SFL. From the documentation, I'd say they each use about 250-300ma with the backlight on and not transmitting. The battery turned the 914 over very smartly. The engine is easy to start in any case so it wasn't really working that hard. Three or 4 prop blades and it starts. That's 2 or 3 rotations of the engine. Okay, that's not an EarthX battery which claims to have a battery management system built in. From the physical sizes of these batteries, I suspect they are assembled from individual cells not unlike the 26650 styles offered by A123 and others. http://tinyurl.com/ktyfbnp These would be 'rated' at something on the order of 2.3 to 2.5aH per cell. A 2-row, 8-cell assembly would offer a 12V battery with a capacity on the order of 5aH. A number which is inconsistent with your battery only endurance experience. Their 16-cell product would have a cell weight of 75g x 16 or 1.2Kg. Their catalog weight for the 16 cell product is 1.6Kg which leaves 0.4Kg for enclosure . . . sounds about right. So their 16-cell product would clock in with a 10aH legacy rating and 1.2Kg x 125Wh/Kg or 150Wh of total energy. Given the relatively flat discharge curves of lithium chemistries (low internal impedance) one might estimate that 125Wh will be available at a 10A discharge rate. So it seems that the 16-cell device would run a 100W demand engine for 1.2 hours when new and 1 hour at end of life. Let's get some hard numbers on the current draw for your experiments and see why you didn't get better performance from your '5aH' product. Aerovolts doesn't claim to offer an internal BMS but they do offer the external BMS charger with words like: ------------------------------------ The Aerovoltz Battery Management System Balance Charger is the only charger on the market designed specifically to work with the Aerovoltz battery. The primary advantage to the Aerovoltz BMS Blance Charger is the balance charging function. The Balance mode balances the voltage of each cell or cell pack while charging. By balance charging your Aerovoltz Performance Battery you insure that the battery is operating at its maximum power. Periodic balance charging can also double the expected life of your battery. This charger is not required to operate your Battery, but it is highly recommended. The only charger made to work with the Battery Management System. Plugs directly into the batteries BMS port. Able to balance charge all the cells individually for max power and durability. Can double the expected life of the battery. Has Charge, Fast Charge, Balance Charge, and Storage Charge Functions. Available in 120V (US) and 240V (EUR). ------------------------------------- The problem I have with these special chargers is rooted in the fact that when we put an airplane away, the battery is generally topped off . . . It's my understanding that these super-smart chargers work their magic while recharging a partially to completely depleted battery. How then would such a charger benefit the OBAM aircraft owner/operator? . . . and DOUBLE the life of the battery? THAT'S a VERY LONG POLE in the tent for cost-of-ownership for a battery that is already VERY expensive compared to the device it purports to replace. Aerovolts also claims: ----------------------------------------- The 16 Cell has a massive 500PCA and 28 Pbeq AHs at only 3 lbs of weight. The 16 Cell is a direct replacement of the Yuasa YTX30L-BS, GYZ-20HL, and YTX24HL-BS. ----------------------------------------- The Yuasa YTX30L is a 22 pound, SVLA battery that would clock in at about 24aH . . . I submit that while the 16-cell Aerovolts product may CRANK like the Yuasa battery, it is NOT A DIRECT REPLACEMENT when it comes to keeping the fires lit up front . . . in fact, it's probably less than half the capacity of the Yuasa battery. Emacs! We'll keep asking questions . . . Bob . . . ________________________________________________________________________________
Date: Mar 18, 2014
From: "Robert L. Nuckolls, III" <nuckolls.bob(at)aeroelectric.com>
Subject: Lithium saga slogs ever onward . . .
I downloaded the detail spec sheet for A123 26650 cells at . . . http://tinyurl.com/n73f9gj This cell and others like it seem to be the core ingredient of choice for many of the off-the-shelf battery products that are cropping up everywhere. There's an interesting graphic in this data sheet . . . Emacs! Notice that the family of curves acknowledges the fact that this cell, like all other sources of energy, has a source impedance. This is illustrated by the fact that heavier loads produce initial plots on the curve that are successively lower in voltage as the load current rises. I.e., the higher the load, the higher will be I(squared)R energy losses due to internal heating. But then something magic happens. All three plots come together at just under 2.5aH . . . gee . . . where did all those wasted Watt-seconds go during discharge but magically showed up again before the cell dropped below 2V? Can't believe much that you read in the papers . . . gotta be careful about data sheets too . . . Bob . . . ________________________________________________________________________________
Subject: Re: New Over-Voltage Protection Architecture
From: "Eric M. Jones" <emjones(at)charter.net>
Date: Mar 18, 2014
Joe, Before you reinvent anything, let me show you my Proposed Perihelion Power Protector (attached). Repost from 20JUL07: (time flys...) I talk to many builders who are particularly concerned about frying their expensive avionics and glass panels. These parts are typically 1/3 of the airplane cost. Planning the wiring of a small aircraft involves installing systems to safeguard the buses against failure of the alternator. Either an internally regulated alternator or its more adaptable brother with an external regulator still has the potential problem of a runaway condition, as well as a short circuit of the main battery to alternator B-lead, and frequent load dump OV conditions. A solution is herein proposed Linear Technology has introduced a clever device, the LT4356-1 Overvoltage Protection Regulator and Inrush Limiter to address all these worries. Not only thatthey list Automotive/Avionic Surge Protection is one of its chief applications. The LTC approach takes the high voltage from a load dump or failed alternator and REGULATES it while the situation is being tamed. The IC package is 3mm X 4mm and drives a big N-MOSFET that does the heavy work of regulating the voltage. So what happens? Normally the alternator current is carried by the fully on N-MOSFET. Assume the current is 50A, and the Rds(on) is 0.012 ohms. The dissipation would be only 30W. This is easy to handle with a reasonable heat sink. Assume the alternator and/or the regulator goes cuckoo. This could happen if the alternator field winding shorts to the B-lead output or the sense lead in the regulator opens, or other untoward goings-on. The alternator output goes into an upward voltage spiral. This voltage is not unlimited, especially if there is a load on it, but could be 80 VDC. Now the LT4356-1 REGULATES the output via the big N-MOSFET on a heat sink. The aircraft buses never sees more than 14.5 Volts (or whatever is desired). This is true for the short time (500 mS) load dumps, and it will regulate a runaway alternator for a time determined by whatever the N-MOSFET and heat sink can dissipate. The LT4356-1 has a timer circuit to protect the N-MOSFET. If the time expires and the stress continues, the fault warning signals an impending power-down and the N-MOSFET shuts off the B-line. Additionally, some models of the LT4356-1 have a spare amplifier/comparator that can be used for any purpose (not shown here). It operates from 4 to 80 VDC, and withstands 30V and up to 100V. A series low voltage P-MOSFET or a Schottky diode can be added to protect against a shorted alternator that would draw reverse current. The device has built-in protection again high current shorts of the B-Line to the battery. The drawing shows a basic form. A few additional parts are necessary. However, consider the parts that WOULDNT be necessary: The B-Line contactor, The OV monitor, The load dump preventer, The B-Line fuse, The alternator switch. No crow bar or linear over voltage switch. I dont have current plans to market this but will build one for myself. Discussion is invited. See attachment for a pdf version of this note with drawing. -------- Eric M. Jones www.PerihelionDesign.com 113 Brentwood Drive Southbridge, MA 01550 (508) 764-2072 emjones(at)charter.net Read this topic online here: http://forums.matronics.com/viewtopic.php?p=420541#420541 Attachments: http://forums.matronics.com//files/power_good_a1_176.pdf http://forums.matronics.com//files/power_protector_schematic_383.pdf http://forums.matronics.com//files/power_protector_revb_419.pdf ________________________________________________________________________________
Date: Mar 18, 2014
From: "Robert L. Nuckolls, III" <nuckolls.bob(at)aeroelectric.com>
Subject: Re: New Over-Voltage Protection Architecture
At 08:17 AM 3/18/2014, you wrote: Joe, Before you reinvent anything, let me show you my Proposed Perihelion Power Protector (attached). Repost from 20JUL07: (time flys...) I talk to many builders who are particularly concerned about frying their expensive avionics and glass panels. These parts are typically 1/3 of the airplane cost. Planning the wiring of a small aircraft involves installing systems to safeguard the buses against failure of the alternator. Either an internally regulated alternator or its more adaptable brother with an external regulator still has the potential problem of a runaway condition, as well as a short circuit of the main battery to alternator B-lead, and frequent load dump OV conditions. "frequent"? Linear Technology has introduced a clever device, the LT4356-1 Overvoltage Protection Regulator and Inrush Limiter to address all these worries. . . . but are they real worries . . . We had an extended discussion on 'load dumps' here on the List back about 2004. A sampling of the work product during those conversations can be accessed at . . . http://tinyurl.com/ngu7ptq Load dump speaks to the naturally occurring response of the engine driven power source, its regulator and perhaps the ship's battery to a sudden reduction of a large load . . . like an air conditioner pump motor, landing gear hydraulic pump motor, etc. Another case supposes that a battery is not present or becomes UNHOOKED during this heavy-load reduction. Not only does the alternator/generator become suddenly UNLOADED . . . the system may also be deprived of it's #1 load dump MITIGATOR . . . the battery. This condition is addressed in DO-160/Mil-STD-704 as a recommendation designing a 14v product to stand off 20V for 1S, 40V for 100mS. Double the voltages for a 28V system. This is the time it takes for an equally qualified OV management system to corral the aberrant energy source and bring the event to a graceful conclusion. You will find the LT4356 cited in the constellation of Google hits in the link cited above. This device is but one of MANY approaches to achieving load-dump robustness in aircraft. I've been testing to DO-160/ Mil-STD-704 for 40+ years and never found it necessary to go to such extremes in parts count to protect the input power port of my proposed appliance. I'm not suggesting that the LT4356 does not perform as advertised but I will suggest that it's an inelegant solution to a problem that been the least of our design challenges for decades. I suggest that load-dump stresses that fall outside the time honored design goals don't happen . . . Recall how many times I've written about the paucity of accidents having root cause in electrical system issues. Of the few which did have electrically fertilized roots, load-dump damage was not among them. Further, how many installation manuals for your $high$ electro- whizzies suggest any need to mitigate transient effects above and beyond the DO-169 stresses to which they have already qualified their product? Can you imagine the cries out of the darkness and renting of clothes should some big name like Garmin hold forth with such a recommendation? Such an assertion would mean one or both of two things (1) DO-160 was found lacking or (2) Garmin was unable to design to DO-160 design goals. You would be less scorned if they predicted that the sun would rise in the north tomorrow or that water molecule of H3O2 is predicted to freeze at 50C. I did a pitch trim controller for the Lear 30/50 series airplanes that would stand off 80 volts for tens if seconds . . . not milliseconds . . . it was no big deal. My biggest challenge in that design was to beat the parts count down in a worry-mitigation watch-dog with about twice the parts count of the controller. Over the years, failures in the system were predominantly in the watch-dog . . . not the controller. The point being that one should be cautious when taking on new worries about spikes, surges, bumps, dumps, wiggles and jiggles on the bus that have been pretty well understood for a long time. We can look back at pyramid builders and marvel at their cleverness and knowledge . . . but for them it was all in a day's work. http://tinyurl.com/n3swgjr For those of us who made a vocation of simple, low cost, robust designs in electrical systems . . . it's all in a days work. The simple ideas upon which that work is based is timeless, unchanging and indestructible. Only the chefs and the recipes for success change . . . hopefully for the better. Bob . . . ________________________________________________________________________________
Date: Mar 18, 2014
From: D L Josephson <dlj04(at)josephson.com>
Subject: Re: Lithium batteries
Perhaps those who have had LiFePO4 or other lithium batteries fail after deep discharge would be willing to disassemble the various brands and report what they find inside. This would possibly be more useful than asking the sales people from the battery packagers for data they don't wish to reveal. I have been working with several groups who are researching electric aircraft. The energy density and other characteristics of the available cells are well known; at best a packaged battery is the sum of the cells inside. If there is a BMS there are three simple questions to ask: is there a low voltage disconnect during discharge, is there a high voltage disconnect during charge, and how many cells are monitored together in determining these voltages? -- David Josephson ________________________________________________________________________________
Date: Mar 18, 2014
From: "Robert L. Nuckolls, III" <nuckolls.bob(at)aeroelectric.com>
Subject: RE: EarthX battery data
At 11:23 AM 3/18/2014, you wrote: >Dear Bob, > >I do thank you for your input and responses and >as this is a virgin market for us, based on the >experimental aircraft market seeking us out, we >are re-evaluating our labeling, our manual, and >questions that need to be answered and really do appreciate your feedback. > >Our batteries were designed and marketed towards >the power sport market and our website is geared >towards customers, not engineers or OEM >manufacturers, but we do have this data and will >incorporate the needed information so our >customers can make an informed decision. Okay, I can help you with that effort. I have written dozens of product performance specifications for products that fly on both commercial and military airplanes. > > >As far as the 80% lighter=85=85.all lead acid >manufacturers have different weights on the same >lead acid replacement battery and then will also >list different weights on the exact same lead >acid battery out there. For example, the PC680 >is listed as 15.4 pounds on the Odyssey website, >it is listed as 14.8 pounds on an Amazon listing >and it is listed as 16 pounds on Battery Mart >website. Another lead acid battery that is >recommended to replace a PC680 is the Big Crank >ETX30L which weighs 23 pounds. And of course, >you can find different weighs listed for this >battery as well but for simplicity, I will use the 23 pounds. Yes, but you're not considering ALL of the features for equivalency. I note that the lithium chemistry entrepreneurs are fond of the term "Lead-Acid Equivalency". It appears that this term was invented out of whole cloth to assert, "This lithium battery cranks an engine as well as that lead-acid battery." This is a useful term for folks interested only with getting an engine started. I suggest to you that airplanes are another matter entirely. Airplanes have complex instrumentation, navigation and communications requirements that need to be powered when and if the alternator fails. A smaller number of airplanes also feature electronically managed engines with significant energy demands on the order of 100 watts. If one expects to sustain flight under battery-only conditions, a common design goal is 1 hour of endurance, 2 hours would be MUCH better. Hence, it is critical that your marketing literature include data not unlike that which I cited for the Panasonic lead-acid batteries. If you want to drop into a battery box once occupied by a PC680, consider the following data offered by Hawker/Enersys concerning the performance of that battery . . . http://tinyurl.com/ng9jhua Emacs! The PC680, when new, will carry a 130 watt demand for one hour . . . only 45 minutes at end of battery life. Hawker/Enersys doesn't have a 22 pound battery (10Kg) in that style, but if they did, it would offer about 10/7 or about 1.4 times the capability of the PC680. What is the one-hour delivery capability of your 3.5 pound battery? > >The battery the we recommend to replace this >battery is 3.5 pounds, for a 77-78% weight >savings, depending on what weight you want to >use compared to the Odyssey battery and 85% >lighter than the Big Crank. We do list the >actual weight in our battery spec=92s page on the >website and it someone really needs to know the >exact percent weight savings on the battery they >are using for their application, they will have >to do the math and get a real weight of the lead >acid battery they are using compared to >ours. Therefore, we do not feel there is a >disconnect on our claim that we can be 80% lighter. BUT . . . while your 3.5 pound offering will start the engine as smartly as the 22 pound example, the 3.5 pound battery CANNOT support critical electronics for the same time as the 22 pound battery. For this reason, I counsel caution with respect to that PbEq number. There's no engineering validation for the term in an aviation market. Unfortunately, most DIY airplane builders are NOT well versed in the legacy terminology . . . much less new terms created to promote the low internal impedance of lithium cells. Based on the above assertions, I suggest there is a huge disconnect when you're talking to builders and maintainers of personal aircraft. Assertion of this equivalency at Cessna, Beech, Mooney or Piper would probably get you more than raised eyebrows. > >And again, I have forwarded this to our >engineering department and when they are able, >they will get back to you or provide me with the answers to your questions. > > >Sincerely, > > >Kathy Nicoson >Global Sales Director >(970) 301-6064 >Fax: (970) 674-9544 >www. Earthxmotorsports.com >sales(at)earthxmotorsports.com Very well. Looking forward to it. Please convey my willingness to be of assistance in crafting competent and lucid literature for marketing to aviation. Kindest regards, Bob . . . ________________________________________________________________________________
Date: Mar 19, 2014
From: "Robert L. Nuckolls, III" <nuckolls.bob(at)aeroelectric.com>
Subject: RE: EarthX battery data
A heads-up to the List on my conversations with EarthX. Of ALL the lithium wannabes I've approached for expanded data on the operation and performance of their batteries, EarthX has been the ONLY one to respond in any positive and professional manner. The e-mails I've forwarded to the List are conversations with an individual in sales. I've received a data dump from an applications person who promises to draw design and performance talents into the conversation. I'm not going to forward any more emails to the List out of respect for their willingness to share what may be considered proprietary data. But I want to make the List aware of the fact that EarthX seems interested in widening their customer base on a foundation of well considered application of their products. I'm going to offer them assistance in crafting functional specifications that speak purely to performance in terms that would be appreciated by my colleagues in TC aviation. At the same time, I will acquire foundation for offering solid advice to the OBAM aviation community as to suitability to task for EarthX . . . or any other lithium offering. Watch this space. Bob . . . Bob . . . ________________________________________________________________________________
Subject: Re: New Over-Voltage Protection Architecture
From: "user9253" <fransew(at)gmail.com>
Date: Mar 19, 2014
I like Eric's idea of opening the alternator output circuit in case of too high voltage, rather than shorting out the field. One thing to consider is, what happens if this device fails (even if unlikely)? Of course the same question could be asked of the crowbar type of O.V. protection. From a pilot's perspective, it is desired to have electrical power no matter what goes wrong, provided that the cost and weight penalties are not too great. Some builders install a second aircraft battery, which is a huge weight penalty. How about combining Bob's suggestion of having a backup voltage regulator with Eric's circuit (or modified AEC/9003)? In the event of over-voltage, automatically turn off voltage regulator number one, and at the same time, turn on voltage regulator number two, along with a warning light. Not only would the electrical system be protected against over-voltage, but the alternator would keep on supplying electrical power to the aircraft. Joe -------- Joe Gores Read this topic online here: http://forums.matronics.com/viewtopic.php?p=420616#420616 ________________________________________________________________________________
Subject: Re: RE: EarthX battery data
From: william mills <wtrooper(at)gmail.com>
Date: Mar 19, 2014
Bob - I recognized EarthX's honesty and sincerity in response to your inquiries. Their cooperative tone is indicative of a highly principled business ethic, sadly lacking with some (most?) vendors. EarthX should be recognized for this. Thank you - Bill SF bay area On Mar 19, 2014, at 6:23 AM, Robert L. Nuckolls, III wrote: > > > > A heads-up to the List on my conversations with EarthX. > > Of ALL the lithium wannabes I've approached for expanded > data on the operation and performance of their batteries, > EarthX has been the ONLY one to respond in any positive > and professional manner. > > The e-mails I've forwarded to the List are conversations > with an individual in sales. I've received a data dump > from an applications person who promises to draw design > and performance talents into the conversation. > > I'm not going to forward any more emails to the List > out of respect for their willingness to share what > may be considered proprietary data. > > But I want to make the List aware of the fact that > EarthX seems interested in widening their customer > base on a foundation of well considered application > of their products. > > I'm going to offer them assistance in crafting functional > specifications that speak purely to performance > in terms that would be appreciated by my colleagues > in TC aviation. At the same time, I will acquire > foundation for offering solid advice to the > OBAM aviation community as to suitability to > task for EarthX . . . or any other lithium > offering. > > Watch this space. > > Bob . . . > > > Bob . . . > > > > > ________________________________________________________________________________
Date: Mar 19, 2014
From: Jeff Luckey <jluckey(at)pacbell.net>
Subject: Re: New Over-Voltage Protection Architecture
You might be approaching a situation where "the cure is worse than the dise ase".- You have to be careful when stacking widgets on top of gizmos in p ursuit of greater reliability.- That approach quite often leads to greate r complexity & lower reliability.=0A=0A=0A-Jeff=0A=0A=0A___________________ _____________=0A From: user9253 <fransew(at)gmail.com>=0ATo: aeroelectric-list @matronics.com =0ASent: Wednesday, March 19, 2014 8:01 AM=0ASubject: AeroEl ectric-List: Re: New Over-Voltage Protection Architecture=0A =0A=0A--> Aero Electric-List message posted by: "user9253" =0A=0AI like Eric's idea of opening the alternator output circuit in case of too high v oltage, rather than shorting out the field.- One thing to consider is, wh at happens if this device fails (even if unlikely)?- Of course the same q uestion could be asked of the crowbar type of O.V. protection.=0A- From a pilot's perspective, it is desired to have electrical power no matter what goes wrong, provided that the cost and weight penalties are not too great. - Some builders install a second aircraft battery, which is a huge weight penalty.=0A- How about combining Bob's suggestion of having a backup vol tage regulator with Eric's circuit (or modified AEC/9003)?- In the event of over-voltage, automatically turn off voltage regulator number one, and a t the same time, turn on voltage regulator number two, along with a warning light.- Not only would the electrical system be protected against over-v oltage, but the alternator would keep on supplying electrical power to the aircraft.=0AJoe=0A=0A--------=0AJoe Gores=0A=0A=0A=0A=0ARead this topic onl ine here:=0A=0Ahttp://forums.matronics.com/viewtopic.php?p=420616#420616 = ________________________________________________________________________________
From: "Roger & Jean" <mrspudandcompany(at)verizon.net>
Subject: Re: New Over-Voltage Protection Architecture
Date: Mar 19, 2014
> How about combining Bob's suggestion of having a backup voltage regulator > with Eric's circuit (or modified AEC/9003)? In the event of over-voltage, > automatically turn off voltage regulator number one, and at the same time, > turn on voltage regulator number two, along with a warning light. Not > only would the electrical system be protected against over-voltage, but > the alternator would keep on supplying electrical power to the aircraft. That's all well and good if the failure is the regulator! My experience, however has been that the alternator is several times more prone to failure than the regulator, and this senerio will not fix a defective alternator! Roger ________________________________________________________________________________
From: Stuart Hutchison <stuart(at)stuarthutchison.com.au>
Subject: EarthX lithium
Date: Mar 20, 2014
G'day Bob N. Your comments in the PDF file about battery Ah rating under different operating conditions reminded me of an interesting annecdote. About 10 years ago, Energiser began marketing a series of cheap (approx $20) AA battery powered chargers for cell phones. They are a simple plastic case (inc circuit) for two primary AA cells, plus a short adaptor cord option for the various cell phone models. Turns out they come fitted with lithium cells for good reason. >From spec data sheets at the time, the rating for a standard Energiser alkaline AA cell was typically around 2700 mAh. The lithium cells were about 3200 mAh, yet manufacturers quoted 5 to 7 times the endurance for lithium cells in high power devices. This seemed counter-intuitive given a similar mAh rating. In practice, the lithium cells can deliver higher current without the internal resistance soaring through the roof, so most of their energy is delivered to the load rather than wasted as heat. It was easily proven by replacing the AA lithium cells with alkaline cells. The alkaline cells became too hot to touch and delivered about half a cell phone charge, while the lithium cells delivered about 2.5 full charges. Conversely, using lithium cells in a low power device such as a wall clock would offer a very poor return on investment, given the lithium AA cells cost about 4 times the price. Cheers, Stu -----Original Message----- From: owner-aeroelectric-list-server(at)matronics.com [mailto:owner-aeroelectric-list-server(at)matronics.com] On Behalf Of Robert L. Nuckolls, III Sent: Tuesday, March 18, 2014 3:41 AM Subject: AeroElectric-List: EarthX lithium I just became aware of EarthX batteries based on comments here on the list. I visited their website and studied a variety of assertions and comparisons designed to promote sales. At first blush, these folks are hanging their marketing hats on the same philosophy as other suppliers of lithium technologies. See attached document . . . The DEARTH of good engineering data for these products makes it difficult if not impossible to craft a well considered integration of the current COTS (commericial off the shelf) offerings onto airplanes. I had to delay submission of the last of four Kitplanes articles for a month . . . not because theres a paucity of choices . . . but because the folks selling them can't define performance in rational numbers. Still digging . . . Bob . . . ________________________________________________________________________________
Date: Mar 20, 2014
From: "Robert L. Nuckolls, III" <nuckolls.bob(at)aeroelectric.com>
Subject: EarthX lithium
> . . . using lithium cells in a low power device such as a wall > clock would offer a very >poor return on investment, given the lithium AA cells cost about 4 times the >price. Yeah, your talking about these critters . . . Emacs! They're in the Size 91 package (AA) but offer a lithium-Iron couple that's in the same ballpark as the carbon-zinc and zinc-manganese flashlight cells (1.5 volts). If you compare E91 alkaline cell data Vhttp://tinyurl.com/nt2x8hd with the L91 lithium-iron data http://tinyurl.com/otkeubs We see that both products are in the same ballpark for contained energy but the L91 cell internal resistance is about 1/2 that of the E91. So not only do you get slightly more total energy, your access to that energy at high discharge rates suffers about 1/2 the losses in heat. This 'lithium' product is a different animal from the devices we're pondering as replacement for engine-cranking/standby service. The boss-hogg lithium cells are 3.3V couples and generally much lower resistance still . . . It's unfortunate that characteristic performance of two cells are not presented in the same pews . . . suffice it to say that the approx 1/2 internal resistance will toss off less energy under high demand loads. On the Nickle-Metal Hydride side of the house one may acquire 1.2V chemistry with very attractive internal impedances . . . http://tinyurl.com/nj5xx85 At 1.2V vs. 1.5 volts the contained energy in watt-seconds suffers slightly but they'll readily deliver their contents at high rates. Down-side is that the self-discharge characteristics of Ni-Mh compared to the Lithium-Iron or Zinc-Manganese cells is pretty sad. Up-side is that you can readily re-use these cells many times over and their acquisition costs are nominal. http://tinyurl.com/nklo97k Your right, lithium-iron AA cells poor value compared to other choices . . . unless your design goals absolutely demand high rate discharge support AND long shelf life. Its all in the design goals . . . Bob . . . ________________________________________________________________________________
Date: Mar 20, 2014
From: "Robert L. Nuckolls, III" <nuckolls.bob(at)aeroelectric.com>
Subject: Re: New Over-Voltage Protection Architecture
At 10:58 AM 3/19/2014, you wrote: >You might be approaching a situation where "the cure is worse than >the disease". You have to be careful when stacking widgets on top >of gizmos in pursuit of greater reliability. That approach quite >often leads to greater complexity & lower reliability. Good put. What is the line of thought that drives the notion that simple removal of field voltage from a runaway system is inconsistent with our assessment of risk? I'll have to ask around . . . I've been disconnected from the field service loop in regulators for some years . . . but I don't recall any controllers ever coming back to B&C where a regulator fallen from grace was blessed with salvation by the ov protection system. I'm not suggesting that such failure rates are zero but they ARE quite small. Further the prudent response to a competent FMEA dictates that we include such protection in spite of those low failure rates. It's been a long time since I've observed a car approaching me at night with lights that were too blue/bright demonstrating the fact that a poor battery was doing its best to stand off a runaway alternator. I'm also reading anecdotal bits about stock, automotive alternators being incorporated onto aircraft sans ov protection. It MIGHT be that contemporary regulators have achieved 10 to the minus 9 or better failure rates that suggest the ov protection is no longer necessary/useful. If anyone runs across such an incident, I'd like to hear about it. Bob . . . ________________________________________________________________________________
Subject: Re: EarthX Batteries
From: speedy11(at)aol.com
Date: Mar 20, 2014
I concur with Bob's comments. My dealings with the folks at EarthX has be en professional and forthcoming. It appears they are pushing hard to become the market leaders and their bat tery monitoring system may be just the thing to get them there. I have decided to distribute their batteries at my FBO operation and I will be ordering two batteries soon to replace the PC680s in my RV-8A (Reno Rac e 84). I will follow up with non-technical, but nonetheless interesting da ta points on my experience using the lithium batteries. Stan Sutterfield Spruce Creek FBO Daytona Beach A heads-up to the List on my conversations with EarthX. Of ALL the lithium wannabes I've approached for expanded data on the operation and performance of their batteries, EarthX has been the ONLY one to respond in any positive and professional manner. The e-mails I've forwarded to the List are conversations with an individual in sales. I've received a data dump from an applications person who promises to draw design and performance talents into the conversation. I'm not going to forward any more emails to the List out of respect for their willingness to share what may be considered proprietary data. But I want to make the List aware of the fact that EarthX seems interested in widening their customer base on a foundation of well considered application of their products. I'm going to offer them assistance in crafting functional specifications that speak purely to performance in terms that would be appreciated by my colleagues in TC aviation. At the same time, I will acquire foundation for offering solid advice to the OBAM aviation community as to suitability to task for EarthX . . . or any other lithium offering. Watch this space. Bob . . . ________________________________________________________________________________
Subject: Re: New Over-Voltage Protection Architecture
From: "user9253" <fransew(at)gmail.com>
Date: Mar 21, 2014
A voltage regulator is not likely to fail unless overheated. An over-voltage protection device is less likely to fail because it is usually not subject to excessive heat. What is more likely to fail is an electrical connection or switch somewhere between the main bus and the voltage regulator. Good workmanship, strain relieved terminals, and a dab of grease to prevent corrosion, will all help. Even if properly installed, switches and connections can corrode over time. These problems are unlikely to be detected during annual inspections. Many modern avionics can operate on voltages between 10 and 30, and can withstand minor voltage excursions. Over-voltage protection is like buying insurance. Is it worth the premium? Joe -------- Joe Gores Read this topic online here: http://forums.matronics.com/viewtopic.php?p=420718#420718 ________________________________________________________________________________
Subject: Re: New Over-Voltage Protection Architecture
From: "Eric M. Jones" <emjones(at)charter.net>
Date: Mar 21, 2014
I learned to fly in a club with five Cessnas, and the only alternator failure I was personally involved with was an alternator bracket that broke from fatigue. (There was one "failure to charge" whose cause I never learned.) If you look at all the ways an aircraft can fail, I think alternator failures are low on the list. I have not heard of a NEW (not rebuilt) Nippondenso alternator that failed at all, save overheating, drowning, old age, etc. This might be true of other manufacturers as well. Any reports to the contrary? Rather than trying to protect the electrical system from a failed alternator, perhaps some effort in determining when an alternator should be replaced would be a good approach. 1000 hours, engine TBO? Most electrical parts benefit from not being driven too hard, and being kept cool. If this is done, count me as unconvinced that OVP is necessary with an internally-regulated modern alternator. -------- Eric M. Jones www.PerihelionDesign.com 113 Brentwood Drive Southbridge, MA 01550 (508) 764-2072 emjones(at)charter.net Read this topic online here: http://forums.matronics.com/viewtopic.php?p=420721#420721 ________________________________________________________________________________
Date: Mar 21, 2014
From: Charlie England <ceengland7(at)gmail.com>
Subject: Re: New Over-Voltage Protection Architecture
On 3/20/2014 2:11 PM, Robert L. Nuckolls, III wrote: > > > At 10:58 AM 3/19/2014, you wrote: >> You might be approaching a situation where "the cure is worse than >> the disease". You have to be careful when stacking widgets on top of >> gizmos in pursuit of greater reliability. That approach quite often >> leads to greater complexity & lower reliability. > > Good put. > > What is the line of thought that drives the > notion that simple removal of field voltage from > a runaway system is inconsistent with our > assessment of risk? > > I'll have to ask around . . . I've been disconnected > from the field service loop in regulators for some > years . . . but I don't recall any controllers ever coming > back to B&C where a regulator fallen from grace > was blessed with salvation by the ov protection > system. > > I'm not suggesting that such failure rates are zero but > they ARE quite small. Further the prudent response > to a competent FMEA dictates that we include such > protection in spite of those low failure rates. > > It's been a long time since I've observed a car > approaching me at night with lights that were too > blue/bright demonstrating the fact that a poor > battery was doing its best to stand off a > runaway alternator. > > I'm also reading anecdotal bits about stock, > automotive alternators being incorporated onto > aircraft sans ov protection. It MIGHT be that > contemporary regulators have achieved 10 to the > minus 9 or better failure rates that suggest > the ov protection is no longer necessary/useful. > > If anyone runs across such an incident, I'd like > to hear about it. > > > Bob . . . Are you asking for incidents of *non* failures with internally regulated alternators & no protection? I've been running a Denso on an RV-4 Lyc O320 for about a decade. Not a lot of hours (probably around 400), but no issues with OV. This is a very simple VFR a/c that only recently acquired a transponder, so my financial and safety risks have been pretty low. I'll probably incorporate some type of protection on the -7 I'm building, just because I'll have more money tied up in the panel. Charlie ________________________________________________________________________________
Date: Mar 21, 2014
From: David Josephson <dlj04(at)josephson.com>
Subject: Re: New Over-Voltage Protection Architecture
> What is the line of thought that drives the > notion that simple removal of field voltage from > a runaway system is inconsistent with our > assessment of risk? Only that removal of field voltage is impossible with a permanent magnet alternator and inconvenient with an internally regulated one. ________________________________________________________________________________
Date: Mar 21, 2014
From: D L Josephson <dlj04(at)josephson.com>
Subject: Re: Earth-X lithium
Just as lead-acid electrochemistry is fairly well understood, lithium cell vendors and types are well known. Every maker of small "aviation" lithium batteries uses the same LiFePO4 18650 cell type as far as I know, which is also the same as Tesla uses in their car battery packs, and the same chemistry used in most smartphones today. There are various quality levels available, with the leader seeming to be the Panasonic NCR18650A, which is honestly rated at 2.9 amp-hours at 3.2 volts (more than 2.9 can be claimed by assuming a lower cutout voltage, which requires more precise low voltage disconnect to avoid damage.) Packs are made of strings of 4 cells in series, which is why you see 8-, 12-, 16 etc. cell "batteries." At last year's CAFE Electric Aircraft Symposium, Eric Darcy from NASA, who is responsible for designing the lithium batteries for the Space Station, gave some pointers on testing these ubiquitous cells for quality (you measure open circuit voltage over a few weeks -- cells that droop faster than the norm are suspect) and the measures that need to be taken to assure safe charging and low voltage disconnect. We learned that quality was primarily related to clean shearing and even winding of the internal foil materials, with failures mostly due to minute stray particles of the sheared material causing leakage, or misalignment of the wound structure allowing the edges to touch. Ideally each cell is charged separately. Next best is to charge each pack of 4 cells separately. The trick comes in designing a battery management system inside the box that manages charge current for multiple packs and at the very least alerts you if something is wrong. A battery company that will actually explain how they are doing these things, and takes the time to explain why a watt-hour of lithium cells might be more useful than the same watt-hour of lead acid, would be worth paying attention to. ________________________________________________________________________________
Date: Mar 21, 2014
From: Jeff Luckey <jluckey(at)pacbell.net>
Subject: Re: New Over-Voltage Protection Architecture
=0A=0A=0A=0A________________________________=0A From: David Josephson <dlj0 4(at)josephson.com>=0ATo: aeroelectric-list(at)matronics.com =0ASent: Friday, Mar ch 21, 2014 9:21 AM=0ASubject: AeroElectric-List: Re: New Over-Voltage Prot Josephson =0A=0A=0A=0A>- - - What is the line of thought that drives the=0A>- - - notion that simple removal of fi eld voltage from=0A>- - - a runaway system is inconsistent with our =0A>- - - assessment of risk?=0A=0AOnly that removal of field voltag e is impossible with a permanent magnet =0Aalternator and inconvenient with an internally regulated one.=0A=0ADavid,=0A=0AI've never had occasion to f iddle with a PM alternator. =0A=0AI've always thought of a run-away conditi on as=0Aa regulator failure where it is no longer "regulating" =0Aand apply ing full field current.=0A=0ACan a PM alternator "run-away"?=0A=0A-Jeff=0A =========================0A =================== ________________________________________________________________________________
Date: Mar 21, 2014
From: "Robert L. Nuckolls, III" <nuckolls.bob(at)aeroelectric.com>
Subject: Re: New Over-Voltage Protection Architecture
At 11:21 AM 3/21/2014, you wrote: > > >> What is the line of thought that drives the >> notion that simple removal of field voltage from >> a runaway system is inconsistent with our >> assessment of risk? > >Only that removal of field voltage is impossible with a permanent >magnet alternator and inconvenient with an internally regulated one. That's why all the z-figures that incorporate pm alternators use a disconnect relay paired with an ov sensor to effect a disconnect . . . This unique characteristic of the PM alternator is what suggested that they be called a 'dynamo' to separate them out from the herd of engine driven power sources. B-lead disconnect has also been suggested in Figure z-24 of the 'Connection's collection of architectures. Bob . . . ________________________________________________________________________________
Date: Mar 21, 2014
From: "Robert L. Nuckolls, III" <nuckolls.bob(at)aeroelectric.com>
Subject: Re: New Over-Voltage Protection Architecture
At 08:23 AM 3/21/2014, you wrote: A voltage regulator is not likely to fail unless overheated. An over-voltage protection device is less likely to fail because it is usually not subject to excessive heat. What is more likely to fail is an electrical connection or switch somewhere between the main bus and the voltage regulator. Good workmanship, strain relieved terminals, and a dab of grease to prevent corrosion, will all help. Even if properly installed, switches and connections can corrode over time. These problems are unlikely to be detected during annual inspections. There are two approaches to system reliability analysis. One approach assumes NO backup . . . either because such features are too costly/bulky/heavy/etc or simply not possible. If any of these features are critical to comfortable termination of flight, they get designed and tested for very high probability of meeting a 'design service life' that is generally some multiple of 'practical service life'. For example, flown long enough, the wings on EVERY aluminum airplane are going to break off. This is because aluminum, unlike steel, has a stress-to-cycles plot that never goes 'flat'. If you stress and relieve a steel part to maxiumum ever expected loads 10,000,000 times without breaking it, the part is considered 'golden' and will last forever. Aluminum has no such feature . . . it eventually fails at any stress loading . . . the cycles may be very high but there is no flat spot on the s/n curve to failure. Hence you see a totally different approach for the design setting service limits for qualification of structural parts on airplanes. Approaches that have continuously evolved over the years particularly in response to incidents like this . . . http://www.aloha.net/~icarus/243a.jpg Emacs! An alternative technique for making design decisions can be adopted for items that are not immediately catastrophic. The generally categorized by criticality level not unlike software under DO-178 . . . (a) Catastrophic - Failure may cause multiple fatalities, usually with loss of the airplane. (b) Hazardous - Failure has a large negative impact on safety or performance, or reduces the ability of the crew to operate the aircraft due to physical distress or a higher workload, or causes serious or fatal injuries among the passengers. (c) Major - Failure significantly reduces the safety margin or significantly increases crew workload. May result in passenger discomfort (or even minor injuries). (d) Minor - Failure slightly reduces the safety margin or slightly increases crew workload. Examples might include causing passenger inconvenience or a routine flight plan change. (e) No Effect - Failure has no impact on safety, aircraft operation, or crew workload. For OBAM aircraft, we're free to tighten up our spectrum of criticality level. Depending on the design of our plan-B, we can generally drop to 3 categories . . . (a) Engine stops and we're going to descend . . . NOW (b) Some appliance goes dark and finding our way to comfortable landing . . . preferably at airport of intended destination . . . is at risk. (c) Some appliance goes dark but while convenient, is not critical to continued flight, navigation, approach to landing and parking the airplane. These three categories focus on system components like fuel pumps, ECM, nav receivers, orientation aids, panel lighting, electronic ignition, comm, and perhaps xponder. The builder/pilot has to decide what order of important fits their particular project and the environment in which they intend to fly. Then an architecture and hardware compliment needs to be crafted such that no single failure takes down more than one accessory and all really useful or critical accessories have a plan-B. The elegant solution minimizes weight, cost, volume, parts-count and pilot work loads. It takes an appreciation of the thoughts outlined above to understand the rationale for careful consideration before ov protection is no longer in your plans. Recall that MTBF numbers say NOTHING about the behavior of any single part. If one strives for predictable behavior, then you venture into the world of 'established reliability' accessories where a great deal of money has been spend to design, test, manufacture and perhaps even screen finished goods to weed out infant mortality . . . The easiest way to deal with SYSTEM reliability with internally regulated alternators is to ASSUME it will fail and install a plan-b . . . as long as that addition is not a significant cost/weight adder. OV protection adds little overhead. Second alternators add some weight and up-front costs but minimizes cost of ownership. Second batteries are probably cheaper up front but add more weight and perpetual cost of ownership burdens for preventative maintenance and r/r costs common to an expendable commodity. Don't forget one super-significant feature of the engine driven power source: It's an inexhaustible source of energy . . . at potentially high voltages (100+ volts). The alternator is unique in this regard. The battery can deliver a lot of current but its energy reservoir is limited. The alternator is current limited but for all practical purposes, it's output voltage and total energy is not limited. These features are foundations on which you build your personal failure mode effects analysis. An analysis that could not care less about anecdotal reliability narratives and builds a foundation of system reliability on logic assembly of simple-ideas irrespective of any perceptions of probability. Many modern avionics can operate on voltages between 10 and 30, and can withstand minor voltage excursions. Over-voltage protection is like buying insurance. Is it worth the premium? OV protection is not about minor excursions. It's about unleashing the flame thrower first against your battery (by the way, if it's a tiny lithium super-cranker, it will toss in the towel much faster than your 18 or 24Ah SVLA brick) and then upon system accessories. Just a few steps from where I sit right now is a lab where these features and effects have been studied for years. It's not a concept to be dismissed lightly. A runaway alternator is the acme of electrical disasters on about any DC power system. If you're willing to buy into the anecdotal 'never heard of it happening' . . . then ask the supplier of your alternator/regulator combination if he'll guarantee replacement of electro-whizzies on your panel if his gizmo fails? This is why we elected to bring the B&c alternator controllers to market as a TRIO of accessories that offered a very favorable FMEA. The thought was that if the customer wanted to go a different route, they're certainly free to do so . . . but not with our product. Proceed with both caution and confidence borne out of lessons learned . . . Bob . . . ________________________________________________________________________________
Subject: Std density D sub pins.
From: Tim Andres <tim2542(at)sbcglobal.net>
Date: Mar 22, 2014
Bob, what is the current rating on the Std density D sub pins? This is re: your avionics ground bus and the 18 gauge com wires going into it. I have the oversize pins that accept 18 awg, but I'm thinking the ~8 amps intermittent load needs to be divided over two pins. Thanks Tim ________________________________________________________________________________
Date: Mar 22, 2014
From: "Robert L. Nuckolls, III" <nuckolls.bob(at)aeroelectric.com>
Subject: Re: Std density D sub pins.
At 09:10 AM 3/22/2014, you wrote: > >Bob, what is the current rating on the Std density D sub pins? This >is re: your avionics ground bus and the 18 gauge com wires going >into it. I have the oversize pins that accept 18 awg, but I'm >thinking the ~8 amps intermittent load needs to be divided over two pins. >Thanks >Tim Splitting it onto 2 pins with 6", 22AWG ballast pigtails is never wrong . . . but the super-pin might be okay. What accessory are you talking about? Bob . . . ________________________________________________________________________________
Date: Mar 22, 2014
From: "Robert L. Nuckolls, III" <nuckolls.bob(at)aeroelectric.com>
Subject: Lamar Super-Switch solid state contactor
At 11:15 AM 3/22/2014, you wrote: >Bob, >I have been told you have a solid state master relay that will not >get hot like my one amp current relay. Could you please tell me the >part number for the relay. Your book has been a real inspiration. Thank you. Lamar used to manufacture one, Aircraft Spruce used to sell them. I don't find them on either website. I think the product has been pulled. At one time the factory had me on a 'list ' to receive an engineering sample for evaluation . . . but I've never seen one in the 'flesh' . . . What's your application? What are your design goals? Is it just the heat that bothers you? Yes, they are hot to touch but that does not automatically translate into an evil property. The light bulb gets hot too but delivers the advertised performance. Bob . . . ________________________________________________________________________________
Date: Mar 22, 2014
From: Tim Andres <tim2542(at)sbcglobal.net>
Subject: Re: Std density D sub pins.
This is for a- GTN 650 GPS/NAV/COMM. All the grounds out of that box.=0AT im=0A=0A=0A=0A=0A________________________________=0A From: "Robert L. Nucko lls, III" =0ATo: aeroelectric-list@matronics .com =0ASent: Saturday, March 22, 2014 9:38 AM=0ASubject: Re: AeroElectric- List: Std density D sub pins.=0A =0A=0A--> AeroElectric-List message posted by: "Robert L. Nuckolls, III" =0A=0AAt 09:1 Andres =0A> =0A> Bob, what is the current rating on the Std density D sub pins? This is re: your avionics ground bus and the 1 8 gauge com wires going into it. I have the oversize pins that accept 18 aw g, but I'm thinking the ~8 amps intermittent load needs to be divided over two pins.=0A> Thanks=0A> Tim=0A=0A=0A- Splitting it onto 2 pins with 6", 22AWG=0A- ballast pigtails is never wrong . . .=0A- but the super-pin m ight be okay.=0A=0A- What accessory are you talking about?=0A=0A=0A- Bo ====================== ________________________________________________________________________________
Date: Mar 22, 2014
Subject: Re: Std density D sub pins.
From: Bill Allen <billallensworld(at)gmail.com>
Hi Guys, Good question Tim <> I'm looking at something similar right now Bob; I'm not sure I understand correctly what << with 6", 22AWG ballast pigtails >> means? Can you clarify? regards, Bill Allen On 22 March 2014 17:31, Tim Andres wrote: > This is for a GTN 650 GPS/NAV/COMM. All the grounds out of that box. > Tim > > > ------------------------------ > *From:* "Robert L. Nuckolls, III" > *To:* aeroelectric-list(at)matronics.com > *Sent:* Saturday, March 22, 2014 9:38 AM > *Subject:* Re: AeroElectric-List: Std density D sub pins. > > nuckolls.bob(at)aeroelectric.com> > > At 09:10 AM 3/22/2014, you wrote: > tim2542(at)sbcglobal.net> > > > > Bob, what is the current rating on the Std density D sub pins? This is > re: your avionics ground bus and the 18 gauge com wires going into it. I > have the oversize pins that accept 18 awg, but I'm thinking the ~8 amps > intermittent load needs to be divided over two pins. > > Thanks > > Tim > > > Splitting it onto 2 pins with 6", 22AWG > ballast pigtails is never wrong . . . > but the super-pin might be okay. > > What accessory are you talking about? > > > Bob . . . -Mat============== > > > * > > > * > > ________________________________________________________________________________
Subject: Re: Std density D sub pins.
From: "donjohnston" <don@velocity-xl.com>
Date: Mar 22, 2014
And what is a "super pin"? Read this topic online here: http://forums.matronics.com/viewtopic.php?p=420851#420851 ________________________________________________________________________________
Subject: Re: Std density D sub pins.
From: "donjohnston" <don@velocity-xl.com>
Date: Mar 22, 2014
And what is a "super pin"? Read this topic online here: http://forums.matronics.com/viewtopic.php?p=420852#420852 ________________________________________________________________________________
Date: Mar 22, 2014
From: "Robert L. Nuckolls, III" <nuckolls.bob(at)aeroelectric.com>
Subject: Re: Std density D sub pins.
At 12:31 PM 3/22/2014, you wrote: >This is for a GTN 650 GPS/NAV/COMM. All the grounds out of that box. >Tim Okay, I'm seeing grounds on P1001, 1004 and 1003 that start out as multiple pins and splice down to single conductors of 18 or 20AWG depending on system voltage for a total of 7 wires. Without conversation with the folks who set it up, the rationale is not known to us. I've seen this 'pin doubling' on a variety of radios over the past 20 years. One thing you could do is run individual 20AWG wires from ALL 7 pins on the radio to independent pins in the ground bus. This would cover a host of rationals from quest for low impedance groung path (shaky), redundancy (a little less shaky) or load sharing in the pins (which is my best guess). I sold this philosophy onto the GQM163 super sonic sea skimmer about 10 years ago. http://tinyurl.com/7h9h76r I think if it were my airplane, I'd go with 7 separate wires in bundle to the ground bus. Bob . . . ________________________________________________________________________________
Date: Mar 22, 2014
From: "Robert L. Nuckolls, III" <nuckolls.bob(at)aeroelectric.com>
Subject: Re: Std density D sub pins.
At 02:17 PM 3/22/2014, you wrote: > >And what is a "super pin"? Somebody, I think Garmin, spec'd a special pin for d-sub connectors that would accept larger than 20AWG wire. This is extra long such that the crimp happens outside the connector housing. I'm mystified by this product . . . in no way does it improve on the current rating of the individual pin, only the ability to attach wires rated for a lot more current than the pin . . . wouldn't use one myself. Bob . . . ________________________________________________________________________________
Date: Mar 23, 2014
From: "Robert L. Nuckolls, III" <nuckolls.bob(at)aeroelectric.com>
Subject: Re: P-Leads for a Seawind
Oh Great Guru: Thank you sir . . . I'll endeavor to live up to your expectations . . . I am installing a dual efis single ahrs GRT system with remote eis in a Seawind. I am installing dual alternator dual battery electrical system per your Z-14 schematic with parts from B & C. Okay, you've got more SYSTEM reliability than most twin-engine, TC aircraft . . . Having experienced multiple electrical failures in 20 yrs. of a '63 Cherokee and already one alternator failure in 5yrs of my current '78 C-182RG, I'm dead set on having a few backup "steam" gages. Perhaps "living confidence" as opposed to "dead disappointment" is the better term . . . Using the 10.4" EFIS displays, I don't have much panel space to play with. There's just enough room under the EFIS's on the PIC side for 2 1/4 airspeed and manifold pressure gages, a Tiny Tach and a MicroTim altimeter, which have internal batteries. Why the batteries? You got more sources of 'juice' in this airplane than Jonnie Appleseed has apples . . . Emacs! GRT connects to the P-leads on each mag with a 39 or 27 Kohm resistor, claiming the resistor will act as a fuse if there's a problem in the EIS. The resistor isolated signal tap is a time honored, well proven technique for sampling the p-lead signals. Tiny Tach people want me to drill a hole in the plug wire cap and wrap several turns of "antenna" around a plug wire insulation. This "antenna" wire insulation is about .06" diam. The plug insulation is a close fit inside the rubber plug that seals the mag. The mag is pressurized from my supercharger. A good seal is desirable. I discussed this problem with a TinyTach tech and he allowed a direct connection is not their recommendation, but the tiny resistor might be o.k., especially if one also included a tiny fuse (my suggestion.) I am suspicious of his design. Getting into a spark-plug wire to sample engine rpm borders on . . . well . . . I'll be nice. I 'sampled' the #1 plug wire on a Baron WWAAAYyyyy back when for the purpose of driving a propeller synchronizer system. For that task, I found that one could detect the CURRENT in the plug wire from OUTSIDE the shield with a toroidal current transformer. Got a nice signal consistent with firing of #1 cylinder. But getting 'under the shield' is unnecessary . . . particularly for the purpose of measuring RPM. See if this guy will email me a copy of his schematic . . . honorable non-disclosure agreements assumed . . . and I'll see if there's a way to do more sanitary job of integrating his product onto airplanes. Alternatively, send me your tach and I'll do some non-invasive investigations as to the nature of its needs for an input signal. My question: How do I select a fuse? If I connected at the mag where the EIS goes, I would have twenty five feet of coax to get to the TinyTach. If I connect at the panel, not more that one foot. If the tach cannot be satisfied with perfectly good data available at the magneto switch, I'd look for a better product. Another question: Is Bogert the only place in the world I can get P-Leads for my TCM mags? $90 ea. for the smallest 30" leads seems awfully high. and they are charging $3 or $4/ft. for each additional foot. (14AWG MIL-C-27500 retails for $1.47 in Spruce.) Their catalog only goes to 20 ft. & I need closer to 25. Since I'd have to cut in anyway, couldn't I just buy the minimum 30" and splice in more coax with soldersleeves? Don't use factory fabricated p-leads. They're priced for the captive, TC aircraft market. You don't need 14 AWG wire either. Here's a good source for 16 AWG at 50-cents/foot http://tinyurl.com/nczzsfa Poke around on eBay and you might come up with some 20 or 18AWG which is more appropriate. Build your own P-leads. I have some shielded 22AWG trio wire left. You could used this stuff and just parallel up the strands. It's NOT critical. You may wonder why two great big EFIS displays. GRT is the only one to easily allow live video input. Seawind drivers are installing cameras in their tip floats to watch the hull approach glassy water--makes for "greasers" on pavement a possibility too. (My dirty old men copilots want to plug in a dvd player and watch dirty movies on the way to Oshkosh.) What ever floats your seaplane my friend. That's a really impressive machine. Lets not make it any more difficult than necessary to meet requirements. Bob . . . ________________________________________________________________________________
Date: Mar 24, 2014
From: "Robert L. Nuckolls, III" <nuckolls.bob(at)aeroelectric.com>
Subject: Re: P-Leads for a Seawind
I forgot to ask. Are your magnetos relatively new or are they the the legacy Bendix mags? The older mags have a knurled nut that holds the 'cigarette terminal' in the connection well on the back of the mag. If you build your own p-leads for older mags, those parts may be hard to find . . . but there are folks on the AeroElectric-List that can help. Bob . . . ________________________________________________________________________________
Subject: Re: New Over-Voltage Protection Architecture
From: "user9253" <fransew(at)gmail.com>
Date: Mar 24, 2014
There has been a recent discussion on VansAirforce about over-voltage problems. http://www.vansairforce.com/community/showthread.php?t=111102 Since over-voltage protection does not cost much or weigh much, I think that it is worthwhile having, considering the damage that could be done by too high voltage. -------- Joe Gores Read this topic online here: http://forums.matronics.com/viewtopic.php?p=420931#420931 ________________________________________________________________________________
Date: Mar 24, 2014
From: D L Josephson <dlj04(at)josephson.com>
Subject: Re: P-leads
The concept of a "fuse" to keep faults in a tachometer from disabling an ignition needs to be stamped out, it is a dangerous misconception. I know of at least one incident where this almost caused a fatal crash. It takes very little current through a P lead from a magneto or points wire in a Kettering ignition to disrupt performance of the ignition. Can you depend on this amount of current blowing a fuse? Is the resistor going to open if there is a fault? Is the engine going to keep running well if the far end of the resistor is shorted to ground or +12? Not unless it has been tested to do so. There are many tach designs that work without an ignition pickup. VDO and other companies make programmable tachs that can be set to take a signal from one phase of an alternator output, or an inductive pickup from a flywheel or magneto case. Proper design of a tach could also yield failsafe operation but that's still putting a lot of faith in something you haven't tested. The "proof by assertion" that I've seen from a few instrument vendors isn't really confidence-inspiring. -- David Josephson ________________________________________________________________________________
From: "R. curtis" <mrspudandcompany(at)verizon.net>
Subject: New Over-Voltage Protection Architecture
Date: Mar 24, 2014
> Since over-voltage protection does not cost much or weigh much, I think > that it is worthwhile having, considering the damage that could be done by > too high voltage. Over voltage protection is recommended for an externally regulated alternator. However, this has been kicked around for the internally regulated unit, and I am not sure that there is, as yet, a good fool proof, method for this alternator. Roger -- Do you have a slow PC? Try a Free scan http://www.spamfighter.com/SLOW-PCfighter?cid=sigen ________________________________________________________________________________
From: "Vern Little" <sprocket@vx-aviation.com>
Subject: Re: New Over-Voltage Protection Architecture
Date: Mar 24, 2014
I was plagued by overvoltages on my externally regulated alternator, until I found a permanent, fool-proof solution: Schematic: http://www.vx-aviation.com/rv-9a/photos/Electrical/Regulator_relay_app.jpg This circuit bypasses the problematic alternator breaker - master switch loop - wiring loop by connecting the master bus directly to the voltage regulator through a fuselink and a relay. Since I have the battery and contactor on the firewall, the total wiring length for this is about 20", with minimal voltage drops. Ensure that you spray the relay connections with Corrosion-X or bed them in silicone grease (DC4). OVP still works normally. A side effect is that almost all alternator noise disappeared. I used to hear wig-wags and strobe noise in my headset, now it's gone. Voltage is rock-solid and does not vary with load. This design should also work with an internally regulated alternator without OVP. The ones with OVP should have a circuit breaker instead of the fuseable link. Downside to that is that the CB should be located close to the alternator and cannot be reset in flight. Cheers, Vern -----Original Message----- From: user9253 Sent: Monday, March 24, 2014 9:57 AM Subject: AeroElectric-List: Re: New Over-Voltage Protection Architecture There has been a recent discussion on VansAirforce about over-voltage problems. http://www.vansairforce.com/community/showthread.php?t=111102 Since over-voltage protection does not cost much or weigh much, I think that it is worthwhile having, considering the damage that could be done by too high voltage. -------- Joe Gores Read this topic online here: http://forums.matronics.com/viewtopic.php?p=420931#420931 ________________________________________________________________________________
Subject: Re: DIY CBOVM: variations on a theme
From: "user9253" <fransew(at)gmail.com>
Date: Mar 24, 2014
I built Bob's circuit. To test it, I wired it in series with an automotive lamp to a DC-DC converter like this one: http://tinyurl.com/kmpqcb8 I slowly turned the pot to increase the voltage. The lamp illuminated at almost 17 volts. Since my Rotax rectifier/regulator puts out about 13.8 volts, I decided to lower the set point of the over-voltage protector by substituting a 5.1K resistor for the 5.6K. Now the test lamp comes on at 16.2 volts. When the voltage increases above 15.8, the circuit becomes sensitive. Just touching the circuit with a digital voltmeter probe is enough to fire the SCR. Using a different voltmeter had the same affect. No doubt that in an electrically noisy environment like an airplane, the circuit will trip at 15.8 volts. That is not a problem though. 15.8 volts is 2 volts above my aircraft system voltage. If spring weather ever comes to Michigan, I will install it in the airplane. Thanks for designing the circuit, Bob. You are my mentor. Joe -------- Joe Gores Read this topic online here: http://forums.matronics.com/viewtopic.php?p=420938#420938 ________________________________________________________________________________
Date: Mar 24, 2014
From: K <kleh(at)dialupatcost.ca>
Subject: Re: P-leads
Yes "Fuse" is not the terminology I'd use for such a resistor. Agree that the resistor definitely needs to be large enough value that it won't interfere if grounded and testing it should not be overlooked. Where I've seen this go bad was when a wire was run to the resistor mounted in the tach. With age and moisture the wire insulation started breaking down. The resistor should be physically at the ignition pick off point, NOT remotely located. I apply the same concept to picking off a fuel flow signal from an electronic fuel injector. Ken On 24/03/2014 1:37 PM, D L Josephson wrote: > > > The concept of a "fuse" to keep faults in a tachometer from disabling an > ignition needs to be stamped out, it is a dangerous misconception. I > know of at least one incident where this almost caused a fatal crash. It > takes very little current through a P lead from a magneto or points wire > in a Kettering ignition to disrupt performance of the ignition. Can you > depend on this amount of current blowing a fuse? Is the resistor going > to open if there is a fault? Is the engine going to keep running well if > the far end of the resistor is shorted to ground or +12? Not unless it > has been tested to do so. > > There are many tach designs that work without an ignition pickup. VDO > and other companies make programmable tachs that can be set to take a > signal from one phase of an alternator output, or an inductive pickup > from a flywheel or magneto case. Proper design of a tach could also > yield failsafe operation but that's still putting a lot of faith in > something you haven't tested. The "proof by assertion" that I've seen > from a few instrument vendors isn't really confidence-inspiring. > > -- > David Josephson > > ________________________________________________________________________________
Date: Mar 24, 2014
From: "Robert L. Nuckolls, III" <nuckolls.bob(at)aeroelectric.com>
Subject: Re: DIY CBOVM: variations on a theme
At 01:41 PM 3/24/2014, you wrote: I built Bob's circuit. To test it, I wired it in series with an automotive lamp to a DC-DC converter like this one: http://tinyurl.com/kmpqcb8 I slowly turned the pot to increase the voltage. The lamp illuminated at almost 17 volts. Since my Rotax rectifier/regulator puts out about 13.8 volts, I decided to lower the set point of the over-voltage protector by substituting a 5.1K resistor for the 5.6K. Now the test lamp comes on at 16.2 volts. When the voltage increases above 15.8, the circuit becomes sensitive. Just touching the circuit with a digital voltmeter probe is enough to fire the SCR. Using a different voltmeter had the same affect. No doubt that in an electrically noisy environment like an airplane, the circuit will trip at 15.8 volts. That is not a problem though. 15.8 volts is 2 volts above my aircraft system voltage. If spring weather ever comes to Michigan, I will install it in the airplane. That should not be so sensitive to probing. How is this circuit packaged? Can you send me a pucture? Bob . . . ________________________________________________________________________________
Date: Mar 24, 2014
From: "Robert L. Nuckolls, III" <nuckolls.bob(at)aeroelectric.com>
Subject: Re: New Over-Voltage Protection Architecture
At 01:06 PM 3/24/2014, you wrote: ><sprocket@vx-aviation.com> > >I was plagued by overvoltages on my externally regulated alternator, >until I found a permanent, fool-proof solution: Can you post a schematic of your original installation? When you say you were plagued with over-voltages . . . was your ov protection tripping a lot . . . or were there ov events for which there was no installed protection? Bob . . . ________________________________________________________________________________
Date: Mar 25, 2014
From: "Robert L. Nuckolls, III" <nuckolls.bob(at)aeroelectric.com>
Subject: Re: P-Leads for a Seawind
At 12:58 AM 3/25/2014, you wrote: Thanks for taking an interest. I called LyCon in Visalia, Ca. where I had the engine modified and he clued me the type # is 66LN-21 and he had the parts in stock. They're on the way. Good show . . . I'll get 50ft. of 20 AWG MIL-C-27500 & I'm good to go. Agreed . . . I could still use some advice as to how to connect to a TinyTach at the panel. GRT has me pick off tach signal from ea. mag p-lead at the engine through a 39 or 27Kohm resistor, claims the resistor will blow like a fuse if anything shorts in their EIS TinyTach wants 5 to ten turns of the core wire of their coax around one plug wire just inside the cap on the mag. Mechanically, this is not so hot, as there is a rubber sealing plug between the plug wire insulator and the socket of the mag holding supercharger pressure. How do I reconcile 200v or so of high p-lead current with 30kv or so of low current at a plug wirefor a direct connection? For an extra 10bucks. TinyTach will supply enough wire to get back to the engine. Would it be satisfactory to cut the shield of a plug wire outside of the mag, wrap their 5 to 10 turns of insulated "antenna" coax core around the plug insulator, and cover it back up with braid and solder sleeves? Sounds pretty messy, doesn't it? I sure would like to pick the signal off clean at the ignition switch. This supplier is oblivious to the nature of the stuff he's attempting to monitor. He also doesn't understand the way things are done on airplanes. Tell him that's where you want to sample engine rpm . . . on wires that are already in place. Adding new wire from a plug wire is not only sloppy, it's an invitation to radiated ignition noise. Is this your ONLY tach option? The whole world builds tachs. I'd dump this product. The $time$ your going to spend to educate him and perhaps to adapt his product to your airplane is more than the tach is worth. Bob . . . ________________________________________________________________________________
Date: Mar 25, 2014
From: "Robert L. Nuckolls, III" <nuckolls.bob(at)aeroelectric.com>
Subject: Re: New Over-Voltage Protection Architecture
At 11:57 AM 3/24/2014, you wrote: > >There has been a recent discussion on VansAirforce about >over-voltage >problems. http://www.vansairforce.com/community/showthread.php?t=111102 >Since over-voltage protection does not cost much or weigh much, I >think that it is worthwhile having, considering the damage that >could be done by too high voltage. Have you cited the right thread? These postings are all dated back in the 2007-08 time frame. Long time members on the List will recall some exchanges with Paul M who was really excited about crowbar shutdown techniques . . . he railed on for years choosing to ignore thousands of applications for the technique in both experimental and certified aircraft. Seems some of his buddies tried to build the DIY circuit and had some nuisance tripping problems. I made some modifications to the circuit based on their valuable feedback. But in any case, like any device that 'triggers on a voltage threshold' there are physical constraints for reliable performance not the least of which is compact size and perhaps metallic, grounded enclosure. The etched circuit board for the production version is only about .6" wide by 1" long and takes surface mount components. VERY small aperture for potentially offending signals. Bob . . . ________________________________________________________________________________
Subject: Re: New Over-Voltage Protection Architecture
From: "Eric M. Jones" <emjones(at)charter.net>
Date: Mar 25, 2014
Sensitron, a NY based manufacturer of electronic modules for aircraft, used to make a really beefy transient voltage suppressor. I recall it was thousands of dollars and was potted into a heatsink. It did OVP and/or load dump prevention with one basic part...parallel zeners I think. They also make solid-state relays and assorted electro-whizzies mostly for 28V military aircraft. They have a great catalog worth perusing. -------- Eric M. Jones www.PerihelionDesign.com 113 Brentwood Drive Southbridge, MA 01550 (508) 764-2072 emjones(at)charter.net Read this topic online here: http://forums.matronics.com/viewtopic.php?p=420982#420982 ________________________________________________________________________________
Date: Mar 25, 2014
From: Sam Marlow <sam.marlow(at)roadrunner.com>
Subject: Re: Marker Beacon Remote Indicators?
BobsV35B(at)aol.com wrote: > Good Afternoon Bob and Bill, > May I ask why you want the marker beacon indication? It is/was part > of the low frequency range system which was shut down over sixty years > ago. It has been retained for use with a few ILSs and some NDB > approaches, but such use is rapidly being discontinued. > I know of no fix designated by a marker beacon that cannot be found > via some other legal IFR data. > That is especially true if one has an IFR approved GPS installed. > There is nothing wrong with having a marker beacon receiver installed, > but you can eliminate an antenna and some wiring by getting rid of it. > You might even be able to save some panel space. > My vote is to forget about the marker Beacon Receiver. > Happy Skies, > Old Bob > * > > > * We at the airlines still use them on Cat2 approaches.................. ________________________________________________________________________________
Subject: Re: New Over-Voltage Protection Architecture
From: "user9253" <fransew(at)gmail.com>
Date: Mar 25, 2014
> Have you cited the right thread? These postings are all > dated back in the 2007-08 time frame. Hmm, the link works for me. http://www.vansairforce.com/community/showthread.php?t=111102 All 12 posting in this thread are from the last 2 or 3 days. Joe -------- Joe Gores Read this topic online here: http://forums.matronics.com/viewtopic.php?p=420992#420992 ________________________________________________________________________________
From: "Roger & Jean" <mrspudandcompany(at)verizon.net>
Subject: Re: New Over-Voltage Protection Architecture
Date: Mar 25, 2014
> >> Have you cited the right thread? These postings are all >> dated back in the 2007-08 time frame. > > Hmm, the link works for me. > http://www.vansairforce.com/community/showthread.php?t=111102 > All 12 posting in this thread are from the last 2 or 3 days. Perhaps Bob was looking at the (Join Date) for these posters and not the actual posting date. Roger ________________________________________________________________________________
From: BobsV35B(at)aol.com
Date: Mar 25, 2014
Subject: Re: Marker Beacon Remote Indicators?
Good Afternoon Sam, Just for kicks, I called up the ILS Rwy 9L CAT II & III at KORD. I find that to execute that approach you must have a GPS, airline or no airline. It uses many GPS distance fixes for the approach and the distances are from the ISAJ GPS fix which is colocated with the inner marker. Put ISAJ in lieu of the inner marker in your op specs and I will bet two bucks against a doughnut that it will be approved? Happy Skies, Old Bob In a message dated 3/25/2014 9:13:42 A.M. Central Daylight Time, sam.marlow(at)roadrunner.com writes: --> AeroElectric-List message posted by: Sam Marlow BobsV35B(at)aol.com wrote: > Good Afternoon Bob and Bill, > May I ask why you want the marker beacon indication? It is/was part > of the low frequency range system which was shut down over sixty years > ago. It has been retained for use with a few ILSs and some NDB > approaches, but such use is rapidly being discontinued. > I know of no fix designated by a marker beacon that cannot be found > via some other legal IFR data. > That is especially true if one has an IFR approved GPS installed. > There is nothing wrong with having a marker beacon receiver installed, > but you can eliminate an antenna and some wiring by getting rid of it. > You might even be able to save some panel space. > My vote is to forget about the marker Beacon Receiver. > Happy Skies, > Old Bob > * > > > * We at the airlines still use them on Cat2 approaches.................. Do Not Archive ________________________________________________________________________________
Date: Mar 26, 2014
From: "Robert L. Nuckolls, III" <nuckolls.bob(at)aeroelectric.com>
Subject: The lithium saga slogs onward . . .
In my quest to identify suppliers of lithium batteries who will share operational details of their BMS (battery management system) AND useful contained energy plots, few suppliers have demonstrated an interest in satisfying the unique curiosity of those who build and fly airplanes. So far, EarthX is the only one who has responded positively. I have documentation that describes their BMS operating philosophy. No family of discharge curves yet . . . but them, most of their customers are narrowly focused on engine cranking cupcakes with a frosting of weight reduction. I need to wrap of installment IV of the battery series for Kitplanes . . . as of this moment, I'm still short the necessary information to beat the drums in favor of ANY commercial off the shelf lithium product for OBAM aircraft. My misgivings are NOT rooted in either disappointment for weight savings or ability to get the engine started. The problem is centered on knowing what we don't know . . . exactly what size battery is needed to replace the legacy SVLA products in which we have considerable confidence. Watch this space . . . Bob . . . ________________________________________________________________________________
Subject: Re: DIY CBOVM: variations on a theme
From: "user9253" <fransew(at)gmail.com>
Date: Mar 26, 2014
I repeated testing of the over-voltage circuit today, doing a better job of connecting wires. If the voltage is steadily increased, the SCR fires at 16.2 volts. But if the voltage is set at 16.0 volts and held there, the SCR would fire after waiting a couple of minutes. Probing the input with a voltmeter intermittently caused the SCR to fire only when the voltage was above 15.9. Circuit instability, if any, is within 0.1 volt of the set-point. The adjustable voltage power supply only cost $6. Who knows how stable that is. The components are all through-hole, not surface mounted, and are mounted to a 1 x 2 inch area of PCB. I am happy with the circuit. I intend to connect the over-voltage circuit in series with a relay coil that is powered by a 3 amp fuse.. The normally closed contacts of the relay will be in series with the AC output of the permanent magnet dynamo. In case of high voltage, the SCR will fire, connecting one leg of the coil to ground to energize the relay, opening the AC circuit to the rectifier/voltage regulator. The SCR will continue to hold the relay energized until battery power is disconnected from the relay coil. This will not be a crowbar type of over-voltage protection. So no circuit breaker is required. High voltage will not blow the fuse. Joe -------- Joe Gores Read this topic online here: http://forums.matronics.com/viewtopic.php?p=421067#421067 ________________________________________________________________________________
Date: Mar 26, 2014
From: Jeff Luckey <jluckey(at)pacbell.net>
Subject: Re: DIY CBOVM: variations on a theme
Joe,=0A=0A1. How much power can the dynamo produce?- =0A2. Are there only 2 wires for the AC output? (I've never worked w/ a PM type alternator but most non-PM machines have 3 wires for AC output)=0A3. As you probably alrea dy know, on some relays, the current-handling capability of the NC contacts is less than the NO contacts=0A=0A=0A-Jeff=0A=0A=0A=0A____________________ ____________=0A From: user9253 <fransew(at)gmail.com>=0ATo: aeroelectric-list@ matronics.com =0ASent: Wednesday, March 26, 2014 7:40 AM=0ASubject: AeroEle ctric-List: Re: DIY CBOVM: variations on a theme=0A =0A=0A--> AeroElectric- List message posted by: "user9253" =0A=0AI repeated test ing of the over-voltage circuit today, doing a better job of connecting wir es.- If the voltage is steadily increased, the SCR fires at 16.2 volts. - But if the voltage is set at 16.0 volts and held there, the SCR would f ire after waiting a couple of minutes.- Probing the input with a voltmete r intermittently caused the SCR to fire only when the voltage was above 15. 9.- Circuit instability, if any, is within 0.1 volt of the set-point.- The adjustable voltage power supply only cost $6.- Who knows how stable t hat is.- The components are all through-hole, not surface mounted, and ar e mounted to a 1 x 2 inch area of PCB.- I am happy with the circuit.=0A - I intend to connect the over-voltage circuit in series with a relay coi l that is powered by a 3 amp fuse..- The normally closed contacts of the relay will be in series with the AC output of the permanent magnet dynamo. - In case of high voltage, the SCR will fire, connecting one leg of the c oil to ground to energize the relay, opening the AC circuit to the rectifie r/voltage regulator.- The SCR will continue to hold the relay energized u ntil battery power is disconnected from the relay coil.=0A- This will not be a crowbar type of over-voltage protection.- So no circuit breaker is required.- High voltage will not blow the fuse.=0AJoe=0A=0A--------=0AJoe Gores=0A=0A=0A=0A=0ARead this topic online here:=0A=0Ahttp://forums.matron ============== ________________________________________________________________________________
Date: Mar 26, 2014
From: "Robert L. Nuckolls, III" <nuckolls.bob(at)aeroelectric.com>
Subject: Continental Motors magneto data
Hat tip to Neal George who has provided us with a helpful library of service and training data on Continental's magneto systems. List members are invited to review the documents available at: http://tinyurl.com/llpc5lj . . . and download as their interest and needs dictate. Bob . . . ________________________________________________________________________________
Date: Mar 26, 2014
From: K <kleh(at)dialupatcost.ca>
Subject: Re: DIY CBOVM: variations on a theme
In regards to stability you might check that there is a capacitor across the voltage reference diode and another from the SCR gate to ground. If you happen to have an old circuit diagram dated before about 2005 it may not show that update. One nice thing about the crowbar and circuit breaker in the Z diagrams is that the voltage is brought under control immediately when the SCR fires. If I understand your post it sounds like your implementation waits for the relay to open and for any subsequent arcing to extinguish. Perhaps that is good enough but it seems preferable to use the circuit breaker as per the Z diagrams so that the SCR clamps the voltage faster. Might be other small advantages as well such as a subsequent dead battery could not cause the alternator to come back on line. Ken On 26/03/2014 10:40 AM, user9253 wrote: > > > I repeated testing of the over-voltage circuit today, doing a better > job of connecting wires. If the voltage is steadily increased, the > SCR fires at 16.2 volts. But if the voltage is set at 16.0 volts and > held there, the SCR would fire after waiting a couple of minutes. > Probing the input with a voltmeter intermittently caused the SCR to > fire only when the voltage was above 15.9. Circuit instability, if > any, is within 0.1 volt of the set-point. The adjustable voltage > power supply only cost $6. Who knows how stable that is. The > components are all through-hole, not surface mounted, and are mounted > to a 1 x 2 inch area of PCB. I am happy with the circuit. I intend > to connect the over-voltage circuit in series with a relay coil that > is powered by a 3 amp fuse.. The normally closed contacts of the > relay will be in series with the AC output of the permanent magnet > dynamo. In case of high voltage, the SCR will fire, connecting one > leg of the coil to ground to energize the relay, opening the AC > circuit to the rectifier/voltage regulator. The SCR will continue to > hold the relay energized until battery power is disconnected from the > relay coil. This will not be a crowbar type of over-voltage > protection. So no circuit breaker is required. High voltage will > not blow the fuse. Joe > > -------- Joe Gores > > > Read this topic online here: > > http://forums.matronics.com/viewtopic.php?p=421067#421067 > > ________________________________________________________________________________
Subject: Re: DIY CBOVM: variations on a theme
From: "user9253" <fransew(at)gmail.com>
Date: Mar 26, 2014
Jeff, 1. The Rotax 912 dynamo can generate up to about 20 amps. Although many recommend not loading it to more than 16 amps. My plane uses about 10 amps. 2. Yes, the AC output has two wires and is single phase. 3. If I am interpreting this data sheet correctly http://pewa.panasonic.com/assets/pcsd/catalog/cb-catalog.pdf the normally closed contacts can handle 30 amps. I suspect that, over the long term, corrosion due to lack of exercise will be more of a problem than the contact rating. Joe -------- Joe Gores Read this topic online here: http://forums.matronics.com/viewtopic.php?p=421074#421074 ________________________________________________________________________________
Date: Mar 26, 2014
From: "Robert L. Nuckolls, III" <nuckolls.bob(at)aeroelectric.com>
Subject: Re: New Over-Voltage Protection Architecture
>Hmm, the link works for me. >http://www.vansairforce.com/community/showthread.php?t=111102 >All 12 posting in this thread are from the last 2 or 3 days. Yeah, had a momentary short between the heaphones. Let's parse the narrative . . . Last Wednesday I flew from Melbourne to Hobart in my RV-7 (about an hour of the 2.5hr flight is over water - Bass Strait). Faultless day, faultless flight. Yesterday I jumped in my RV and flew from Hobart to Bruny Island (short flight - no problems). When I departed Bruny Island I got a Low Voltage Warning on my EFIS a couple of minutes after take off. I have an EXP2 Buss DC Load Center with indicator panel and GRT EFIS. EXP2 Indicator panel showed that the Alternator had switched off and the GRT was showing 13.3Volts and dropping. "Dropping" is not very quantified but the initial bus voltage excursion will be rather steep. A battery charges a 14.5 and discharges at 12.5 . . . so the first few seconds, probably less than a minute will show a steep drop of more than a volt. [] Note that this exemplar battery loaded to 1C starts 'working' at just over 12 volts. If he had been fitted with a flight worthy, 17 Ah battery and could reduce his loads to say 4A (.2C) his trip home would have been uneventful . . . I know you shouldn't turn the alternator off when the engine is running however I cycled the Alt Switch to see if i could get it to reset but no change resulted. There is no rational prohibition for turning the alternator on/off while the engine is running . . . particularly if there's a flight worthy battery in service. At this point I was over water so I requested a direct track to Hobart and landed to check it out (while I still had some volts). By the time I landed I didn't have enough volts to start the engine. I recharged the battery overnight. This morning I started the engine and the Alternator came back on line and operated as normal. After warm ups I obtained a clearance and departed Hobart....3 mins later the Alternator failed again and I returned to Hobart. What's wrong with this story? After the FIRST failure of the alternator, the battery was incapable of supporting sustained flight for more than a few minutes. Before I shut down I momentarily switched the master switch off and then immediately back on - this recycled the alternator and it was operating normally again. I suspected it may have been an over voltage condition because the EXP2 Buss has over-volt protection which turns off the Alternator and you have to remove battery power to reset it. I did a ground run for approximately 3 minutes at 2200rpm but it continued to operate normally (i thought this might cause it to fail). I shut down and removed the cowls and checked the belt and checked the connectors. Removed and refitted the connector on the back of the Alternator - all seemed fine. Following this I replaced the cowls and then inspected the EXP2 Buss to see if there were any loose connections. All seemed OK. I started the engine again and Alternator was operating normally after run-ups and a higher rpm check. I taxied out to the runway and I watched the indicator panel on departure...shortly after take off (before I had reduced the rpm) the indicator panel showed a high voltage warning which then reverted to a low voltage warning probably due to over volt protection kicking in. I completed the circuit and landed, tied down my RV and came to the conclusion that I wasn't going to fly back over Bass Strait in a hurry so I jumped on a commercial flight home. The peek-n-poke is a good and necessary part of the troubleshooting activity. His decision to use alternate transportation was prudent. We've discussed system reliability philosophies here on the list for many years. Crafting design goals should be followed with design, testing and maintenance to those goals. If this gentleman had crafted a plan-b with capable and confident endurance (can anybody say Z-13/8?) it's unlikely that this story would have been written. Yes, things break on airplanes. The real story here is not the difficulty with his alternator or it's associated controls. It's about his ABSOLUTE DEPENDENCE on the alternator for comfortable transition over deep water. So now the dilemma - I didn't bring the Alternator home with me so I can't bench test it. My thoughts are to purchase a replacement alternator and take it to Hobart and fit it to see if the problem goes away. If not, then I will have a spare alternator and the problem is more likely to be a fault in the over volt protection on the EXP2 Buss. Has anyone had a similar experience with the Vans 60amp Alternator and EXP2 Buss combination? If yes - what conclusions did you come to? I have had a previous Internal Voltage Regulator failure with this same Alternator which was repaired. I believe the previous failure may have been caused by turning on the alternator switch after start up. Ever since that point I have been turning on the Alternator switch prior to start up and turning off after shutdown. My Engine/Alternator has now done 193 hours. Any help/thoughts would be appreciated because when I fly back to pick up my RV I want to be confident that I can get it home. All the rest of this narrative is irrelevant to the core issues illustrated. This story isn't about resolving an alternator problem, it's about lack of SYSTEM reliability. Bob . . . ________________________________________________________________________________
Date: Mar 26, 2014
From: "Robert L. Nuckolls, III" <nuckolls.bob(at)aeroelectric.com>
Subject: Re: DIY CBOVM: variations on a theme
At 10:23 AM 3/26/2014, you wrote: >Joe, > >1. How much power can the dynamo produce? Most are limited to about 30A. The single phase (2-wire) offerings by B&C, Rotax and others are in the 8-20A class. The 3-phase, 3-wire devices for the larger 'tractors' can get you 30A or so. The problem with PM alternators is not the power generating ability of the mechanism but the power handling ability of the rectifier/ regulator. Unlike the 3-phase automotive alternator that runs output power through simple junction diodes. The regulator carries only a couple amps of field current. In the PM regulator, total output current of up to 30A flows through regulator semiconductors. Hence the heat sinks and limited size with respect to power. Bob . . . ________________________________________________________________________________
Subject: Re: DIY CBOVM: variations on a theme
From: "user9253" <fransew(at)gmail.com>
Date: Mar 26, 2014
Ken, Thanks for suggesting the capacitors. The circuit is stable unless the voltage gets very close to the trip point. That is OK with me as long as there is no nuisance tripping. Time will tell. Good point about the crowbar circuit acting immediately by clamping the supply to ground. The data sheet for my relay says it will operate in 15 milliseconds. I am hoping the battery will limit the voltage until the relay contacts open. I will try to land before the battery runs down enough (almost completely dead) to drop out the relay. Joe -------- Joe Gores Read this topic online here: http://forums.matronics.com/viewtopic.php?p=421078#421078 ________________________________________________________________________________
Date: Mar 26, 2014
From: "Robert L. Nuckolls, III" <nuckolls.bob(at)aeroelectric.com>
Subject: The lithium saga slogs onward . . .
FIY . . . got a peek inside this battery from True Blue Power . . . who has achieved the first TSO'd offering for GA aircraft. http://tinyurl.com/lxh6wqm The battery is 17Ah (in AIRCRAFT Ah at 1-hour - not a 20 hour rate) and 24V. Weighs about 16 pounds. So a 12v, 17Ah battery would be about half that weight. TB17 This battery is an array of 56 cylindrical cells not unlike these 26650 cells available from a variety of sources. 1pc 3.2V LiFePO4 Li-ion LFP26650 energy type 3200mAh flat cap b In rough numbers, this cell is a 10Wh storage medium. 56 cells x 10Wh suggests you can expect about 560Wh from the array. This is consistent with nameplate energy rating for the battery at 430Wh. Being one of the few batteries where detailed system integration data are available, we can begin to size the task for incorporation of this or similar products into airplanes. A 12V version of the battery would be 215Wh so if you have an electrically dependent engine with a 100W service load, you could expect about 2 hours endurance for just an engine. Peel off 50-73 watts for electro-whizzies, your battery only endurance expectations are just over 1 hour for new battery, just under 1 hour for end-of-life battery. Now for the sobering facts. There are two circuit boards in the battery management system (BMS). One is just under the heat-sink fins on top of the battery. It's populated with a goodly number of FAT transistors, probably N-MOS FETs in TO-247 plastic packages. The other board occupies a whole side of the battery and is covered with perhaps 2-3 dozen surface mount ic's and lots of jelly-bean parts. There's probably more silicon in this battery than in your ICOM comm transceiver, gps and transponder combined. Further, I believe this battery will sell for about $3200; so one might ball-park a 12v battery at $1600. Of course, that's 'airplane' pricing that carries the burden of TSO certification. This product is the leading edge example of what it takes to get approval for TC aviation. Other product MIGHT offer similar capabilities for $less$ if they've not run the TSO gauntlet. Just giving you guys a heads-up on what I'm discovering as that battery articles move forward . . . The REAL significance of this discovery exercise is that to get 1-hour of battery only endurance in a 200W airplane will take an array of 28 lithium cells of the size cited above. This means that batteries like this [] http://tinyurl.com/k8yhurl which are 4 x 4 arrays of similar cells probably fall short of the 1-hour endurance hypothetical half-a-True-Blue endurance cited above. Further, this pretty little red thing cannot feature the same degree of battery management that's built into the True Blue product. Tying this back to the venerable PC680 . . . Emacs! We see that teh PC680 at the 1-hour rate is a 12.3Ah battery when new. A 175W airplane will say lit up for about 45 minutes on a new battery. Bob . . . ________________________________________________________________________________
Date: Mar 26, 2014
From: "Robert L. Nuckolls, III" <nuckolls.bob(at)aeroelectric.com>
Subject: Re: DIY CBOVM: variations on a theme
At 11:45 AM 3/26/2014, you wrote: > >In regards to stability you might check that there is a capacitor >across the voltage reference diode and another from the SCR gate to >ground. If you happen to have an old circuit diagram dated before >about 2005 it may not show that update. > >One nice thing about the crowbar and circuit breaker in the Z >diagrams is that the voltage is brought under control immediately >when the SCR fires. If I understand your post it sounds like your >implementation waits for the relay to open and for any subsequent >arcing to extinguish. Perhaps that is good enough but it seems >preferable to use the circuit breaker as per the Z diagrams so that >the SCR clamps the voltage faster. Might be other small advantages >as well such as a subsequent dead battery could not cause the >alternator to come back on line. For the run-of-the-mill ov conditions with a good battery, an ov even is not one of extreme urgency. The ships air-worthy battery will stand off a runaway alternator for significant intervals. If one waited 100 mS to evaluate an ov condition for a true runaway alternator, it wouldn't be a bad thing. In fact, the next generation ov module will do just that. Sensing an ov condition start a 100mS timer that continue to run as long as the voltage does not drop back below 16V before the timer runs out. On the other hand, a second threshold at 20V assumes no air worthy battery is present and an immediate trip is initiated. We're still going to pull down on the field supply breaker . . . Bob . . . ________________________________________________________________________________
Date: Mar 27, 2014
From: "Robert L. Nuckolls, III" <nuckolls.bob(at)aeroelectric.com>
Subject: Photo needed
I'm needing a nose-on picture of an OBAM aircaft for use on the counter display for the AEC9012 programmable wig-wag at Osh . . . Thanks! Bob . . . ________________________________________________________________________________
Subject: Re: Marker Beacon Remote Indicators?
From: "donjohnston" <don@velocity-xl.com>
Date: Mar 28, 2014
BobsV35B(at)aol.com wrote: > What I am trying to do is help folks save money, space and weight on their airplanes. > > Seems any serious IFR pilot will have a GPS and, with a GPS, the marker beacon becomes superfluous. Just trying to save the troops a few bucks and some space in their flying machine. > > Happy Skies, > > Old Bob > I'm in the process of connecting my avionics. I installed a marker beacon antenna only to discover that the audio panel I'm installing doesn't have marker beacon indicators. I contacted the manufacture and they explained that marker beacon transmitters will start to be decommissioned starting next year. So they decided to remove that function. Read this topic online here: http://forums.matronics.com/viewtopic.php?p=421197#421197 ________________________________________________________________________________
Date: Mar 31, 2014
From: "Robert L. Nuckolls, III" <nuckolls.bob(at)aeroelectric.com>
Subject: Re: Photo needed
At 07:29 PM 3/27/2014, you wrote: > > >I'm needing a nose-on picture of an OBAM aircaft >for use on the counter display for the AEC9012 >programmable wig-wag at Osh . . . Thanks to all who have contributed to this call for photos! I've received some very nice suggestions . . . some of which are excellent candidates for the cover of R13. It has been suggested that I clarify my original request. I'm crafting a counter-top display for the AEC9012 programmable wig-wag. I'd like have a full wing span, nose-on view of an airplane on which I'll mount white leds out on the wings . . . and paste the whole over a background that emulates an in-flight scenario. Hence, the photo need not . . . be of an airplane in flight, sitting on the ground would be just fine. Preferably a tricycle gear so that the tail feathers are disposed in a 'level flight' position to the nose. Bob . . . ________________________________________________________________________________
Date: Mar 31, 2014
From: Jan de Jong <jan_de_jong(at)casema.nl>
Subject: Re: DIY CBOVM: variations on a theme
That's another issue with a lithium battery - it does not keep OV down well - even if not fitted with a BMS. And it seriously dislikes overvoltage. It thus requires a fast OV switch at the alternator. And if it has a BMS it will also have a battery OV cutoff switch built in. Making the battery unavailable just when it is needed. My conclusion has been that if I want Lithium I will have to make the under/over cell voltage and cell temperature monitoring and top balancing myself. With backed up OV protection at the alternator. And have the individual cells accessible to thermistors. Jan de Jong On 3/26/2014 8:45 PM, Robert L. Nuckolls, III wrote: > > > At 11:45 AM 3/26/2014, you wrote: >> >> In regards to stability you might check that there is a capacitor >> across the voltage reference diode and another from the SCR gate to >> ground. If you happen to have an old circuit diagram dated before >> about 2005 it may not show that update. >> >> One nice thing about the crowbar and circuit breaker in the Z >> diagrams is that the voltage is brought under control immediately >> when the SCR fires. If I understand your post it sounds like your >> implementation waits for the relay to open and for any subsequent >> arcing to extinguish. Perhaps that is good enough but it seems >> preferable to use the circuit breaker as per the Z diagrams so that >> the SCR clamps the voltage faster. Might be other small advantages as >> well such as a subsequent dead battery could not cause the alternator >> to come back on line. > > For the run-of-the-mill ov conditions with a good > battery, an ov even is not one of extreme urgency. > The ships air-worthy battery will stand off a runaway > alternator for significant intervals. If one waited > 100 mS to evaluate an ov condition for a true runaway > alternator, it wouldn't be a bad thing. > > In fact, the next generation ov module will do just > that. Sensing an ov condition start a 100mS timer > that continue to run as long as the voltage does not > drop back below 16V before the timer runs out. > > On the other hand, a second threshold at 20V assumes > no air worthy battery is present and an immediate trip > is initiated. > > We're still going to pull down on the field supply > breaker . . . > > > Bob . . . > > ________________________________________________________________________________
Date: Mar 31, 2014
From: "Robert L. Nuckolls, III" <nuckolls.bob(at)aeroelectric.com>
Subject: Re: DIY CBOVM: variations on a theme
At 12:15 PM 3/31/2014, you wrote: > >That's another issue with a lithium battery - it does not keep OV >down well - even if not fitted with a BMS. And it seriously dislikes >overvoltage. It thus requires a fast OV switch at the alternator. >And if it has a BMS it will also have a battery OV cutoff switch >built in. Making the battery unavailable just when it is needed. > >My conclusion has been that if I want Lithium I will have to make >the under/over cell voltage and cell temperature monitoring and top >balancing myself. >With backed up OV protection at the alternator. >And have the individual cells accessible to thermistors. Your perceptions of being inadequately informed with respect to operating details of the various lithium products are accurate. There is no industry standard for the term "BMS". I'm working the 4th installment on the series of battery articles for Kitplanes. The past 7 weeks have been enlightening. I'm not sure that alternator OV protection needs to be anything beyond the ordinary . . . I'm still stirring the stew of simple-ideas . . . Bob . . . ________________________________________________________________________________
Date: Mar 31, 2014
From: Jan de Jong <jan_de_jong(at)casema.nl>
Subject: Re: DIY CBOVM: variations on a theme
The implicit "trust us" attitude is in any case unwarranted. Full specifications of the built-in electronics are absolutely necessary. What I have seen is not very compatible with aircraft use. We do not want OV cut-out built-in (should be done somewhere up in the charging path). We probably do not want LV cut-out built-in either (saving the battery may not always be the highest goal). But we want monitoring and warning. And as a pilot I want to know about temperatures. When used well (correct voltages and currents within limits, no recharging ever after full discharge) the remaining danger is a manufacturing fault, with temperature discrepancy the only available signal. And a measure of top balancing is needed unless we are willing to adopt a discipline of doing that externally. A certain minimum balancing current per Ah, say 50mA or so is required. I've seen balancing currents quoted for Chinese BMS products that are likely useless for any realistic battery size (they may be intended for UPS batteries - very long charging times and smallish charging currents). And the balancing algorithm is often primitive - most balancers only balance at the very end of the charging process by comparing a cell voltage with a high limit-voltage, not during the charging process by comparing a cell voltage with the voltages of other cells. The latter allows good balancing with a lower alternator set voltage. In exchange for a few percent of charge you get much longer life - apparently. I wish you wisdom with the 4th article on batteries (great articles so far, I approve of the current editor in chief of KP!) Jan de Jong On 3/31/2014 8:12 PM, Robert L. Nuckolls, III wrote: > > > At 12:15 PM 3/31/2014, you wrote: >> >> >> That's another issue with a lithium battery - it does not keep OV >> down well - even if not fitted with a BMS. And it seriously dislikes >> overvoltage. It thus requires a fast OV switch at the alternator. >> And if it has a BMS it will also have a battery OV cutoff switch >> built in. Making the battery unavailable just when it is needed. >> >> My conclusion has been that if I want Lithium I will have to make the >> under/over cell voltage and cell temperature monitoring and top >> balancing myself. >> With backed up OV protection at the alternator. >> And have the individual cells accessible to thermistors. > > Your perceptions of being inadequately informed > with respect to operating details of the various > lithium products are accurate. > > There is no industry standard for the term "BMS". > I'm working the 4th installment on the series > of battery articles for Kitplanes. The past 7 > weeks have been enlightening. > > I'm not sure that alternator OV protection needs > to be anything beyond the ordinary . . . I'm still > stirring the stew of simple-ideas . . . > > > Bob . . . > ________________________________________________________________________________
Date: Mar 31, 2014
From: "Robert L. Nuckolls, III" <nuckolls.bob(at)aeroelectric.com>
Subject: Photo needed
At 03:40 PM 3/31/2014, you wrote: >Bob, > >I couldn't find a plain front on shot that was suitable. I don't >suppose the attached would be any good though? > >All the shots I sent are my own work. Yes! That will work! Thanks. Unless you're submitting candidate photos for R13 front cover, know that Ed has nicely filled my needs for the AEC9012 wig-wag display. We're still compiling a library of R13 Cover photos. Just as we did last time, I'll probably pick 5 or 6 from what's offered and the AeroElectric-List members will pick the winner. Thanks to all who assisted with the current project. Bob . . . ________________________________________________________________________________
Date: Mar 31, 2014
From: "Robert L. Nuckolls, III" <nuckolls.bob(at)aeroelectric.com>
Subject: Re: DIY CBOVM: variations on a theme
At 03:00 PM 3/31/2014, you wrote: The implicit "trust us" attitude is in any case unwarranted. Full specifications of the built-in electronics are absolutely necessary. that will be a primary premise of the 'last' article. What I have seen is not very compatible with aircraft use. We do not want OV cut-out built-in (should be done somewhere up in the charging path). This is certainly the legacy philosophy for lead-acid and/or Ni-Cad batteries wherein the battery was a 'current sink' of demonstrable value for managing a runaway generator/alternator. But let's assume for the moment that one of our future design goals is to craft and prove airworthiness of an alternator-only operation protocol. Now we need to show that the runaway alternator can be controlled within the DO160/MilSTD704 envelope. If that proves to be practical, then the electronics in the battery need not assume any duties beyond those for protecting the battery . . . the battery MIGHT be relieved of duty for standing guard at the gates standing off runaway alternators. We probably do not want LV cut-out built-in either (saving the battery may not always be the highest goal). But we want monitoring and warning. Yeah . . . but it depends on WHERE the l.v. cutout occurs. Toyota writes software intended to maintain their lithium cells at some place between 30 and 85% capacity. This is the ENERGY operating range that offers the advertised cell life. But they don't force disconnect the battery . . . as far as I've been able to learn so far. Depending on the BMS designer's charter from on-high . . . he may well have other bounds. Without advocating for any particular energy range . . . I'll suggest that any offering that includes a BMS would do well to tell all . . . inquiring pilots NEED to know. And as a pilot I want to know about temperatures. When used well (correct voltages and currents within limits, no recharging ever after full discharge) the remaining danger is a manufacturing fault, with temperature discrepancy the only available signal. And a measure of top balancing is needed unless we are willing to adopt a discipline of doing that externally. A certain minimum balancing current per Ah, say 50mA or so is required. I've seen balancing currents quoted for Chinese BMS products that are likely useless for any realistic battery size (they may be intended for UPS batteries - very long charging times and smallish charging currents). Exactly. An these issues drive cost of ownership. Yes, the lithium products have stellar performance in some venues but with LIMITS that much be UNDERSTOOD and incorporated into the user's operating philosophy. $time$ fiddling with batteries is $time$ not available to go flying. And the balancing algorithm is often primitive - most balancers only balance at the very end of the charging process by comparing a cell voltage with a high limit-voltage, not during the charging process by comparing a cell voltage with the voltages of other cells. The latter allows good balancing with a lower alternator set voltage. In exchange for a few percent of charge you get much longer life - apparently. "Apparently" is the operative term. The family of lithium cells available for incorporation into finished goods is not large . . . There's probably no more than a half dozen shakers-n-movers in the cylindrical cell market. They're all pretty big names . . . I doubt that core capabilities of the range of offerings varies much. But we see a large variation in proposed operating philosophies across the spectrum of "assemblers" who package these cells up for market. I wish you wisdom with the 4th article on batteries (great articles so far, I approve of the current editor in chief of KP!) Thank you my friend. I'm learning. But I wish that the questions were being answered faster than they surface! This first past at lithium-for-airplanes will probably be disappointing for many who hope that a an ideal drop-in-replacement for SVRLA is going to emerge. But as you've noted, no matter what the advertising hype says, "Their ain't no such thing as a lithium drop-in for lead-acid." For anything beyond the day-vfr machine with gravity flow fuel and magnetos, the prudent owner/pilot would do well to understand the limitations. Further, assuming a willingness to $invest$ in weight savings know that changes to ownership and operating philosophy are inevitable. Thank you for contributing to the dialog . . . Bob . . . ________________________________________________________________________________
Subject: Simple Diode Tester
From: "Eric M. Jones" <emjones(at)charter.net>
Date: Mar 31, 2014
Simple Diode Tester. Here is a simple diode tester you may find useful: http://www.energpolarit.com/docs/muscletest_withdirect.pdf -------- Eric M. Jones www.PerihelionDesign.com 113 Brentwood Drive Southbridge, MA 01550 (508) 764-2072 emjones(at)charter.net Read this topic online here: http://forums.matronics.com/viewtopic.php?p=421409#421409 ________________________________________________________________________________
Date: Mar 31, 2014
Subject: Re: Simple Diode Tester
From: "Jeff B." <loboflyer(at)gmail.com>
Just think! If you apply this to anything experiencing interference on your airplane, you can solve the problem! If you apply it to your airplane it might become invisible to radar! So, how hard do we tap to wake the electrical system up? -Jeff- On Mon, Mar 31, 2014 at 6:59 PM, Eric M. Jones wrote: > emjones(at)charter.net> > > Simple Diode Tester. > > Here is a simple diode tester you may find useful: > > http://www.energpolarit.com/docs/muscletest_withdirect.pdf > > -------- > Eric M. Jones > www.PerihelionDesign.com > 113 Brentwood Drive > Southbridge, MA 01550 > (508) 764-2072 > emjones(at)charter.net > > > Read this topic online here: > > http://forums.matronics.com/viewtopic.php?p=421409#421409 > > ________________________________________________________________________________
Date: Apr 01, 2014
From: Jan de Jong <jan_de_jong(at)casema.nl>
Subject: Re: DIY CBOVM: variations on a theme
> What I have seen is not very compatible with aircraft use. We do not > want OV cut-out built-in (should be done somewhere up in the charging > path). > > This is certainly the legacy philosophy for lead-acid > and/or Ni-Cad batteries wherein the battery was a > 'current sink' of demonstrable value for managing > a runaway generator/alternator. > > But let's assume for the moment that one of our > future design goals is to craft and prove airworthiness > of an alternator-only operation protocol. Now we need > to show that the runaway alternator can be controlled > within the DO160/MilSTD704 envelope. > > If that proves to be practical, then the electronics > in the battery need not assume any duties beyond those > for protecting the battery . . . the battery MIGHT be > relieved of duty for standing guard at the gates > standing off runaway alternators. The current lead-acid battery has 2 tasks when an OV event occurs: 1. slowing the event down to facilitate OVP without false triggering 2. keeping the battery-connected busses powered when the charging system responsible for the OV event is disconnected from the bus Both tasks must be covered or made unnecessary with any new alternative use of a battery. Jan de Jong ________________________________________________________________________________
Date: Apr 01, 2014
From: "Robert L. Nuckolls, III" <nuckolls.bob(at)aeroelectric.com>
Subject: Re: Photo needed
Got the picture I needed for the present task . . . thanks to all who helped out. Also received some pictures that I think are good candidates for the cover of R13. I'll be archiving these for future reference. As the reprint date grows near, I'll be asking for any additional submissions. The List members voted on the most suitable image for R12 . . . I'm inclined to do the same for R13. Bob . . . ________________________________________________________________________________
Subject: Icom A210 Low Headset Feedback once engine starts
From: "Nik" <nikpotter(at)web.de>
Date: Apr 03, 2014
Hello, I installed the Icom A210 into my Rebel and it tests out fine on all parameters...except when the engine starts! The RX volume is too low to hear regardless of how high I set the Headset volume or play with the squelch. I have tried various headsets to eliminate that the problem lies with my ANR headsets. I have gone back through and tested all wires and proofed that the magnetos are properly ground. From the very low volume I can ascertain no noise/hash so I don't think the issue is suppressors or other interference. My system does not have a noise suppressor on the alternator, but as I said, I'm not hearing any interference. I suspect that the radio is picking up on the cabin noise and canceling out the sound - no idea. Any suggestions greatly appreciated. Read this topic online here: http://forums.matronics.com/viewtopic.php?p=421526#421526 ________________________________________________________________________________
Date: Apr 03, 2014
From: "Robert L. Nuckolls, III" <nuckolls.bob(at)aeroelectric.com>
Subject: Re: Icom A210 Low Headset Feedback once engine starts
At 04:41 AM 4/3/2014, you wrote: > >Hello, >I installed the Icom A210 into my Rebel and it tests out fine on all >parameters...except when the engine starts! The RX volume is too >low to hear regardless of how high I set the Headset volume or play >with the squelch. I have tried various headsets to eliminate that >the problem lies with my ANR headsets. I have gone back through and >tested all wires and proofed that the magnetos are properly >ground. From the very low volume I can ascertain no noise/hash so I >don't think the issue is suppressors or other interference. Let's make sure I understand. While operating battery only, engine off, volume is normal . . . in fact, you can turn it up so far that it's uncomfortably loud? >My system does not have a noise suppressor on the alternator, but as >I said, I'm not hearing any interference. Did you try operating the radio with engine running but with alternator turned off? >I suspect that the radio is picking up on the cabin noise and >canceling out the sound - no idea. >Any suggestions greatly appreciated. That radio doesn't have anything you might call a 'smart audio system'. Try listening with the headset microphone unplugged . . . i.e. headphones only to see if that makes a difference. Except for received signal volume, does the "squelch test function" seem to work? In other words, opening the squelch under any engine operating condition produce the normal 'dead air' hiss from the receiver? . . . and it simply becomes lower volume after the engine is running? Bob . . . ________________________________________________________________________________
Date: Apr 06, 2014
From: Jeff Luckey <jluckey(at)pacbell.net>
Subject: Magneto Noise
=0A=0AI flew w/ a friend recently and when trying to get awos for our desti nation airport (which is a pretty weak signal) it was almost impossible to hear due to magneto noise.- The radio is a Garmin 430W in a Cherokee and we were around 10 miles from the airport.- When we got within 3 or 4 mile s the signal was much clearer.- All other transmissions with ATC & towers were acceptable but I noticed at high volume I could clearly hear magneto noise in the background.=0A=0A=0AI'm wondering if that's just the way it is or if there is a problem with the magnetos or P lead wiring.=0A=0A=0AThere must be one of Bob's handy-dandy write-ups on this topic but a quick searc h of AeroElectric.com was in vain.=0A=0ACan someone point me in the right d irection?=0A=0ATIA,=0A=0A-Jeff=0A ________________________________________________________________________________
Date: Apr 06, 2014
From: Tim Andres <tim2542(at)sbcglobal.net>
Subject: Re: Magneto Noise
Not normal. Does it go away when you switch either mag off? YOu sure its a mag?=0ATim=0A=0A=0A=0A=0A________________________________=0A From: Jeff Luc key =0ATo: "aeroelectric-list(at)matronics.com" =0ASent: Sunday, April 6, 2014 10:04 AM=0ASubject: AeroElectric-List: Magneto Noise=0A =0A=0A=0A=0A=0AI flew w/ a friend rece ntly and when trying to get awos for our destination airport (which is a pr etty weak signal) it was almost impossible to hear due to magneto noise.- The radio is a Garmin 430W in a Cherokee and we were around 10 miles from the airport.- When we got within 3 or 4 miles the signal was much clearer .- All other transmissions with ATC & towers were acceptable but I notice d at high volume I could clearly hear magneto noise in the background.=0A =0A=0AI'm wondering if that's just the way it is or if there is a problem w ith the magnetos or P lead wiring.=0A=0A=0AThere must be one of Bob's handy -dandy write-ups on this topic but a quick search of AeroElectric.com was i n vain.=0A=0ACan someone point me in the right direction?=0A=0ATIA,=0A=0A-J ==================== ________________________________________________________________________________
From: "David Lloyd" <skywagon(at)charter.net>
Subject: Re: Magneto Noise
Date: Apr 06, 2014
I would suspect the "shield" part of the P-lead cable has broken and is no longer grounded at the mag. housing. 2nd, a small chance that the heavy grounding cable from the engine to aircraft frame is missing or not making a full connection due to corrosion or other flaw. Dave ------------------------------------------------------------------------- ------- ----- Original Message ----- From: Tim Andres To: aeroelectric-list(at)matronics.com Sent: Sunday, April 06, 2014 12:13 PM Subject: Re: AeroElectric-List: Magneto Noise Not normal. Does it go away when you switch either mag off? YOu sure its a mag? Tim ------------------------------------------------------------------------- ----- From: Jeff Luckey <jluckey(at)pacbell.net> To: "aeroelectric-list(at)matronics.com" Sent: Sunday, April 6, 2014 10:04 AM Subject: AeroElectric-List: Magneto Noise I flew w/ a friend recently and when trying to get awos for our destination airport (which is a pretty weak signal) it was almost impossible to hear due to magneto noise. The radio is a Garmin 430W in a Cherokee and we were around 10 miles from the airport. When we got within 3 or 4 miles the signal was much clearer. All other transmissions with ATC & towers were acceptable but I noticed at high volume I could clearly hear magneto noise in the background. I'm wondering if that's just the way it is or if there is a problem with the magnetos or P lead wiring. There must be one of Bob's handy-dandy write-ups on this topic but a quick search of AeroElectric.com was in vain. Can someone point me in the right direction? TIA, -Jeff http://www.matronics.com/Navigator?AeroElectri" href="http://forums.matronics.com/">http://forums.matronics.com _="nofollow" target="_blank" href="http://www.matronics.com/contributi= ________________________________________________________________________________
From: "mick muller" <mmul6471(at)bigpond.net.au>
Subject: RE: AeroElectric-List Digest: 3 Msgs - 04/06/14
Date: Apr 07, 2014
WE had a similar problem with noise, and the issue turned out to be the Condensers in the Magneto. This was despite the Mag having been overhauled about 170 hours prior to that. Mick -----Original Message----- From: owner-aeroelectric-list-server(at)matronics.com [mailto:owner-aeroelectric-list-server(at)matronics.com] On Behalf Of AeroElectric-List Digest Server Sent: Monday, 7 April 2014 4:58 PM Subject: AeroElectric-List Digest: 3 Msgs - 04/06/14 * ================================================= Online Versions of Today's List Digest Archive ================================================= Today's complete AeroElectric-List Digest can also be found in either of the two Web Links listed below. The .html file includes the Digest formatted in HTML for viewing with a web browser and features Hyperlinked Indexes and Message Navigation. The .txt file includes the plain ASCII version of the AeroElectric-List Digest and can be viewed with a generic text editor such as Notepad or with a web browser. HTML Version: http://www.matronics.com/digest/digestview.php?Style=82701&View=html&Chapter 14-04-06&Archive=AeroElectric Text Version: http://www.matronics.com/digest/digestview.php?Style=82701&View=txt&Chapter 2014-04-06&Archive=AeroElectric =============================================== EMail Version of Today's List Digest Archive =============================================== ---------------------------------------------------------- AeroElectric-List Digest Archive --- Total Messages Posted Sun 04/06/14: 3 ---------------------------------------------------------- Today's Message Index: ---------------------- 1. 10:05 AM - Magneto Noise (Jeff Luckey) 2. 12:14 PM - Re: Magneto Noise (Tim Andres) 3. 01:01 PM - Re: Magneto Noise (David Lloyd) ________________________________ Message 1 _____________________________________ From: Jeff Luckey <jluckey(at)pacbell.net> Subject: AeroElectric-List: Magneto Noise =0A=0AI flew w/ a friend recently and when trying to get awos for our desti nation airport (which is a pretty weak signal) it was almost impossible to hear due to magneto noise.- The radio is a Garmin 430W in a Cherokee and we were around 10 miles from the airport.- When we got within 3 or 4 mile s the signal was much clearer.- All other transmissions with ATC & towers were acceptable but I noticed at high volume I could clearly hear magneto noise in the background.=0A=0A=0AI'm wondering if that's just the way it is or if there is a problem with the magnetos or P lead wiring.=0A=0A=0AThere must be one of Bob's handy-dandy write-ups on this topic but a quick searc h of AeroElectric.com was in vain.=0A=0ACan someone point me in the right d irection?=0A=0ATIA,=0A=0A-Jeff=0A ________________________________ Message 2 _____________________________________ From: Tim Andres <tim2542(at)sbcglobal.net> Subject: Re: AeroElectric-List: Magneto Noise Not normal. Does it go away when you switch either mag off? YOu sure its a mag?=0ATim=0A=0A=0A=0A=0A________________________________=0A From: Jeff Luc key =0ATo: "aeroelectric-list(at)matronics.com" =0ASent: Sunday, April 6, 2014 10:04 AM=0ASubject: AeroElectric-List: Magneto Noise=0A =0A=0A=0A=0A=0AI flew w/ a friend rece ntly and when trying to get awos for our destination airport (which is a pr etty weak signal) it was almost impossible to hear due to magneto noise.- The radio is a Garmin 430W in a Cherokee and we were around 10 miles from the airport.- When we got within 3 or 4 miles the signal was much clearer .- All other transmissions with ATC & towers were acceptable but I notice d at high volume I could clearly hear magneto noise in the background.=0A =0A=0AI'm wondering if that's just the way it is or if there is a problem w ith the magnetos or P lead wiring.=0A=0A=0AThere must be one of Bob's handy -dandy write-ups on this topic but a quick search of AeroElectric.com was i n vain.=0A=0ACan someone point me in the right direction?=0A=0ATIA,=0A=0A-J ==================== ________________________________ Message 3 _____________________________________ From: "David Lloyd" <skywagon(at)charter.net> Subject: Re: AeroElectric-List: Magneto Noise I would suspect the "shield" part of the P-lead cable has broken and is no longer grounded at the mag. housing. 2nd, a small chance that the heavy grounding cable from the engine to aircraft frame is missing or not making a full connection due to corrosion or other flaw. Dave ------------------------------------------------------------------------- ------- ----- Original Message ----- From: Tim Andres To: aeroelectric-list(at)matronics.com Sent: Sunday, April 06, 2014 12:13 PM Subject: Re: AeroElectric-List: Magneto Noise Not normal. Does it go away when you switch either mag off? YOu sure its a mag? Tim ------------------------------------------------------------------------- ----- From: Jeff Luckey <jluckey(at)pacbell.net> To: "aeroelectric-list(at)matronics.com" Sent: Sunday, April 6, 2014 10:04 AM Subject: AeroElectric-List: Magneto Noise I flew w/ a friend recently and when trying to get awos for our destination airport (which is a pretty weak signal) it was almost impossible to hear due to magneto noise. The radio is a Garmin 430W in a Cherokee and we were around 10 miles from the airport. When we got within 3 or 4 miles the signal was much clearer. All other transmissions with ATC & towers were acceptable but I noticed at high volume I could clearly hear magneto noise in the background. I'm wondering if that's just the way it is or if there is a problem with the magnetos or P lead wiring. There must be one of Bob's handy-dandy write-ups on this topic but a quick search of AeroElectric.com was in vain. Can someone point me in the right direction? TIA, -Jeff http://www.matronics.com/Navigator?AeroElectri" href="http://forums.matronics.com/">http://forums.matronics.com _="nofollow" target="_blank" href="http://www.matronics.com/contributi ----- No virus found in this message. Checked by AVG - www.avg.com ________________________________________________________________________________
Date: Apr 08, 2014
From: "Robert L. Nuckolls, III" <nuckolls.bob(at)aeroelectric.com>
Subject: Re: Magneto Noise
At 12:04 PM 4/6/2014, you wrote: I flew w/ a friend recently and when trying to get awos for our destination airport (which is a pretty weak signal) it was almost impossible to hear due to magneto noise. The radio is a Garmin 430W in a Cherokee and we were around 10 miles from the airport. When we got within 3 or 4 miles the signal was much clearer. All other transmissions with ATC & towers were acceptable but I noticed at high volume I could clearly hear magneto noise in the background. I'm wondering if that's just the way it is or if there is a problem with the magnetos or P lead wiring. Was this a new condition or has it been that way 'forever'. It's difficult for the neophyte observer to tell the difference between ignition noises getting into the system by way of plug wire radiation versus p-lead radiation. First, make sure that the perceived noise goes up and down with volume control settings on radio. This all but guarantees that it's coming in through the antenna. Then, disconnect both p-leads from the back of the magnetos and run the engine. See if the noise is still heard . . . turn to unused frequency . . . open squelch. If you can still hear it, then look for bad plug wire. If not, hook up one p-lead at a time and repeat experiment . . . see if noise is predicated on either or both of the p-leads being hooked up. Once you've identified the offending radiator, you need to figure out why. If this is a 'new' condition, then you're looking for something that has changed. Bad cap in mag (probably not both) broken ground lead on p-lead shield. If possible/ practical wire p-lead wires per Figure Z-27 http://tinyurl.com/n3oy37f Ground p-lead shields at magneto end only . . . use p-lead shields to ground mag switch. Remove and existing ground to the mag switch. If the noise is coming from both p-leads, there are 'approved' magneto noise filters http://tinyurl.com/l3s4mrj that most mechanics will install with a simple log-book entry. Bob . . . Bob . . . ________________________________________________________________________________
Date: Apr 08, 2014
From: Thomas Blejwas <tomblejwas(at)yahoo.com>
Subject: When are diodes needed?
In adapting Z-07 and/or Z-19, I'm confused about the use and purpose of dio des.- As I understand them, diodes allow current to flow in one direction , with a small voltage drop, but not in the opposite direction, except for a small leakage.- The voltage drop times the forward current yields a pow er loss that results in heat, which may need to be dissipated to keep the h eat from damaging the diode or other nearby equipment.- The power lost ma y also be an issue in a battery-only situation after loss of an alternator, for example.- O.K. so far?=0A=0APairs of diodes are used in Z-19 for the two battery feeds for the main fuel pump, for example.- When both feeds are on, this would keep a "good" battery from feeding a "bad" one.- Is th at the primary purpose?- In Z-07, only one feed to the bus with the fuel pump has a diode because judicious switching can isolate either battery.- Correct?- Is there something more that I'm missing?- For example, if I 'm not concerned about one battery "feeding" another, do I still need diode s to "smooth" the feed to a bus from two different batteries?- Thanks.=0A =0ATom=0A=0ASent from my iPad ________________________________________________________________________________
Subject: Z13/8 Main Alternator.
From: "MikeDunlop" <mdunlop001(at)aol.com>
Date: Apr 09, 2014
I'm finishing off my electrical system in a Long-EZ that includes the SD-8, exactly as per the Z13/8 and have to make a final choice of the main alternator. For cost and availability locally I've decided to go with either a Lucas 60amp or 70amp. The advice I'm seeking is regarding the regulator, these units have an internal regulator, so do I keep the internal regulator? (remember the Z13/8 has the OV protection between the regulator and the bus) or do I do a conversion to use a B&C external regulator (I have a spare one I could use). I've read many, many threads on OV and runaway alternators etc. and am not too sure if using the internal regulator in the Z13/8 architecture will give me complete protection. Regards Mike Dunlop (UK) Read this topic online here: http://forums.matronics.com/viewtopic.php?p=421821#421821 ________________________________________________________________________________
Subject: Re: When are diodes needed?
From: "user9253" <fransew(at)gmail.com>
Date: Apr 09, 2014
It is unlikely that a battery will be bad enough that the other "good" battery would feed it. A more important reason for isolating the two fuel pump circuits in Z-19 is to prevent alternator charging current from back-feeding through small wires and fuses intended to carry only the fuel pump load. There is more than one version of Z-07. Looking at revision 5, the diode in the "Engine B" wire prevents engine starting current and main bus load current from flowing through the "Engine B" wire from the Aux battery. I do not know what you mean by diodes "smoothing" the feed. Joe -------- Joe Gores Read this topic online here: http://forums.matronics.com/viewtopic.php?p=421822#421822 ________________________________________________________________________________
Subject: Re: noise problem on radio
From: Sacha <uuccio(at)gmail.com>
Date: Apr 09, 2014
I'm back trying to look for a solution to my noise problem on the radio. The original thread was started in Feb so for those who missed it, the installation is a Rotax 912UL on a Kitfox with a Trio Avionics Propilot Autopilot, a GNS430 and a GMA240 audio panel. The mag leads are unshielded. I think is a NEW problem, as the first few hours of flying this installation did not seem to present this problem. I say I think because I'm not 100% sure here because I don't think I turned the a/p on during the first few flights (it was a year ago). The noise appears in the form of an RX signal in the GNS430, and is correlated to two things: - the a/p box being on (not necessarily the servos though) - the engine RPM being above 3800 RPM Specifically, when the a/p is off, I never hear any noise. And when the a/p is on and the RPM is below 3500 (e.g. in a descent) the noise can be squelched off by the audio panel (though the radio still shows an RX signal). But if I'm in a climb and have the a/p on then the only way I can get rid of the noise is by turning the volume of the audio panel right down (the volume of the GNS430 radio does not affect the loudness of the noise). I have tried to turn up the squelch on the GNS430 but I'm not sure I'm doing it right. I press the volume button once and then turn it, but it doesn't appear that the pressing the button makes any difference. Am I doing something wrong there? Below are the answers to some of Bob N's previous suggestions. > On 27 Feb 2014, at 15:15, "Robert L. Nuckolls, III" wrote: > > > > No, I haven't. Is it worth getting my hands on an SWR meter? > > If your shield ground is loose at the radio, it can open a > pathway for conducting otherwise insignificant > signals into the receiver. > > If you don't have ready access to one, hold off acquiring > it. But check the mechanical integrity of your coax > connections at the back of the radio. How well does the > comm transceiver work? Have you detected any shortfall > in performance? > The coax connections look ok mechanically. And the com transceiver appears to work fine, although I haven't tried it over longer distances. > > http://www.trioavionics.com/Pro%20Pilot%20Manual%203.8.pdf > > Okay . . . that's a busy little box. Refresh my > memory, is this noise a new thing or has it always > been present. Also, when receiving a weak signal > (tune in an ATIS and fly away from airport until > signal starts to get noisy . . . then turn a/p > on/off and judge how much effect the a/p noise > has on reception of weak signals). I'll try that next time I can get an ATIS (I am 60 miles from the nearest ATIS and not in theory allowed to fly more than 500 feet above the nearest obstacle in a 3 mile radius, so it's quite difficult to get reception). > > I sniffed around yesterday with the handheld and its regular antenna. What > I noticed was that as soon as I switch on the Avionics and the handheld > antenna is close to them, I hear a bunch of noise, but I guess that's > normal. When I switch on the AP and move the handheld antenna close to the > servo cables (that are shielded), I can hear a stronger kind of buzzing > noise. I really have to be almost touching those shielded cables though in > order to pick anything up. I didn't try putting the antenna close to the > radio's COM coax connector though. > > You wont see any noise coming OUT of this junction, > but it is a potential point of ingress for noises > that tend to pile up behind the panel. > > Do you have any pictures of how to build these? Is this > http://www.emcesd.com/tt120100.htm the right idea? > > Yes! good find. I got a little ahead of myself > in sifting the simple ideas. Don't run off and build > one (or e-field probe) yet. > > It's not clear to me yet as to operational significance > of the noise. DO-160 ALLOWS certain levels of noise > while putting potential victims ON NOTICE that such > noises may be present but normally insignificant. > > My sense of the situation from your narrative > so far is that while the A/P is a noteworthy > contributor . . . it's not the sole potential > antagonist . . . we may discover that it is > within practical limits and you just need to > tighten the squelch on the receiver a tad. I tried (see above) but not sure I'm doing it right. > > I think I've related my experiences with the > symphony of noises that are often heard in > various systems while sitting on the ground > with engines off and wearing headphones . . . > noises that are completely insignificant > while in flight. > > Let's size the task before we get out hammers- > n-saws. > > Bob . . . > Sacha ________________________________________________________________________________
Date: Apr 09, 2014
From: Thomas Blejwas <tomblejwas(at)yahoo.com>
Subject: Re: When are diodes needed?
Thanks Joe,=0A=0AI now understand the potential problem with feeding throug h small wires and fuses.- Embarrassed that I missed it. -"Smoothing" wa s not really what I should have said.- I have been told that, when two su pplies-through diodes are connected to a-component, the diodes ensure t hat the one with the higher voltage supplies the current.- As the voltage in-that one drops, the transition to the other is seamless.- True?- -Without the diodes, I have two batteries in parallel,-which yields an average voltage, assuming similar internal resistances.- Not clear to me the advantage of one over the other.- =0A=0AThanks again for your help. -=0A=0ATom=0A=0A=0A=0A-=0A=0A=0A>________________________________=0A> F rom: user9253 =0A>To: aeroelectric-list(at)matronics.com =0A>Sent: Wednesday, April 9, 2014 8:54 AM=0A>Subject: AeroElectric-List: R e: When are diodes needed?=0A> =0A>=0A>--> AeroElectric-List message poste d by: "user9253" =0A>=0A>It is unlikely that a battery w ill be bad enough that the other "good" battery would feed it.- A more im portant reason for isolating the two fuel pump circuits in Z-19 is to preve nt alternator charging current from back-feeding through small wires and fu ses intended to carry only the fuel pump load.=0A>There is more than one ve rsion of Z-07.- Looking at revision 5, the diode in the "Engine B" wire p revents engine starting current and main bus load current from flowing thro ugh the "Engine B" wire from the Aux battery.=0A>- I do not know what you mean by diodes "smoothing" the feed.=0A>Joe=0A>=0A>--------=0A>Joe Gores =0A>=0A>=0A>=0A>=0A>Read this topic online here:=0A>=0A>http://forums.matro =========================0A =========================0A =========================0A >=0A>=0A>=0A>=0A>=0A> ________________________________________________________________________________
Date: Apr 09, 2014
Subject: Re: Z13/8 Main Alternator.
From: Bill Allen <billallensworld(at)gmail.com>
Hi Mike, So you're nearly finished? Will you be flying this year? If you come past in May/June Gary Hertzler will be here again for more Ez talk :^) Bill On 9 April 2014 15:44, MikeDunlop wrote: > > I'm finishing off my electrical system in a Long-EZ that includes the > SD-8, exactly as per the Z13/8 and have to make a final choice of the main > alternator. > > For cost and availability locally I've decided to go with either a Lucas > 60amp or 70amp. The advice I'm seeking is regarding the regulator, these > units have an internal regulator, so do I keep the internal regulator? > (remember the Z13/8 has the OV protection between the regulator and the > bus) or do I do a conversion to use a B&C external regulator (I have a > spare one I could use). > > I've read many, many threads on OV and runaway alternators etc. and am not > too sure if using the internal regulator in the Z13/8 architecture will > give me complete protection. > > Regards > > Mike Dunlop (UK) > > > Read this topic online here: > > http://forums.matronics.com/viewtopic.php?p=421821#421821 > > ________________________________________________________________________________
Subject: Re: When are diodes needed?
From: "user9253" <fransew(at)gmail.com>
Date: Apr 09, 2014
It is true that a load will draw current from whichever source has the higher voltage. Joe -------- Joe Gores Read this topic online here: http://forums.matronics.com/viewtopic.php?p=421830#421830 ________________________________________________________________________________
Date: Apr 09, 2014
Subject: EI MVP-50 on E Bay
From: John McMahon <blackoaks(at)gmail.com>
Just saw this for sale new & unopened unit on E Bay under Avionics and it is closing soon; if interested the link is: http://www.ebay.com/itm/141240683303?ssPageName=STRK:MESELX:IT&_trksid=p3984.m1555.l2649 EI MVP-50 6 Cylinder Experimental Aircraft Engine Monitor -- John McMahon ________________________________________________________________________________
Date: Apr 09, 2014
From: "Robert L. Nuckolls, III" <nuckolls.bob(at)aeroelectric.com>
Subject: Re: Z13/8 Main Alternator.
At 09:44 AM 4/9/2014, you wrote: > >I'm finishing off my electrical system in a Long-EZ that includes >the SD-8, exactly as per the Z13/8 and have to make a final choice >of the main alternator. > >For cost and availability locally I've decided to go with either a >Lucas 60amp or 70amp. The advice I'm seeking is regarding the >regulator, these units have an internal regulator, so do I keep the >internal regulator? (remember the Z13/8 has the OV protection >between the regulator and the bus) or do I do a conversion to use a >B&C external regulator (I have a spare one I could use). > >I've read many, many threads on OV and runaway alternators etc. and >am not too sure if using the internal regulator in the Z13/8 >architecture will give me complete protection. Alternators with built in regulators . . . as a GENERAL rule, DO NOT feature ov protection consistent with legacy design goals. One exception I'm aware of is PlanePower who appears to be using a customized built-in regulator with crowbar ov protection and positive crew-controlled power to the field. At the time of this writing, it's the ONLY alternator offering that conforms to the legacy alternator control/ ov protection philosophy. I'm not sure if I understand your question . . . The SD-8 should be installed as shown in the drawing. Further, its ov protection, control and regulation is entirely independent of any choices you make for a main alternator. If you're contemplating an over-the-counter automotive alternator with built in regulator . . . it will PROBABLY perform as advertised. But it will not conform to the design goals adopted by all TC aircraft and the majority of OBAM aircraft. If you have a B&C alternator controller laying around, it's generally not difficult to modify any stock automotive machine to accept EXTERNAL power, regulation, and protection. If you're not up to the task of doing the mod, a local alternator repair shop may be willing/able to do it for you. Bob . . . ________________________________________________________________________________
Date: Apr 09, 2014
From: Jeff Luckey <jluckey(at)pacbell.net>
Subject: Re: When are diodes needed?
Joe,=0ANot sure how to answer without a little more context.- Can you giv e us a sketch of the circuit in question?=0A=0AIn the mean time, maybe this will help:=0A=0ACrude diode symbol: - ->|=0A=0A=0A=0ACurrent will flow t hru the diode (left to right) in the following circuit:=0A=0A5 volts -- --- --- - 4 volts=0A=0A---------- ->| ------------=0A=0A=0AC urrent will NOT flow backward (right to left) thru the diode in the followi ng circuit:=0A=0A4 volts -- --- --- - 5 volts=0A=0A------ ---- ->| ------------=0A=0AThe diode acts as a "check-valve" and prevents c urrent from flowing backwards=0A=0A=0AHope this helps=0A=0A-Jeff=0A=0A=0A__ ______________________________=0A From: user9253 <fransew(at)gmail.com>=0ATo: aeroelectric-list(at)matronics.com =0ASent: Wednesday, April 9, 2014 11:18 AM =0ASubject: AeroElectric-List: Re: When are diodes needed?=0A =0A=0A--> Aer oElectric-List message posted by: "user9253" =0A=0AIt is true that a load will draw current from whichever source has the higher vo ltage.=0AJoe=0A=0A--------=0AJoe Gores=0A=0A=0A=0A=0ARead this topic online here:=0A=0Ahttp://forums.matronics.com/viewtopic.php?p=421830#421830=0A = ________________________________________________________________________________
Subject: Re: Z13/8 Main Alternator.
From: "MikeDunlop" <mdunlop001(at)aol.com>
Date: Apr 09, 2014
Bill, Nice to hear from you again, I've called by your hangar a few times but missed you. Yes I'm hoping to get things wrapped up soon and start the testing phase. I will definitely like to meet Gary again and talk EZ, I picked up some very useful information last time that made me change the cowling air intake design and the prop attachment method. Regards Mike Read this topic online here: http://forums.matronics.com/viewtopic.php?p=421841#421841 ________________________________________________________________________________
Subject: Re: Z13/8 Main Alternator.
From: "MikeDunlop" <mdunlop001(at)aol.com>
Date: Apr 09, 2014
Bob, Thanks very much for the information. I will modify the alternator and use the B&C regulator. The SD-8 is installed exactly as per Z13/8, totally independent of the main alternator. Regards Mike (UK) Read this topic online here: http://forums.matronics.com/viewtopic.php?p=421843#421843 ________________________________________________________________________________
From: Bob McCallum <robert.mccallum2(at)sympatico.ca>
Subject: Re: When are diodes needed?
Date: Apr 09, 2014
Yes. All else being equal if you connect both a 12V and a 6V source in parallel to a single load without any other intervening components then the 12V sour ce will supply the load and the 6V source will sit idly by doing nothing. Bob McC > Subject: AeroElectric-List: Re: When are diodes needed? > From: fransew(at)gmail.com > Date: Wed=2C 9 Apr 2014 11:18:58 -0700 > To: aeroelectric-list(at)matronics.com > > > It is true that a load will draw current from whichever source has the hi gher voltage. > Joe > > -------- > Joe Gores > > > > > Read this topic online here: > > http://forums.matronics.com/viewtopic.php?p=421830#421830 > > > > > > > =========== =========== =========== =========== > > > ________________________________________________________________________________
Date: Apr 10, 2014
Subject: Re: When are diodes needed?
From: Bob Verwey <bob.verwey(at)gmail.com>
Is there a practical application for this? On 09/04/2014, Bob McCallum wrote: > Yes. > > All else being equal if you connect both a 12V and a 6V source in parallel > to a single load without any other intervening components then the 12V > source will supply the load and the 6V source will sit idly by doing > nothing. > > Bob McC > >> Subject: AeroElectric-List: Re: When are diodes needed? >> From: fransew(at)gmail.com >> Date: Wed, 9 Apr 2014 11:18:58 -0700 >> To: aeroelectric-list(at)matronics.com >> >> >> It is true that a load will draw current from whichever source has the >> higher voltage. >> Joe >> >> -------- >> Joe Gores >> >> >> >> >> Read this topic online here: >> >> http://forums.matronics.com/viewtopic.php?p=421830#421830 >> >> >> >> >> >> >> > =========== > =========== > =========== > =========== >> >> >> > -- Best... Bob Verwey ________________________________________________________________________________
From: "William Schertz" <wschertz(at)comcast.net>
Subject: Re: When are diodes needed?
Date: Apr 10, 2014
I believe that the 12V source will try to charge the 6V source, overcharging the 6V battery if a diode is not present. From: Bob McCallum Sent: Wednesday, April 09, 2014 3:37 PM Subject: RE: AeroElectric-List: Re: When are diodes needed? Yes. All else being equal if you connect both a 12V and a 6V source in parallel to a single load without any other intervening components then the 12V source will supply the load and the 6V source will sit idly by doing nothing. Bob McC > Subject: AeroElectric-List: Re: When are diodes needed? > From: fransew(at)gmail.com > Date: Wed, 9 Apr 2014 11:18:58 -0700 > To: aeroelectric-list(at)matronics.com > > > It is true that a load will draw current from whichever source has the higher voltage. > Joe > > -------- > Joe Gores > > > > > Read this topic online here: > > http://forums.matronics.com/viewtopic.php?p=421830#421830 > > > > Archive Search & Download, 7-Day Browse, Chat, FAQ, > --> http://www.matronics.com/Navigator?AeroElectric-List > ======================= > > > ________________________________________________________________________________
From: Bob McCallum <robert.mccallum2(at)sympatico.ca>
Subject: Re: When are diodes needed?
Date: Apr 10, 2014
Yes absolutely ! (assuming the sources are batteries. If they are diode iso lated power supplies (alternators) the original statement stands) I wasn't suggesting that this was a practical thing to do. Mearly that the higher voltage source will supply the load. Normally we would be talking pa ralleled batteries or a battery and an alternator. In either case the highe r voltage source will supply the load. All else being equal=2C the lower vo ltage source will be idle until the system voltage sags to equal the lower. (even this is an oversimplification=2C but hopefully gets the idea across. The higher of two paralleled differing voltages will be supplying the load s=2C including attempting to raise the lower voltage source. That's the bas is of how a charging system works) Bob McC From: wschertz(at)comcast.net Subject: Re: AeroElectric-List: Re: When are diodes needed? Date: Thu=2C 10 Apr 2014 07:12:02 -0500 =0A =0A =0A =0A =0A =0A I believe that the 12V source will try to charge the 6V source=2C =0A overcharging the 6V battery if a diode is not present.=0A =0A =0A =0A =0A From: Bob McCallum =0A Sent: Wednesday=2C April 09=2C 2014 3:37 PM=0A =0A Subject: RE: AeroElectric-List: Re: When are diodes =0A needed?=0A =0A =0A Yes. All else being =0A equal if you connect both a 12V and a 6V source in parallel to a =0A single load without any other intervening components then the 12V source wi ll =0A supply the load and the 6V source will sit idly by doing nothing. Bob =0A McC =0A > Subject: AeroElectric-List: Re: When are diodes needed? > From: =0A fransew(at)gmail.com > Date: Wed=2C 9 Apr 2014 11:18:58 -0700 > To: =0A aeroelectric-list(at)matronics.com > > --> AeroElectric-List =0A message posted by: "user9253" > > It is =0A true that a load will draw current from whichever source has the higher =0A voltage. > Joe > > -------- > Joe Gores > =0A > > > > Read this topic online here: > =0A > http://forums.matronics.com/viewtopic.php?p=421830#421830 > =0A > > > Archive Search & Download=2C 7-Day Browse=2C Chat=2C =0A FAQ=2C > --> http://www.matronics.com/Navigator?AeroElectric-List > =0A ======================= > > > =0A =0A href="http://www.matronics.com/Navigator?AeroElectric-List">http://www.ma tronics.com/Navigator?AeroElectric-List=0A href="http://forums.matronics.com">http://forums.matronics.com=0A href="http://www.matronics.com/contribution">http://www.matronics.com/c =0A =0A =0A =0A =0A =0A ============0A ============0A ============0A ============0A =0A ________________________________________________________________________________
Date: Apr 10, 2014
Subject: Re: Z13/8 Main Alternator.
From: Bill Allen <billallensworld(at)gmail.com>
Yes, I was away Jan/Feb. I'll be back here much more after Easter as Gary will be here then, so May/June would be a good time - call or text me? Bill On 9 April 2014 21:22, MikeDunlop wrote: > > Bill, > > Nice to hear from you again, I've called by your hangar a few times but > missed you. > > Yes I'm hoping to get things wrapped up soon and start the testing phase. > > I will definitely like to meet Gary again and talk EZ, I picked up some > very useful information last time that made me change the cowling air > intake design and the prop attachment method. > > Regards > > Mike > > > Read this topic online here: > > http://forums.matronics.com/viewtopic.php?p=421841#421841 > > ________________________________________________________________________________
Subject: Re: EarthX lithium
From: "user9253" <fransew(at)gmail.com>
Date: Apr 11, 2014
There was a recent comparison of a LiFePo battery versus Lead Acid and the test results posted in a graph on VansAirforce. http://www.vansairforce.com/community/showthread.php?t=111788 It seems that the actual Amp Hour rating of the LiFePo is only half of the advertised rating. Even so, a 36 amp hour LiFePo provides almost the same endurance as a PC680 at less than half of the weight. The author intends to install the LiFePo in his RV-8A. It will be interesting to see if he is satisfied with this battery a few years from now. Joe -------- Joe Gores Read this topic online here: http://forums.matronics.com/viewtopic.php?p=421907#421907 ________________________________________________________________________________
Date: Apr 11, 2014
From: "Robert L. Nuckolls, III" <nuckolls.bob(at)aeroelectric.com>
Subject: Re: EarthX lithium
At 06:45 AM 4/11/2014, you wrote: > >There was a recent comparison of a LiFePo battery versus Lead Acid >and the test results posted in a graph on VansAirforce. >http://www.vansairforce.com/community/showthread.php?t=111788 >It seems that the actual Amp Hour rating of the LiFePo is only half >of the advertised rating. Even so, a 36 amp hour LiFePo provides >almost the same endurance as a PC680 at less than half of the >weight. The author intends to install the LiFePo in his RV-8A. It >will be interesting to see if he is satisfied with this battery a >few years from now. I missed getting the 4th slot filled on time in the series of articles on batteries for Kitplanes. Spent an extra month attempting to engage a number of suppliers/manufacturers in technical conversations about their products. Only one has been eagerly forthcoming (EarthX) and one other begrudgingly offered tid-bits of the requested data. I've offered to work with EarthX to wordsmith specification and sales literature that would get them more than a casual glance and fewer snickers from a TC airframer. Part IV of the battery articles has been submitted to Paul Dye. I've discovered a lot about the current state of LiPo offerings but my inquiries have generated more questions than answers! As the thread cited above demonstrates, there's a good bit yet to be learned about the lithium battery industry. Encourage folks on the VansAirForce-list to join us here on the AeroElectric- List as well. By the way, that PbEq (lead-acid equivalency) thingy is pure marketing hype. It speaks to battery impedance and ability to crank engines . . . says NOTHING about energy available for battery-only ops. Bob . . . ________________________________________________________________________________
Date: Apr 15, 2014
From: "Robert L. Nuckolls, III" <nuckolls.bob(at)aeroelectric.com>
Subject: Schumacher XC75W charger
I've been doing some tests on a 100Ah rated (20 hr rate) battery that uses carbon foam for one of the grids. Ostensibly lighter that a battery with both grids of lead. http://tinyurl.com/q5hu266 Needed a robust charger for getting this puppy re-charged. At the same time, I was interested in evaluating one of Schumacher's current offerings in programmed chargers. Bought a XC75W with nice buttons, annunciators and digital display. Max charge rate was advertised as 20A. The first time I plotted a recharge in the "AGM" mode, the charger wanted to top-off charge at well over 15 volts. It had barely leveled off at 15.7V when the microprocessor deemed the battery charged! The battery wheezed and whistled for a couple hours after the charger shut down. Emacs! On the next cycle, I used the "GEL" mode. This mode tops off at 14.4V . . . can't speak to ideal protocols for GEL batteries but this looks like a good one for AGM as well. It's interesting that it spent about as much time in top-off as it did for charging. Emacs! Will learn more about this charger as testing progresses on the battery. I may see if Schumacher's engineering staff will comment on these findings . . . but in any case, AGM mode in this charger is not recommended for any battery. Watch this space . . . Bob . . . ________________________________________________________________________________
Date: Apr 16, 2014
From: Matt Dralle <dralle(at)matronics.com>
Subject: Matronics Web Server Down for Repair (PLEASE READ)...
Dear Listers, Late Tuesday night 4/15/2014, the Matronics Web server crashed due to a multi-disk RAID 5 failure. The system has complete data backups, but I will likely have to order some replacement disks to rebuild the system. I'll work as quickly as I can to restore the Matronics Web Server. The Matronics EMAIL server is NOT impacted by this issue. All normal Matronics Email List mail will flow as usual. However, the Matronics Web Forums interface will not be available. Also impacted by the Web Server outage will be the AeroElectric web site. I will try to post daily updates on the Web Server restoration status. I apologize for the inconvenience... :-( Best regards, Matt Dralle Matronics Email List and Forum Admin. Matt G Dralle | Matronics | 581 Jeannie Way | Livermore | CA | 94550 925-606-1001 V | 925-606-6281 F | dralle(at)matronics.com Email http://www.matronics.com/ WWW | Featuring Products For Aircraft ________________________________________________________________________________
Date: Apr 17, 2014
Subject: Re: AeroElectric-List Digest: 1 Msgs - 04/16/14
From: Eric Schlanser <eschlanser(at)yahoo.com>
Not sure if this will go through what with the server outage and all, but today I am integrating a diode into the aux alternator power 22 awg wire and I need Bob's comic book on integrating homeless components. If someone hears this, please respond. Hope this works. Eric AeroElectric-List Digest Server wrote: >* > > ================================================= > Online Versions of Today's List Digest Archive > ================================================= > >Today's complete AeroElectric-List Digest can also be found in either of the >two Web Links listed below. The .html file includes the Digest formatted >in HTML for viewing with a web browser and features Hyperlinked Indexes >and Message Navigation. The .txt file includes the plain ASCII version >of the AeroElectric-List Digest and can be viewed with a generic text editor >such as Notepad or with a web browser. > >HTML Version: > > http://www.matronics.com/digest/digestview.php?Style=82701&View=html&Chapter 14-04-16&Archive=AeroElectric > >Text Version: > > http://www.matronics.com/digest/digestview.php?Style=82701&View=txt&Chapter 14-04-16&Archive=AeroElectric > > > =============================================== > EMail Version of Today's List Digest Archive > =============================================== > > > ---------------------------------------------------------- > AeroElectric-List Digest Archive > --- > Total Messages Posted Wed 04/16/14: 1 > ---------------------------------------------------------- > > >Today's Message Index: >---------------------- > > 1. 01:02 AM - Matronics Web Server Down for Repair (PLEASE READ)... (Matt Dralle) > > > >________________________________ Message 1 _____________________________________ > > >From: Matt Dralle <dralle(at)matronics.com> >Subject: AeroElectric-List: Matronics Web Server Down for Repair (PLEASE READ)... > > >Dear Listers, > >Late Tuesday night 4/15/2014, the Matronics Web server crashed due to a multi-disk >RAID 5 failure. The system has complete data backups, but I will likely have >to order some replacement disks to rebuild the system. I'll work as quickly >as I can to restore the Matronics Web Server. > >The Matronics EMAIL server is NOT impacted by this issue. All normal Matronics >Email List mail will flow as usual. However, the Matronics Web Forums interface >will not be available. > >Also impacted by the Web Server outage will be the AeroElectric web site. > >I will try to post daily updates on the Web Server restoration status. > >I apologize for the inconvenience... :-( > >Best regards, > >Matt Dralle >Matronics Email List and Forum Admin. > > >Matt G Dralle | Matronics | 581 Jeannie Way | Livermore | CA | 94550 >925-606-1001 V | 925-606-6281 F | dralle(at)matronics.com Email >http://www.matronics.com/ WWW | Featuring Products For Aircraft > > > > > > > > > ________________________________________________________________________________
Date: Apr 17, 2014
From: "Robert L. Nuckolls, III" <nuckolls.bob(at)aeroelectric.com>
Subject: Re: AeroElectric-List Digest: 1 Msgs - 04/16/14
At 09:06 AM 4/17/2014, you wrote: > >Not sure if this will go through what with the server outage and >all, but today I am integrating a diode into the aux alternator >power 22 awg wire and I need Bob's comic book on integrating >homeless components. If someone hears this, please respond. >Hope this works. The list's are supported by separate hardware that deals with e-mail so your posting is not going into a black hole . . . What kind of diode and where are you wanting to wire it in and what purpose does it serve? If it's a little plastic or glass wire lead diode then you can trim off the diode leads to about 3/8", lap solder the wires to each end and cover with a couple layers of heatshrink. See attached . . . Bob . . . ________________________________________________________________________________
From: "Sacha" <uuccio(at)gmail.com>
Subject: AEC9001-1 Schottky Diode Assembly Instructions
Date: Apr 17, 2014
Along the same lines as Eric... Could someone kindly post the assembly instructions for the AEC9001-1 Schottky Diode? Regards, Sacha ________________________________________________________________________________
Date: Apr 17, 2014
From: Eric Schlanser <eschlanser(at)yahoo.com>
Subject: Re: AeroElectric-List Digest: 1 Msgs - 04/16/14
To the AE_List\Bob,=0A=0AI found the comic book on integrating homeless com ponents.-=0A=0AIn appreciation,=0A=0AEric=0A=0A=0A=0A____________________ ____________=0A From: Eric Schlanser <eschlanser(at)yahoo.com>=0ATo: aeroelect ric-list(at)matronics.com =0ASent: Thursday, April 17, 2014 10:06 AM=0ASubject : Re: AeroElectric-List Digest: 1 Msgs - 04/16/14=0A =0A=0ANot sure if this will go through what with the server outage and all, but today I am integr ating a diode into the aux alternator power 22 awg wire and I need Bob's co mic book on integrating homeless components. If someone hears this, please respond.=0AHope this works.=0AEric =0A=0A=0A=0AAeroElectric-List Digest Ser ver wrote:=0A=0A=0A*=0A=0A===== =====================0A- Online Versions of Today's List Digest Archive=0A=========== ===============0A=0AToday's complete AeroElectr ic-List Digest can also be found in either of the =0Atwo Web Links listed b elow.- The .html file includes the Digest formatted =0Ain HTML for viewin g with a web browser and features Hyperlinked Indexes =0Aand Message Naviga tion.- The .txt file includes the plain ASCII version =0Aof the AeroElect ric-List Digest and can be viewed with a generic text editor =0Asuch as Not epad or with a web browser. =0A=0AHTML Version:=0A=0A- - http://www.mat ronics.com/digest/digestview.php?Style=82701&View=html&Chapter 14-0 4-16&Archive=AeroElectric=0A=0AText Version:=0A=0A- - http://www.matr onics.com/digest/digestview.php?Style=82701&View=txt&Chapter 14-04- 16&Archive=AeroElectric=0A=0A=0A============= ===========0A- EMail Version of Today's List Digest Archive=0A===================== ===0A=0A=0A- - - - - --------------------------------------- -------------------=0A- - - - - - - - - - - - - AeroElectric-List Digest Archive=0A- - - - - - - - - - - - - - - - - - - ---=0A- - - - - - - - - - Total Messages Posted Wed 04/16/14: 1=0A- - - - - ----- -----------------------------------------------------=0A=0A=0AToday's Messa ge Index:=0A----------------------=0A=0A- - 1. 01:02 AM - Matronics We b Server Down for Repair (PLEASE READ)...- (Matt Dralle)=0A=0A=0A=0A_____ ___________________________- Message 1- _______________________________ cs.com>=0ASubject: AeroElectric-List: Matronics Web Server Down for Repair (PLEASE READ)...=0A=0A=0ADear Listers,=0A=0ALate Tuesday night 4/15/2014, t he Matronics Web server crashed due to a multi-disk=0ARAID 5 failure.- Th e system has complete data backups, but I will likely have=0Ato order some replacement disks to rebuild the system.- I'll work as quickly=0Aas I can to restore the Matronics Web Server.- =0A=0AThe Matronics EMAIL server i s NOT impacted by this issue.- All normal Matronics=0AEmail List mail wil l flow as usual.- However, the Matronics Web Forums interface=0Awill not be available.=0A=0AAlso impacted by the Web Server outage will be the AeroE lectric web site.=0A=0AI will try to post daily updates on the Web Server r estoration status.=0A=0AI apologize for the inconvenience...- :-(=0A=0ABe st regards,=0A=0AMatt Dralle=0AMatronics Email List and Forum Admin.=0A=0A =0AMatt G Dralle | Matronics | 581 Jeannie Way | Livermore | CA | 94550=0A9 25-606-1001 V | 925-606-6281 F | dralle@matronics.com Email=0Ahttp://www.ma =============== ________________________________________________________________________________
Date: Apr 17, 2014
From: "Robert L. Nuckolls, III" <nuckolls.bob(at)aeroelectric.com>
Subject: Re: AEC9001-1 Schottky Diode Assembly Instructions
At 10:38 AM 4/17/2014, you wrote: > >Along the same lines as Eric... >Could someone kindly post the assembly instructions for the AEC9001-1 >Schottky Diode? Here 'tis . . . Bob . . . ________________________________________________________________________________
Date: Apr 17, 2014
From: Matt Dralle <dralle(at)matronics.com>
Subject: Matronics Web Server Repaired and ONLINE! (PLEASE READ)
Dear Listers, The new hard drives for the Matronics web server arrived today, Thursday 4/17/2014 and the RAID5 disk array rebuild went very smoothly. Web server is back online and fully functional! No data or files were lost. Additionally, those that frequent the AeroElectric web site will find that it has been restored as well. Best regards, Matt Dralle Matronics Email List and Forum Admin. Matt G Dralle | Matronics | 581 Jeannie Way | Livermore | CA | 94550 925-606-1001 V | 925-606-6281 F | dralle(at)matronics.com Email http://www.matronics.com/ WWW | Featuring Products For Aircraft ________________________________________________________________________________
Subject: Re: AEC9001-1 Schottky Diode Assembly Instructions
From: "Eric M. Jones" <emjones(at)charter.net>
Date: Apr 18, 2014
Please let me Google that for you: https://www.google.com/search?btnG=1&pws=0&q=AEC9001-1+Aeroelectric+Connection+ -------- Eric M. Jones www.PerihelionDesign.com 113 Brentwood Drive Southbridge, MA 01550 (508) 764-2072 emjones(at)charter.net Read this topic online here: http://forums.matronics.com/viewtopic.php?p=422054#422054 ________________________________________________________________________________
Subject: Re: AEC9001-1 Schottky Diode Assembly Instructions
From: Sacha <uuccio(at)gmail.com>
Date: Apr 18, 2014
Thank you Bob. Eric, I tried Google but the matronics site was down and (hard to believe but...) that's the only place that particular document showed up in the Google search. > On Apr 18, 2014, at 14:56, "Eric M. Jones" wrote: > > > Please let me Google that for you: > > https://www.google.com/search?btnG=1&pws=0&q=AEC9001-1+Aeroelectric+Connection+ > > -------- > Eric M. Jones > www.PerihelionDesign.com > 113 Brentwood Drive > Southbridge, MA 01550 > (508) 764-2072 > emjones(at)charter.net > > > > > Read this topic online here: > > http://forums.matronics.com/viewtopic.php?p=422054#422054 > > > > > > > > > > ________________________________________________________________________________
Subject: Alternator selection for Rotax (a little off topic)
From: "rparigoris" <rparigor(at)suffolk.lib.ny.us>
Date: Apr 19, 2014
Hi Group A little off topic so delete if you like. I would like to install a more powerful alternator on my Rotax 914. I have already selected a circuit breaker for the over-voltage crow bar, I was wondering if anyone could recommend an alternator I could use that wouldn't exceed the circuit breakers voltage and current rating? : https://onedrive.live.com/redir?resid=550FC20DBDDB521D!3283&authkey=!AOJ6Dc5VV0v8uVo&v=3&ithint=photo%2c.JPG Ron Parigoris Sorry, I couldn't post this 18 days ago. Insulators were (my best guess) more than 25 feet long, the main casting (gutted) was a mere 30 tons! Read this topic online here: http://forums.matronics.com/viewtopic.php?p=422116#422116 ________________________________________________________________________________
Date: Apr 21, 2014
From: rayj <raymondj(at)frontiernet.net>
Subject: Half failed LEDs
I just removed about 8 blue LEDs from a string of lights that no longer emitted light, but still have continuity. If any one wants them to investigate, I'll mail them out. -- Raymond Julian Kettle River, MN The things we admire in men, kindness and generosity, openness, honesty, understanding and feeling are the concomitants of failure in our system. And those traits we detest, sharpness, greed, acquisitiveness, meanness, egotism and self-interest are the traits of success. And while men admire the quality of the first they love the produce of the second. -John Steinbeck, novelist, Nobel laureate (1902-1968) ________________________________________________________________________________
From: "Sacha" <uuccio(at)gmail.com>
Subject: warning lights for engine T&Ps
Date: Apr 21, 2014
Aero-Electrics, Is there an easy/robust way of incorporating "idiot" warning lights for the engine T&Ps? When I first designed the panel I had in mind tri-colour LEDs which would turn blue if a value was below normal, green if in the normal range and red if above the normal range (e.g. for CHT temps or fuel pressure). I experimented with this circuit https://www.spiyda.com/magento/index.php/three-colour-warning-light.html which I used to measure the various senders resistances but was not able to reliably calibrate the readings to get anything useful. I am determined to do some bench tests to see whether I can make things work better by incorporating a 12V voltage regulator upstream of the circuit to get more repeatable readings. Before I embark on this though I thought I'd check if there is already an established way of doing this? Sacha ________________________________________________________________________________
Subject: Re: warning lights for engine T&Ps
From: "Eric M. Jones" <emjones(at)charter.net>
Date: Apr 22, 2014
> Before I embark on this though I thought I'd > check if there is already an established way of doing this? > Sacha Sacha, There is a common op amp circuit called a "window discriminator" that is used to do this. It is easy to set this up for many variable input voltages or currents. National Semiconductor publishes online collections of op amp circuits which is a good place to start. ps When red and green are mixed, yellow is the visible result...although some color deficient people might have an issue with this. -------- Eric M. Jones www.PerihelionDesign.com 113 Brentwood Drive Southbridge, MA 01550 (508) 764-2072 emjones(at)charter.net Read this topic online here: http://forums.matronics.com/viewtopic.php?p=422234#422234 ________________________________________________________________________________
Date: Apr 22, 2014
From: "Robert L. Nuckolls, III" <nuckolls.bob(at)aeroelectric.com>
Subject: Re: Half failed LEDs
At 07:09 AM 4/21/2014, you wrote: > >I just removed about 8 blue LEDs from a string of lights that no >longer emitted light, but still have continuity. If any one wants >them to investigate, I'll mail them out. When you measure them with an ohmmeter, do they show the same or similar resistance for either polarity of the test leads? Bob . . . ________________________________________________________________________________
Date: Apr 22, 2014
From: "Robert L. Nuckolls, III" <nuckolls.bob(at)aeroelectric.com>
Subject: Re: warning lights for engine T&Ps
>I am determined to do some bench tests to see whether I can make things work >better by incorporating a 12V voltage regulator upstream of the circuit to >get more repeatable readings. Before I embark on this though I thought I'd >check if there is already an established way of doing this? >Sacha Good for you! Suggest you start by quantifying the sensors. Most automotive sensors are of the variable resistance type intended to drive fairly low-sensitivity instruments . . . read operating currents in the tens of milliamps. My favorite approach is to first bias up such sensors with a constant current source (LM317 wired like the Lo Ohms adapters described on my website). Select a bias current that is as large as practical (means output delta-V in relation to delta-R is greatest). Using constant current excitation eliminate variability of readings due to variations in bus voltage. Then get yourself some rough calibration curves for each style of sensor. Water bath on stove works for temp sensors, air tank plumbed to pressure sensor with some needle valves in the pressure/bleed controls is handy. Once you have ball-parked the voltages of interest, you're ready to tackle the detection/ lamp drivers. The device you linked seems like it should do the job. You may have a wiring error . . . or perhaps are wrestling with outside influences for not having stable excitation sources for the sensors. Another approach to consider is using a micro-controller like the PIC12F683 . . . a $1.00 device that will resolve a voltage, deduce significance of its magnitude and then do the appropriate hat dance to drive the LED. The cool thing is that you can put calibration variables into separate, easily modified memory locations such that no potentiometers are needed. Takes a little more head-work to calibrate the critter but once it's set, you're all done and it won't drift. The task you've described can probably be done with no more than a single chip. There are talented, experienced folks here on the List that can help with your software education. I probably have some etched circuit boards laying around that are 95% of what you need . . . already in existence along with enclosures like . . . http://www.aeroelectric.com/Catalog/AEC/9024/9024_Four_Fuction_ Perhaps this is more new te4rritory than you wanted to explore but getting a couple of these things up and running to your task would greatly expand your horizons. Bob . . . ________________________________________________________________________________
Date: Apr 22, 2014
From: rayj <raymondj(at)frontiernet.net>
Subject: Re: Half failed LEDs
Most of them were in the 4-8k range both ways. A couple were in the 90k range both ways, and one was about 90k one way and about 150k the other. They were in series with other LEDs and all carried enough current to allow the other LEDs to continue to light. Raymond Julian Kettle River, MN The things we admire in men, kindness and generosity, openness, honesty, understanding and feeling are the concomitants of failure in our system. And those traits we detest, sharpness, greed, acquisitiveness, meanness, egotism and self-interest are the traits of success. And while men admire the quality of the first they love the produce of the second. -John Steinbeck, novelist, Nobel laureate (1902-1968) On 04/22/2014 08:18 AM, Robert L. Nuckolls, III wrote: > > > At 07:09 AM 4/21/2014, you wrote: >> >> I just removed about 8 blue LEDs from a string of lights that no >> longer emitted light, but still have continuity. If any one wants >> them to investigate, I'll mail them out. > > When you measure them with an ohmmeter, do they > show the same or similar resistance for either > polarity of the test leads? > > > Bob . . . > > ________________________________________________________________________________
Date: Apr 22, 2014
From: "Robert L. Nuckolls, III" <nuckolls.bob(at)aeroelectric.com>
Subject: Re: Half failed LEDs
At 10:31 AM 4/22/2014, you wrote: > >Most of them were in the 4-8k range both ways. A couple were in the >90k range both ways, and one was about 90k one way and about 150k the other. > > >They were in series with other LEDs and all carried enough current >to allow the other LEDs to continue to light. Interesting. I'm not familiar with the failure modes in LEDs . . . so getting first hand look at the parts would probably not be useful. The fact that your readings were so high but with conductivity in both directions suggests that there's some degree of P/N junction activity going on . . . in other words, I suspect they failed in some low energy event as opposed to high energy that tends to fuse things into more solid pieces of material. Bob . . . ________________________________________________________________________________
Date: Apr 22, 2014
From: rayj <raymondj(at)frontiernet.net>
Subject: Re: Half failed LEDs
They just weren't lit one day when I looked at them. They were on 24/7. I'll just recycle them, unless someone else wants them. Raymond Julian Kettle River, MN The things we admire in men, kindness and generosity, openness, honesty, understanding and feeling are the concomitants of failure in our system. And those traits we detest, sharpness, greed, acquisitiveness, meanness, egotism and self-interest are the traits of success. And while men admire the quality of the first they love the produce of the second. -John Steinbeck, novelist, Nobel laureate (1902-1968) On 04/22/2014 04:59 PM, Robert L. Nuckolls, III wrote: > > > At 10:31 AM 4/22/2014, you wrote: >> >> Most of them were in the 4-8k range both ways. A couple were in the >> 90k range both ways, and one was about 90k one way and about 150k the >> other. >> >> >> They were in series with other LEDs and all carried enough current to >> allow the other LEDs to continue to light. > > Interesting. I'm not familiar with the failure modes > in LEDs . . . so getting first hand look at the > parts would probably not be useful. The fact that > your readings were so high but with conductivity in > both directions suggests that there's some degree > of P/N junction activity going on . . . in other > words, I suspect they failed in some low energy > event as opposed to high energy that tends to fuse > things into more solid pieces of material. > > > Bob . . . > > ________________________________________________________________________________
Subject: Re: warning lights for engine T&Ps
From: "user9253" <fransew(at)gmail.com>
Date: Apr 23, 2014
While looking for the collection of Op Amp circuits that Eric referred to, I found this that might be useful to someone: http://www.ti.com/lit/an/snla140b/snla140b.pdf Joe -------- Joe Gores Read this topic online here: http://forums.matronics.com/viewtopic.php?p=422295#422295 ________________________________________________________________________________
Date: Apr 23, 2014
From: "Robert L. Nuckolls, III" <nuckolls.bob(at)aeroelectric.com>
Subject: Re: warning lights for engine T&Ps
At 10:11 AM 4/23/2014, you wrote: > >While looking for the collection of Op Amp circuits that Eric >referred to, I found this that might be useful to someone: >http://www.ti.com/lit/an/snla140b/snla140b.pdf >Joe > >-------- >Joe Gores Good find. This document is a classic. AN-31 has been around for decades and updated many times. It's a great teaching/learning tool! Bob . . . ________________________________________________________________________________
Date: Apr 23, 2014
From: "rv7a.builder" <rv7a.builder(at)yahoo.com>
Subject: Voltage drop thru the essential bus diode as shown
in Z-9. I am getting about 3/4 of a volt drop through the essential buss diode when the battery is at a full 12 volts.-When I throw the essential bus feed s witch the Voltmeter creeps back up to 12 volts. Is this fairly normal?-Th anks. John ________________________________________________________________________________
Date: Apr 23, 2014
From: "Robert L. Nuckolls, III" <nuckolls.bob(at)aeroelectric.com>
Subject: Re: Voltage drop thru the essential bus diode as
shown in Z-9. At 04:08 PM 4/23/2014, you wrote: >I am getting about 3/4 of a volt drop through the essential buss >diode when the battery is at a full 12 volts. When I throw the >essential bus feed switch the Voltmeter creeps back up to 12 volts. >Is this fairly normal? Thanks. John Yes. Diodes do have a small drop associated with them . . . which is insignificant when the alternator is running (bus at 14+ volts) and even less significant when the alternator is not running because the alt-feed switch is closed and the main bus is shut down. Bob . . . ________________________________________________________________________________
Subject: Re: warning lights for engine T&Ps
From: Sacha <uuccio(at)gmail.com>
Date: Apr 24, 2014
> > The device you linked seems like > it should do the job. You may have a wiring error... I tested the circuit on the bench by simulating the senders with a bunch of 1k ohm potentiometers wired in parallel (the resistance range of interest is 30-230 ohms approx in the case of the Rotax senders). Turns out I had indeed made a wiring error. The circuit I linked does the job perfectly as it's designed to measure resistance so it's output is pretty much independent of bus voltage, so there was no need for the dc-dc voltage regulator upstream. I calibrated them by simulating the senders with the 1k pots and connecting them to the gauges and the tri-color LED circuit. The sensitivity can be very finely adjusted with the two 33-turn pots that are on the (approx 1x2 inch) circuit board. I also managed to adapt the circuit to measure absolute bus voltage (for the high/low voltage light) by wiring a 7805 5v regulator thingy with three legs to get a fixed reference voltage. All in all I'm very satisfied with the results! ________________________________________________________________________________
Date: Apr 24, 2014
From: "Robert L. Nuckolls, III" <nuckolls.bob(at)aeroelectric.com>
Subject: Re: warning lights for engine T&Ps
At 04:45 PM 4/24/2014, you wrote: > > > > > The device you linked seems like > > it should do the job. You may have a wiring error... > >I tested the circuit on the bench by simulating the senders with a >bunch of 1k ohm potentiometers wired in parallel (the resistance >range of interest is 30-230 ohms approx in the case of the Rotax senders). >Turns out I had indeed made a wiring error. > >The circuit I linked does the job perfectly as it's designed to >measure resistance so it's output is pretty much independent of bus >voltage, so there was no need for the dc-dc voltage regulator upstream. > >I calibrated them by simulating the senders with the 1k pots and >connecting them to the gauges and the tri-color LED circuit. The >sensitivity can be very finely adjusted with the two 33-turn pots >that are on the (approx 1x2 inch) circuit board. > >I also managed to adapt the circuit to measure absolute bus voltage >(for the high/low voltage light) by wiring a 7805 5v regulator >thingy with three legs to get a fixed reference voltage. > >All in all I'm very satisfied with the results! Good for you sir . . . Bob . . . ________________________________________________________________________________
From: "DEAN PSIROPOULOS" <dean.psiropoulos(at)verizon.net>
Subject: SL-30 Poor VOR reception
Date: Apr 25, 2014
Hello listers: I have an RV-6A that I've been flying for a few years. When I first started flying the airplane I did some rough VOR range reception checks with the SL-30 and I had to get within 20 miles of a VOR station to get the receiver to lock on and the OBS indicator to stabilize with no flag displayed. I continued with the flight test regimen and getting familiar with the airplane and enjoying it. Since flights have been in VFR conditions around the busy Tampa (Florida) airspace I used the panel mounted moving map (GPS/Comm) to make sure I stayed out of the Class B, there was no need to have good VOR reception. The Com portion was the only thing I was using on the SL-30 and that worked very well. Then, a couple months ago I turned on the SL-30 and, nothing, no display or Com so I returned the unit to Garmin for repair. In the meantime I'd been talking to a CFII about working on my instrument proficiency to get back up to FAA standards after a long hiatus. When the SL-30 was repaired and back in the instrument panel I took off on a flight to Ocala to check it out. I climbed up to 4500 feet and watched the OBS and listened for the ID code as I entered the 40 mile radius of the Ocala VOR. Nothing so I kept going on a course straight for the station. Around 20-25 miles out I started getting activity on the indicator but the flags were intermittent and the radio could not maintain a lock. Finally about 15 miles out the indicators stabilized and the flags stayed off, obviously something is amiss. I'm using a Comant CI-159 VOR/ILS antenna. This unit consists of a phenolic "puck" with a BNC connector cast into it for the coax connection. There are two fiberglas elements (thin rods about 18 inches long) that screw into the puck to form a "V" shaped antenna. I have it mounted on the bottom of the fuselage in the back of the airplane under the horizontal stabilizer. The BNC sticks through a hole in the belly and appx 25 foot length of RG-58 coax runs inside the aircraft up to the SL-30 receiver. No gasket was supplied with the antenna so I just bolted the puck to the aluminum using the two bolt holes. After the Ocala flight I removed the puck and elements for closer inspection and checked the connector on the coax cable at the antenna end. Nothing wrong that I could tell just by looking at the equipment. There was a bit of engine oil (from the exhaust) on the puck surface facing the skin. I cleaned it off and made a gasket to go between the puck and the skin to minimize the oil build up again. I have not flown the airplane since but I doubt that lack of a gasket was causing the poor reception, maybe someone here can tell me. The other thing I checked was the resistance between the center pin and outside barrel of the BNC connector on the antenna. Resistance was showing basically a dead short (less than 1 ohm, same reading as touching the meter probes together). I'm not an antenna designer so I don't know if this is what the reading should be or not. Aircraft Spruce has the following description for this antenna: " "V" Dipole VOR/Glide Slope Antenna with detachable elements is similar to the CI 158C-3 with the exception of offering 2-hole mount instead of a 4-hole mount. This V Dipole encompasses reduced static capability with the use of P-Stat paint. Integral ferrite balun provides for higher radiation efficiency." . I don't know if the construction of the antenna would cause the meter to show such a low resistance, maybe some here could shed some light on that. Any ideas, experiences with poor VOR reception here on the list? Since the radio has been repaired and tested by the factory I'd say it's not the radio. I checked the connector at the back of the radio tray and it appears secure so I'm not sure what else to do. Since VORs are being decommissioned there is going to be less and less need for VOR reception but if I have to use victor airways during instrument operations I need to have good reception at the expected range or I won't be able to comply with ATC clearances. On an side note, a fellow RV-9 builder who is using a simple flat metal antenna in the wingtip, has no problem receiving the VOR 40 miles out, and he didn't pay anywhere near what I did for his antenna. Thanks for the help Dean Psiropoulos RV-6A N197DM 200+ hours since 2008 ________________________________________________________________________________
Date: Apr 25, 2014
From: "Robert L. Nuckolls, III" <nuckolls.bob(at)aeroelectric.com>
Subject: Re: SL-30 Poor VOR reception
When the SL-30 was repaired and back in the instrument panel I took off on a flight to Ocala to check it out. I climbed up to 4500 feet and watched the OBS and listened for the ID code as I entered the 40 mile radius of the Ocala VOR. Nothing so I kept going on a course straight for the station. Around 20-25 miles out I started getting activity on the indicator but the flags were intermittent and the radio could not maintain a lock. Finally about 15 miles out the indicators stabilized and the flags stayed off, obviously something is amiss. Since the radio has enjoyed a recent bench check, probability of it being a radio problem is considerably reduced . . . at the bottom of the list . . . I have not flown the airplane since but I doubt that lack of a gasket was causing the poor reception, maybe someone here can tell me. Agreed . . . gaskets tend to be hedges against moisture/dirt and have no effect on performance of any antenna. The other thing I checked was the resistance between the center pin and outside barrel of the BNC connector on the antenna. Resistance was showing basically a dead short (less than 1 ohm, same reading as touching the meter probes together). 40 years ago, this would be uncommon. Nowadays, antenna designers have deduced value for incorporation of impedance matching a balancing networks between the antenna element(s) and the feed line. Further, designing a network with a DC path to ground is a further improvement for reduction of noise due to p-static. This V Dipole encompasses reduced static capability with the use of P-Stat paint. A further step forward in the art and science of antennas for aircraft. Integral ferrite balun provides for higher radiation efficiency." Yeah . . . sort of . . . measurable in the lab but of zero observable significance to the pilot. I don't know if the construction of the antenna would cause the meter to show such a low resistance, maybe some here could shed some light on that. Yes . . . your observations are not out of the ordinary . . . Any ideas, experiences with poor VOR reception here on the list? Since the radio has been repaired and tested by the factory I'd say it's not the radio. I checked the connector at the back of the radio tray and it appears secure so I'm not sure what else to do. Since VORs are being decommissioned there is going to be less and less need for VOR reception but if I have to use victor airways during instrument operations I need to have good reception at the expected range or I won't be able to comply with ATC clearances. On an side note, a fellow RV-9 builder who is using a simple flat metal antenna in the wingtip, has no problem receiving the VOR 40 miles out, and he didn't pay anywhere near what I did for his antenna. Thanks for the help. A not uncommon observation. The quest for uber-efficient antennas on aircraft is problematic. The guys with hammers- n-saws in the lab can do a fine job of 'optimizing' a design to textbook ideals but we're not communicating with space probes or folks with hand-helds over the horizon. The nominal signal-to-noise ratios for air-to-ground contact by radio are huge. For the most part, a "wet-string" will offer serviceable performance as an antenna. Do you have access to an antenna analyzer . . . Emacs! or perhaps an SWR meter . . . Emacs! The FIRST thing to do after checking the obvious (mechanical connections) is to get some numbers. An antenna analyzer hooked to the receiver end of the transmission line is the best way to assess antenna health. Even use of an SWR meter like the 'red dot' instruments off eBay can be excited with the ship's comm transceiver (or a hand held) set to the lowest comm frequency. The SWR should be relatively low even though you're measuring at the top of it's design range. Put a dummy load http://tinyurl.com/cchp3pf at the antenna end of your VOR feed line, connect the feedline to the transceiver and then "talk" to it while monitoring SWR. Alternatively, make a temporary connection of your comm antenna into the VOR receiver input jack and go fly the airplane. I'm betting that you're going to see a marked improvement in VOR performance. If your access to test equipment is limited, you can play the swaptronics game . . . replace the connectors on the ends of the feed line just for the heck of it. It takes less time to do this simple experiment than all the time spent thus far looking for 'rate in the woodpile'. It seems unlikely that the antenna is at fault. feedline installation errors are a higher probability. What ever the problem is . . . it's stone simple Bob . . . ________________________________________________________________________________
Subject: Re: SL-30 Poor VOR reception
From: "user9253" <fransew(at)gmail.com>
Date: Apr 25, 2014
There could be a problem with one of the coax connectors. You could replace the RG-58 cable with RG-400, which has less signal loss. If the coax is bent with too small of a radius, that can cause problems. Another concern is the antenna location. What is the distance between the horizontal stabilizer and the antenna? This distance can have a big affect on the antenna performance. Radio Frequency current behaves much differently than DC current. An open circuit for DC can be a short circuit for RF. And a short circuit for DC can be an open circuit for RF. Joe -------- Joe Gores Read this topic online here: http://forums.matronics.com/viewtopic.php?p=422373#422373 ________________________________________________________________________________
Date: Apr 25, 2014
From: Jeff Luckey <jluckey(at)pacbell.net>
Subject: Thanks Bob
Bob, I forgot to say thank you for the magneto trouble-shooting guide you sent a couple of weeks ago... So thank you, very helpful -Jeff ________________________________________________________________________________
From: "David Lloyd" <skywagon(at)charter.net>
Subject: Re: SL-30 Poor VOR reception
Date: Apr 25, 2014
Great advice by all concerning this problem . . . . However, I would not place the fact that the radio just came out of factory bench testing on the bottom of the problem list. I have had a few situations where factory repairs were very much less than acceptable. The latest was an autopilot recalibration when GPS tracking was not accurate. Unit came back still with poor tracking. After much arguing with company, an officer went down to the final bench test and calibration station and found an error in the factory final test set-up. Another was a Navigator that had very specific squelch breaks occurring at odd but, specific frequencies. Turned out the auto squelch factory test stand was set-up wrong. I don't understand how such problems can exist in a well respected company but, it happens. Let us know what you find with your radio problem. . . ------------------------------------------------------------------------- ------- ----- Original Message ----- From: Robert L. Nuckolls, III To: aeroelectric-list(at)matronics.com Sent: Friday, April 25, 2014 8:52 AM Subject: Re: AeroElectric-List: SL-30 Poor VOR reception When the SL-30 was repaired and back in the instrument panel I took off on a flight to Ocala to check it out. I climbed up to 4500 feet and watched the OBS and listened for the ID code as I entered the 40 mile radius of the Ocala VOR. Nothing so I kept going on a course straight for the station. Around 20-25 miles out I started getting activity on the indicator but the flags were intermittent and the radio could not maintain a lock. Finally about 15 miles out the indicators stabilized and the flags stayed off, obviously something is amiss. Since the radio has enjoyed a recent bench check, probability of it being a radio problem is considerably reduced . . . at the bottom of the list . . . I have not flown the airplane since but I doubt that lack of a gasket was causing the poor reception, maybe someone here can tell me. Agreed . . . gaskets tend to be hedges against moisture/dirt and have no effect on performance of any antenna. The other thing I checked was the resistance between the center pin and outside barrel of the BNC connector on the antenna. Resistance was showing basically a dead short (less than 1 ohm, same reading as touching the meter probes together). 40 years ago, this would be uncommon. Nowadays, antenna designers have deduced value for incorporation of impedance matching a balancing networks between the antenna element(s) and the feed line. Further, designing a network with a DC path to ground is a further improvement for reduction of noise due to p-static. This V Dipole encompasses reduced static capability with the use of P-Stat paint. A further step forward in the art and science of antennas for aircraft. Integral ferrite balun provides for higher radiation efficiency." Yeah . . . sort of . . . measurable in the lab but of zero observable significance to the pilot. I don't know if the construction of the antenna would cause the meter to show such a low resistance, maybe some here could shed some light on that. Yes . . . your observations are not out of the ordinary . . . Any ideas, experiences with poor VOR reception here on the list? Since the radio has been repaired and tested by the factory I'd say it's not the radio. I checked the connector at the back of the radio tray and it appears secure so I'm not sure what else to do. Since VORs are being decommissioned there is going to be less and less need for VOR reception but if I have to use victor airways during instrument operations I need to have good reception at the expected range or I won't be able to comply with ATC clearances. On an side note, a fellow RV-9 builder who is using a simple flat metal antenna in the wingtip, has no problem receiving the VOR 40 miles out, and he didn't pay anywhere near what I did for his antenna. Thanks for the help. A not uncommon observation. The quest for uber-efficient antennas on aircraft is problematic. The guys with hammers- n-saws in the lab can do a fine job of 'optimizing' a design to textbook ideals but we're not communicating with space probes or folks with hand-helds over the horizon. The nominal signal-to-noise ratios for air-to-ground contact by radio are huge. For the most part, a "wet-string" will offer serviceable performance as an antenna. Do you have access to an antenna analyzer . . . or perhaps an SWR meter . . . The FIRST thing to do after checking the obvious (mechanical connections) is to get some numbers. An antenna analyzer hooked to the receiver end of the transmission line is the best way to assess antenna health. Even use of an SWR meter like the 'red dot' instruments off eBay can be excited with the ship's comm transceiver (or a hand held) set to the lowest comm frequency. The SWR should be relatively low even though you're measuring at the top of it's design range. Put a dummy load http://tinyurl.com/cchp3pf at the antenna end of your VOR feed line, connect the feedline to the transceiver and then "talk" to it while monitoring SWR. Alternatively, make a temporary connection of your comm antenna into the VOR receiver input jack and go fly the airplane. I'm betting that you're going to see a marked improvement in VOR performance. If your access to test equipment is limited, you can play the swaptronics game . . . replace the connectors on the ends of the feed line just for the heck of it. It takes less time to do this simple experiment than all the time spent thus far looking for 'rate in the woodpile'. It seems unlikely that the antenna is at fault. feedline installation errors are a higher probability. What ever the problem is . . . it's stone simple Bob . . . ________________________________________________________________________________
Date: Apr 25, 2014
From: "Robert L. Nuckolls, III" <nuckolls.bob(at)aeroelectric.com>
Subject: Re: SL-30 Poor VOR reception
At 01:12 PM 4/25/2014, you wrote: >Great advice by all concerning this problem . . . . > >However, I would not place the fact that the radio just came out of >factory bench testing on the bottom of the problem list. > >I have had a few situations where factory repairs were very much >less than acceptable. >The latest was an autopilot recalibration when GPS tracking was not >accurate. Unit came back still with poor tracking. After much >arguing with company, an officer went down to the final bench test >and calibration station and found an error in the factory final test set-up. >Another was a Navigator that had very specific squelch breaks >occurring at odd but, specific frequencies. Turned out the auto >squelch factory test stand was set-up wrong. >I don't understand how such problems can exist in a well respected >company but, it happens. > >Let us know what you find with your radio problem. . . Another investigative test option is to attach a VOR/COMM hand held to the antenna feed line to rule out panel mounted radio problems. Bob . . . ________________________________________________________________________________
Date: Apr 25, 2014
From: "Robert L. Nuckolls, III" <nuckolls.bob(at)aeroelectric.com>
Subject: Re: Thanks Bob
At 01:00 PM 4/25/2014, you wrote: >Bob, > >I forgot to say thank you for the magneto trouble-shooting guide you >sent a couple of weeks ago... > >So thank you, very helpful > >-Jeff What did you find? Bob . . . ________________________________________________________________________________
Date: Apr 25, 2014
Subject: Re: Thanks Bob
From: "j. davis" <jwd3ca(at)gmail.com>
I'd be interested in seeing that, Jeff. Thanks! On 25 Apr 2014 14:09, "Jeff Luckey" wrote: > Bob, > > I forgot to say thank you for the magneto trouble-shooting guide you sent > a couple of weeks ago... > > So thank you, very helpful > > -Jeff > > * > > > * > > ________________________________________________________________________________
Date: Apr 25, 2014
From: Jeff Luckey <jluckey(at)pacbell.net>
Subject: Magneto Noise
Per request from J. Davis...=0A=0A=0A----- Forwarded Message -----=0AFrom: "Robert L. Nuckolls, III" =0ATo: aeroelectri c-list(at)matronics.com =0ASent: Tuesday, April 8, 2014 8:06 AM=0ASubject: Re: AeroElectric-List: Magneto Noise=0A =0A=0A--> AeroElectric-List message po sted by: "Robert L. Nuckolls, III" =0A=0AAt 12:04 PM 4/6/2014, you wrote:=0A=0AI flew w/ a friend recently and when try ing to get awos for our =0Adestination airport (which is a pretty weak sign al) it was almost =0Aimpossible to hear due to magneto noise.- The radio is a Garmin 430W =0Ain a Cherokee and we were around 10 miles from the airp ort.- When we =0Agot within 3 or 4 miles the signal was much clearer.- All other =0Atransmissions with ATC & towers were acceptable but I noticed at high =0Avolume I could clearly hear magneto noise in the background.=0A =0AI'm wondering if that's just the way it is or if there is a problem =0Aw ith the magnetos or P lead wiring.=0A=0A- - Was this a new condition or has it been that=0A- - way 'forever'. It's difficult for the neophyte =0A- - observer to tell the difference between ignition=0A- - noise s getting into the system by way of plug wire=0A- - radiation versus p- lead radiation.=0A=0A- - First, make sure that the perceived noise goes up=0A- - and down with volume control settings on radio.=0A- - Thi s all but guarantees that it's coming in through=0A- - the antenna.=0A =0A- - Then, disconnect both p-leads from the back of the=0A- - mag netos and run the engine. See if the noise is=0A- - still heard . . . t urn to unused frequency . . .=0A- - open squelch.=0A=0A- - If you c an still hear it, then look for bad plug=0A- - wire. If not, hook up on e p-lead at a time and=0A- - repeat experiment . . . see if noise is pr edicated=0A- - on either or both of the p-leads being hooked up.=0A=0A - - Once you've identified the offending radiator,=0A- - you need t o figure out why. If this is a 'new'=0A- - condition, then you're looki ng for something that=0A- - has changed. Bad cap in mag (probably not b oth)=0A- - broken ground lead on p-lead shield. If possible/=0A- - practical wire p-lead wires per Figure Z-27=0A=0Ahttp://tinyurl.com/n3oy37f =0A=0A- - Ground p-lead shields at magneto end only . . .=0A- - use p-lead shields to ground mag switch. Remove=0A- - and existing ground to the mag switch.=0A=0A- - If the noise is coming from both p-leads, =0A- - there are 'approved' magneto noise filters=0A=0Ahttp://tinyurl.c om/l3s4mrj=0A=0A- - that most mechanics will install with a simple=0A - - log-book entry.=0A=0A- - Bob . . .=0A=0A=0A- Bob . . .- ==================== ________________________________________________________________________________
Date: Apr 25, 2014
Subject: Re: Magneto Noise
From: "j. davis" <jwd3ca(at)gmail.com>
Thanks, Jeff. I was thinking it had to do with ignition issues (rather than radio noise ) but I'll file it away for future reference. On 25 Apr 2014 16:08, "Jeff Luckey" wrote: > Per request from J. Davis... > > > ----- Forwarded Message ----- > *From:* "Robert L. Nuckolls, III" > *To:* aeroelectric-list(at)matronics.com > *Sent:* Tuesday, April 8, 2014 8:06 AM > *Subject:* Re: AeroElectric-List: Magneto Noise > > nuckolls.bob(at)aeroelectric.com> > > At 12:04 PM 4/6/2014, you wrote: > > I flew w/ a friend recently and when trying to get awos for our > destination airport (which is a pretty weak signal) it was almost > impossible to hear due to magneto noise. The radio is a Garmin 430W > in a Cherokee and we were around 10 miles from the airport. When we > got within 3 or 4 miles the signal was much clearer. All other > transmissions with ATC & towers were acceptable but I noticed at high > volume I could clearly hear magneto noise in the background. > > I'm wondering if that's just the way it is or if there is a problem > with the magnetos or P lead wiring. > > Was this a new condition or has it been that > way 'forever'. It's difficult for the neophyte > observer to tell the difference between ignition > noises getting into the system by way of plug wire > radiation versus p-lead radiation. > > First, make sure that the perceived noise goes up > and down with volume control settings on radio. > This all but guarantees that it's coming in through > the antenna. > > Then, disconnect both p-leads from the back of the > magnetos and run the engine. See if the noise is > still heard . . . turn to unused frequency . . . > open squelch. > > If you can still hear it, then look for bad plug > wire. If not, hook up one p-lead at a time and > repeat experiment . . . see if noise is predicated > on either or both of the p-leads being hooked up. > > Once you've identified the offending radiator, > you need to figure out why. If this is a 'new' > condition, then you're looking for something that > has changed. Bad cap in mag (probably not both) > broken ground lead on p-lead shield. If possible/ > practical wire p-lead wires per Figure Z-27 > > http://tinyurl.com/n3oy37f > > Ground p-lead shields at magneto end only . . . > use p-lead shields to ground mag switch. Remove > and existing ground to the mag switch. > > If the noise is coming from both p-leads, > there are 'approved' magneto noise filters > > http://tinyurl.com/l3s4mrj > > that most mechanics will install with a simple > log-book entry. > > Bob . . . > > > Bob . . . > > > > > > <http://www.matronics.com/Navigator?AeroElectric-List> > > * > > > * > > ________________________________________________________________________________
Date: Apr 25, 2014
From: Jeff Luckey <jluckey(at)pacbell.net>
Subject: Re: Thanks Bob
At the moment, my findings are inconclusive.- =0A=0AI began the test proc ess in the run-up area and found that I was not able to reproduce the sympt oms.=0A=0A1. Engine @ 1800 RPM=0A2. Tune 430 to an unused freq=0A3. Open sq uelch=0A=0AI did not hear the popping or buzzing sound that I heard a coupl e of weeks ago in flight (which prompted my original query to the List).=0A =0A=0AThen Life happened & I have not been able to get back to it - maybe t his weekend.=0A=0A-Jeff=0A=0A=0A________________________________=0A From: " Robert L. Nuckolls, III" =0ATo: aeroelectric -list(at)matronics.com =0ASent: Friday, April 25, 2014 12:03 PM=0ASubject: Re: AeroElectric-List: Thanks Bob=0A =0A=0A--> AeroElectric-List message poste d by: "Robert L. Nuckolls, III" =0A=0AAt 01: 00 PM 4/25/2014, you wrote:=0A>Bob,=0A>=0A>I forgot to say thank you for th e magneto trouble-shooting guide you =0A>sent a couple of weeks ago...=0A> =0A>So thank you, very helpful=0A>=0A>-Jeff=0A=0A- What did you find?=0A - - - - - - - - - - - - -Matt Dralle, List Admin. ==== ________________________________________________________________________________
Date: Apr 25, 2014
From: Kelly McMullen <kellym(at)aviating.com>
Subject: Re: SL-30 Poor VOR reception
I agree with connector issues. I do NOT agree with cable issues. Ever since the VOR came on the scene, RG-58 has proved more than adequate on factory built spam cans. Most have runs of over 20 ft from panel to top of tail. Often with one or more connectors in the line. I had such on a Cessna that would easily receive 100nm on H class VOR at 10,000 ft. I could connect handheld to splitter for same antenna and it also would receive same 100+ nm range. RG 142 and 400 are better, but not 6 times better, which is the price difference. I doubt they are even 50% better. On 4/25/2014 10:32 AM, user9253 wrote: > > There could be a problem with one of the coax connectors. You could replace the RG-58 cable with RG-400, which has less signal loss. If the coax is bent with too small of a radius, that can cause problems. Another concern is the antenna location. What is the distance between the horizontal stabilizer and the antenna? This distance can have a big affect on the antenna performance. > Radio Frequency current behaves much differently than DC current. An open circuit for DC can be a short circuit for RF. And a short circuit for DC can be an open circuit for RF. > Joe > > -------- > Joe Gores > > > Read this topic online here: > > http://forums.matronics.com/viewtopic.php?p=422373#422373 > > ________________________________________________________________________________
Date: Apr 26, 2014
From: D L Josephson <dlj04(at)josephson.com>
Subject: Re: Avionics-List: Garmin SL-30 poor VOR reception
This question is being discussed both here and on the avionics list. Commercial RG-58A, mil spec RG-58 (MIL-DTL-17 M17/28-RG058) such as Belden 9203, RG-142 and RG400/LMR400 are all rated between 3.3 and 4.9 dB per 100 ft at 100 MHz -- around 1 dB for a practical airplane installation. Belden 9203 RG58 is among the lowest loss and is mil QPL qualified. Cable loss is not really a factor in a VOR/LOC/GS installation; well made RG58 cable is fine. Cheap RG-58 "type" cable of unknown impedance and bend resistance, who knows. Original RG-58 (solid center conductor, solid PE dielectric) was just fine for decades with receivers far worse than the Garmin. In my experience, connectors are the problem nine times out of ten. Commercial grade crimp connectors from a manufacturer that also supplies military grade connectors (Amp, Amphenol, Kings) crimped with the manufacturer-specified or mil-spec tool checked for the correct crimp dimensions by someone experienced with this process is the key. Original military soldered connectors are also OK but only if the installer is experienced enough to get the dimensions right without melting the dielectric. Quick test, after you've confirmed that the cable has continuity end to end and isn't shorted: grab the cable in one hand and the body of the connector or the crimp sleeve in the other. Using moderate force, can you rotate the connector body with respect to the cable? If so, cut it off and try again, it is not terminated correctly. This isn't the cause of all problems but is usually diagnostic of whether it was crimped right in the first place. The tests Bob suggested are good. A nav receiver will work well -- not optimally for IFR but well enough -- on a comm antenna. ________________________________________________________________________________
Date: Apr 26, 2014
From: "Robert L. Nuckolls, III" <nuckolls.bob(at)aeroelectric.com>
Subject: Re: SL-30 Poor VOR reception
At 04:47 PM 4/25/2014, you wrote: > >I agree with connector issues. I do NOT agree with cable issues. >Ever since the VOR came on the scene, RG-58 has proved more than >adequate on factory built spam cans. Most have runs of over 20 ft >from panel to top of tail. Often with one or more connectors in the >line. I had such on a Cessna that would easily receive 100nm on H >class VOR at 10,000 ft. I could connect handheld to splitter for >same antenna and it also would receive same 100+ nm range. RG 142 >and 400 are better, but not 6 times better, which is the price >difference. I doubt they are even 50% better. Agreed. The strength of vor transmitters and the fundamentally line-of-sight operation makes signal loss in coax a non-issue. The greatest gain for a modern coax is in quality of insulation. Legacy RG-58 is pvc and polyethylene while the modern plastics are tefzel and more robust cousins. RG-400 and siblings are preferred for new construction but I wouldn't replace RG-58 already in place. Bob . . . ________________________________________________________________________________
Date: Apr 26, 2014
From: "Robert L. Nuckolls, III" <nuckolls.bob(at)aeroelectric.com>
Subject: Speaking of coax
While we're on the subject, know that there's a new line of coax cables being offered as lower loss and better shielded than the legacy RG products of PVC/PE . . . They are part of the LMR series products like this http://tinyurl.com/l2xbbqr The value of the part numbers is the outside diameter in thousandths. Hence, LMR195 is the same physical size as RG58 and RG400. The insulation is not high temperature rated, it's more on the order of that stated for RG58 so soldering connectors to this stuff takes some practice. However, for all crimped joints, there's little difference between the LMR and any other style. The LMR195 has a solid center conductor which harkens back to the days when solid center conductor RG58 can short to the shield if installed with a sharp bend radius and then subjected to many years of temperature cycling. This is a low-risk failure . . . I've only heard of it one time and that was while I was still at Cessna the first time. The center conductor insulation is foamed which produces a lower loss than legacy RG-58. The shield is a combination of braid over foil which offers near perfect shielding. . . . and the price is right. Bob . . . ________________________________________________________________________________
Subject: Single ground vs. distributed ground
From: "donjohnston" <don@velocity-xl.com>
Date: Apr 26, 2014
I'm in the process of wiring the panel and avionics on my Velocity (composite, canard). I have built an Avionics Shelf which runs the width of the cabin just behind the panel. The shelf is made out of a pair of 1" aluminum angle stock and a sheet of aluminum on top. On the right side of the shelf I have mounted a ground block that connects to the battery via 2AWG welding cable. As I am connecting wires, the number of wires that are attaching to the ground block keeps increasing. [Embarassed] I can think of a couple of solutions: 1) crimp multiple wires to a single terminal. With the 22AWG wires, I could get up to four wires in a single (red) terminal . 2) Make another ground block and mount it to the left side on the avionics shelf. This would have the added benefit of eliminating ground wires from the bundle crossing over to the right side ground block. I'm leaning towards #2, but I'm wondering what the opinions are on these solutions or if there's one I haven't thought of. Thanks in advance. Don Read this topic online here: http://forums.matronics.com/viewtopic.php?p=422415#422415 ________________________________________________________________________________
Subject: Re: Single ground vs. distributed ground
From: Tim Andres <tim2542(at)sbcglobal.net>
Date: Apr 26, 2014
Buy an avionics ground bus from Bob K. Sweet and simple single point ground system for the avionics. Just wondering....why 2awg wire to the avionics? Seems rather large. Tim > On Apr 26, 2014, at 2:12 PM, "donjohnston" <don@velocity-xl.com> wrote: > > > I'm in the process of wiring the panel and avionics on my Velocity (composite, canard). > > I have built an Avionics Shelf which runs the width of the cabin just behind the panel. The shelf is made out of a pair of 1" aluminum angle stock and a sheet of aluminum on top. > > On the right side of the shelf I have mounted a ground block that connects to the battery via 2AWG welding cable. > > As I am connecting wires, the number of wires that are attaching to the ground block keeps increasing. [Embarassed] > > I can think of a couple of solutions: > > 1) crimp multiple wires to a single terminal. With the 22AWG wires, I could get up to four wires in a single (red) terminal . > > 2) Make another ground block and mount it to the left side on the avionics shelf. This would have the added benefit of eliminating ground wires from the bundle crossing over to the right side ground block. > > I'm leaning towards #2, but I'm wondering what the opinions are on these solutions or if there's one I haven't thought of. > > Thanks in advance. > Don > > > > > Read this topic online here: > > http://forums.matronics.com/viewtopic.php?p=422415#422415 > > > > > > > > > > ________________________________________________________________________________
Subject: Re: Single ground vs. distributed ground
From: "donjohnston" <don@velocity-xl.com>
Date: Apr 26, 2014
tim2542(at)sbcglobal.net wrote: > Buy an avionics ground bus from Bob K. Well that would result in scrapping and undoing what I've already done. > Just wondering....why 2awg wire to the avionics? Seems rather large. Couple of reason: 1) I have more than just the avionics being grounded at that block (trim motor, AP servo, cooling fan, etc.). 2) I had the wire and the room. Read this topic online here: http://forums.matronics.com/viewtopic.php?p=422418#422418 ________________________________________________________________________________
Date: Apr 26, 2014
From: "Robert L. Nuckolls, III" <nuckolls.bob(at)aeroelectric.com>
Subject: Re: OT: power supply noise
At 03:10 PM 4/26/2014, you wrote: > >Greetings, > >I'm considering putting an unregulated power supply (transformers, >diodes, and a condenser) in an old computer box along with the >electronics to run several stepper motors for a CNC system. > >I'm wondering if noise radiated from the power supply will cause >problems for the stepper motor drivers. Probably not. Do you have these parts already in hand? Regulated, clean power supplies up to 350W are pretty cheap on eBay. I seldom build a supply any more, the labor alone is more than the cost of an off-the-shelf, plug-n-play device. Bob . . . ________________________________________________________________________________
From: "Carlos Trigo" <trigo(at)mail.telepac.pt>
Subject: Linking 2 coax cables
Date: Apr 27, 2014
Guys I had to cut and throw away some 2 feet of a Comm antenna cable which I found that had a burnt spot. I don't want to replace all the coax cable, which would be a PIA to do. I also know that I can use a male and a female BNC connectors to make a 2 feet extension, but Is there an elegant and efficient way to connect 2 coax cables to each other, without using a male and a female connectors? Regards Carlos --- Este email est livre de vrus e malware porque a proteo avast! Antivirus est ativa. http://www.avast.com ________________________________________________________________________________
Date: Apr 27, 2014
From: "rv7a.builder" <rv7a.builder(at)yahoo.com>
Subject: Amp meter jumps when transmitting
Hello Group,=0A---- I have a Vans amp meter installed in my RV-7A. When I activate the PTT on the Garmin GNC300XL-the amp meter jumps to ful l scale. I disconnect the antenna coax from back of the radio and hit the P TT and the amp meter needle stays still. I turn off the radio and bring my handheld into the cockpit and hit the transmit button from 2 feet away-an d a very slight movement in the amp meter is detected.-Bringing the handh eld closer to the amp meter will make the needle jump more when the PTT is activated.-I think I have-determined this to be a RF-inference proble m.-My questions are 1) Will the amp meter eventually be damaged by these extreme movements? 2) Is there a fix? 3) Am I alone here or have others exp erience this phenomena?=0A-=0AThanks. John. RV-7A Just about ready to fly ________________________________________________________________________________
Date: Apr 27, 2014
From: "Robert L. Nuckolls, III" <nuckolls.bob(at)aeroelectric.com>
Subject: Re: Amp meter jumps when transmitting
At 07:35 AM 4/27/2014, you wrote: Hello Group, I have a Vans amp meter installed in my RV-7A. When I activate the PTT on the Garmin GNC300XL the amp meter jumps to full scale. I disconnect the antenna coax from back of the radio and hit the PTT and the amp meter needle stays still. I turn off the radio and bring my handheld into the cockpit and hit the transmit button from 2 feet away and a very slight movement in the amp meter is detected. Bringing the handheld closer to the amp meter will make the needle jump more when the PTT is activated. I think I have determined this to be a RF inference problem. I bought a Van's ammeter some years ago to explore it's vulnerabilities to strong RF. See pictures here: http://tinyurl.com/mpx8hze I discovered that no only was the instrument devoid of any firewall for RF interference, it was even directionally sensitive!!! See pictures http://tinyurl.com/m4xlj2u http://tinyurl.com/kvo4vwk Just waving the hand-held from side to side would produce behaviors in the pointer error that mimicked the motion of the antenna. My questions are 1) Will the amp meter eventually be damaged by these extreme movements? No 2) Is there a fix? Probably, but labor intensive and clumsy on a finished instrument. It would have been FAR better that the manufacturer demonstrated a knowledge of DO-160 in the first place. 3) Am I alone here or have others experience this phenomena? No doubt there are many others. Since it's a transient event that affects the display only while talking, it's a cosmetic behavior. You might consider just 'living' with it. With one caveat . . . make sure that your coax connectors are good at both ends of the feedline . . . it wouldn't hurt to do an SWR check. The phenomenon you're observing MIGHT be the result of a very RF-Hot cockpit due to loss of shield ground on either end of the coax. But it your antenna system is golden, then there are no risks for just ignoring it. I wrote to Van's about this condition . . . never heard back from them. It no doubt affects their entire line of instruments with electronic signal conditioning. Bob . . . ________________________________________________________________________________
Date: Apr 27, 2014
From: "Robert L. Nuckolls, III" <nuckolls.bob(at)aeroelectric.com>
Subject: Re: Linking 2 coax cables
At 06:37 AM 4/27/2014, you wrote: > > >Guys > >I had to cut and throw away some 2 feet of a Comm antenna cable which I >found that had a burnt spot. >I don't want to replace all the coax cable, which would be a PIA to do. >I also know that I can use a male and a female BNC connectors to make a 2 >feet extension, but >Is there an elegant and efficient way to connect 2 coax cables to each >other, without using a male and a female connectors? Yeah, sort of but it's pretty process sensitive. I'd vote for the connectors. Bob . . . ________________________________________________________________________________
Date: Apr 27, 2014
From: "rv7a.builder" <rv7a.builder(at)yahoo.com>
Subject: Re: Amp meter jumps when transmitting
Thanks Bob for that great analysis. John.=0AOn Sunday, April 27, 2014 6:57 AM, "Robert L. Nuckolls, III" wrote:=0A olls.bob(at)aeroelectric.com>=0A=0AAt 07:35 AM 4/27/2014, you wrote:=0AHello G roup,=0A- - - I have a Vans amp meter installed in my RV-7A. When I a ctivate =0Athe PTT on the Garmin GNC300XL the amp meter jumps to full scale . I =0Adisconnect the antenna coax from back of the radio and hit the PTT =0Aand the amp meter needle stays still. I turn off the radio and bring =0A my handheld into the cockpit and hit the transmit button from 2 feet =0Aawa y and a very slight movement in the amp meter is detected. =0ABringing the handheld closer to the amp meter will make the needle =0Ajump more when the PTT is activated. I think I have determined this =0Ato be a RF inference p roblem.=0A=0A- - I bought a Van's ammeter some years ago to=0A- - e xplore it's vulnerabilities to strong RF.=0A=0A- - See pictures here: =0A=0Ahttp://tinyurl.com/mpx8hze=0A=0A- - I discovered that no only was the instrument=0A- - devoid of any firewall for RF interference,=0A- - it was even directionally sensitive!!! See=0A- - pictures=0A=0Ahtt p://tinyurl.com/m4xlj2u=0A=0Ahttp://tinyurl.com/kvo4vwk=0A=0A- - Just w aving the hand-held from side to side=0A- - would produce behaviors in the pointer error=0A- - that mimicked the motion of the antenna.=0A=0A =0AMy questions are 1) Will the amp meter eventually be damaged by these =0Aextreme movements?=0A=0A- - No=0A=0A- 2) Is there a fix?=0A=0A- - Probably, but labor intensive and clumsy on=0A- - a finished instru ment. It would have been FAR=0A- - better that the manufacturer demonst rated=0A- - a knowledge of DO-160 in the first place.=0A=0A- 3) Am I alone here or have others experience this phenomena?=0A=0A=0A- - No dou bt there are many others. Since it's=0A- - a transient event that affec ts the display=0A- - only while talking, it's a cosmetic behavior.=0A - - You might consider just 'living' with it.=0A=0A- - With one cav eat . . . make sure that your coax=0A- - connectors are good at both en ds of the feedline . . .=0A- - it wouldn't hurt to do an SWR check.- The=0A- - phenomenon you're observing MIGHT be the=0A- - result of a very RF-Hot cockpit due to loss=0A- - of shield ground on either end of the coax.=0A=0A- - But it your antenna system is golden, then=0A- - there are no risks for just ignoring it.=0A=0A- - I wrote to Van's about this condition . . . never=0A- - heard back from them. It no doub t affects=0A- - their entire line of instruments with=0A- - electro =============== ________________________________________________________________________________
Date: Apr 27, 2014
Subject: Re: Amp meter jumps when transmitting
From: Dj Merrill <deej(at)deej.net>
On 4/27/2014 8:35 AM, rv7a.builder wrote: > I disconnect the antenna coax from back of the radio and hit the PTT > and the amp meter needle stays still. Just as an aside, it is generally a bad idea to activate a transmitter with no antenna attached. There is a good chance of damaging the transmitter by doing this. fyi -Dj -- Dj Merrill - N1JOV - VP EAA Chapter 87 Sportsman 2+2 Builder #7118 N421DJ - http://deej.net/sportsman/ Glastar Flyer N866RH - http://deej.net/glastar/ ________________________________________________________________________________
Date: Apr 27, 2014
From: rayj <raymondj(at)frontiernet.net>
Subject: Re: OT: power supply noise
The parts were part of the CNC kit I bought years ago. It is the power supply for the stepper motors. The output is in the neighborhood of 5 amps at 38Vdc The kit specified an UNregulated power supply because of problems associated with back emf from the stepper motors as they decelerate, as I understand it. The motor drivers derive the required 5Vdc internally from the power supply hookup. My main concern was about radiated noise being picked up because they are both inside an old metal computer case. Thanks for the reply. Raymond Julian Kettle River, MN The things we admire in men, kindness and generosity, openness, honesty, understanding and feeling are the concomitants of failure in our system. And those traits we detest, sharpness, greed, acquisitiveness, meanness, egotism and self-interest are the traits of success. And while men admire the quality of the first they love the produce of the second. -John Steinbeck, novelist, Nobel laureate (1902-1968) On 04/26/2014 10:56 PM, Robert L. Nuckolls, III wrote: > > > At 03:10 PM 4/26/2014, you wrote: >> >> Greetings, >> >> I'm considering putting an unregulated power supply (transformers, >> diodes, and a condenser) in an old computer box along with the >> electronics to run several stepper motors for a CNC system. >> >> I'm wondering if noise radiated from the power supply will cause >> problems for the stepper motor drivers. > > Probably not. Do you have these parts already in hand? > Regulated, clean power supplies up to 350W are pretty > cheap on eBay. I seldom build a supply any more, the labor > alone is more than the cost of an off-the-shelf, plug-n-play > device. > > > Bob . . . > > ________________________________________________________________________________
Date: Apr 27, 2014
From: "rv7a.builder" <rv7a.builder(at)yahoo.com>
Subject: Re: Amp meter jumps when transmitting
Thanks for telling me that. I won't try that again. What actually happens t hat could damage the transmitter? Thanks. John=0AOn Sunday, April 27, 2014 8:10 AM, Dj Merrill wrote:=0A =0A--> AeroElectric-List mes sage posted by: Dj Merrill =0A=0AOn 4/27/2014 8:35 AM, rv7a. builder wrote:=0A> I disconnect the antenna coax from back of the radio and hit the PTT =0A> and the amp meter needle stays still.=0A=0AJust as an asi de, it is generally a bad idea to activate a transmitter =0Awith no antenna attached.- There is a good chance of damaging the =0Atransmitter by doin g this.=0A=0Afyi=0A=0A-Dj=0A=0A-- =0ADj Merrill - N1JOV - VP EAA Chapter 87 =0ASportsman 2+2 Builder #7118 N421DJ - http://deej.net/sportsman/=0AGlasta ________________________________________________________________________________
Subject: Re: Amp meter jumps when transmitting
From: Kevin Belue <kdb.rv10(at)gmail.com>
Date: Apr 27, 2014
I saw this problem years ago also. Their electronic manifold pressure gauge has same issue. I replaced the MP with a non-electric version. The rest of my instruments are from an EFIS. No problems with this configuration. K. Belue RV-10 Sent from my iPhone On Apr 27, 2014, at 8:41 AM, "Robert L. Nuckolls, III" wrote: > > At 07:35 AM 4/27/2014, you wrote: > Hello Group, > I have a Vans amp meter installed in my RV-7A. When I activate the PTT on the Garmin GNC300XL the amp meter jumps to full scale. I disconnect the antenna coax from back of the radio and hit the PTT and the amp meter needle stays still. I turn off the radio and bring my handheld into the cockpit and hit the transmit button from 2 feet away and a very slight movement in the amp meter is detected. Bringing the handheld closer to the amp meter will make the needle jump more when the PTT is activated. I think I have determined this to be a RF inference problem. > > I bought a Van's ammeter some years ago to > explore it's vulnerabilities to strong RF. > > See pictures here: > > http://tinyurl.com/mpx8hze > > I discovered that no only was the instrument > devoid of any firewall for RF interference, > it was even directionally sensitive!!! See > pictures > > http://tinyurl.com/m4xlj2u > > http://tinyurl.com/kvo4vwk > > Just waving the hand-held from side to side > would produce behaviors in the pointer error > that mimicked the motion of the antenna. > > > My questions are 1) Will the amp meter eventually be damaged by these extreme movements? > > No > > 2) Is there a fix? > > Probably, but labor intensive and clumsy on > a finished instrument. It would have been FAR > better that the manufacturer demonstrated > a knowledge of DO-160 in the first place. > > 3) Am I alone here or have others experience this phenomena? > > > No doubt there are many others. Since it's > a transient event that affects the display > only while talking, it's a cosmetic behavior. > You might consider just 'living' with it. > > With one caveat . . . make sure that your coax > connectors are good at both ends of the feedline . . . > it wouldn't hurt to do an SWR check. The > phenomenon you're observing MIGHT be the > result of a very RF-Hot cockpit due to loss > of shield ground on either end of the coax. > > But it your antenna system is golden, then > there are no risks for just ignoring it. > > I wrote to Van's about this condition . . . never > heard back from them. It no doubt affects > their entire line of instruments with > electronic signal conditioning. > > > > > > Bob . . . > > > > ________________________________________________________________________________
Date: Apr 27, 2014
From: "Robert L. Nuckolls, III" <nuckolls.bob(at)aeroelectric.com>
Subject: Re: Amp meter jumps when transmitting
At 09:50 AM 4/27/2014, you wrote: > >On 4/27/2014 8:35 AM, rv7a.builder wrote: >>I disconnect the antenna coax from back of the radio and hit the >>PTT and the amp meter needle stays still. > >Just as an aside, it is generally a bad idea to activate a >transmitter with no antenna attached. There is a good chance of >damaging the transmitter by doing this. Back in the days of germanium output transistors (yes Martha, there WERE germanium devices capable of several watts at VHF) it was considered poor form if not instant death to one's output transistors to key a transmitter into an open circuit. Modern output devices are much more rugged. Further, transmitters at-risk for high SWR damage are fitted with automatic shut-down circuits for protection. Bob . . . ________________________________________________________________________________
Date: Apr 27, 2014
From: "Robert L. Nuckolls, III" <nuckolls.bob(at)aeroelectric.com>
Subject: Re: OT: power supply noise
At 09:56 AM 4/27/2014, you wrote: > >The parts were part of the CNC kit I bought years ago. It is the >power supply for the stepper motors. The output is in the >neighborhood of 5 amps at 38Vdc The kit specified an UNregulated >power supply because of problems associated with back emf from the >stepper motors as they decelerate, as I understand it. Steppers don't generate back emf based on motion of the moving parts. For the fastest possible response time, steppers like to be driven from high resistance if not purely constant current power sources. It's all about that t=L/R thingy. The first stepper systems I crafted used banks of power resistors to raise power source impedance. Modern designs will use constant current output stages in the drivers if they're seeking high accuracy positioning along with fast response. >The motor drivers derive the required 5Vdc internally from the power >supply hookup. > >My main concern was about radiated noise being picked up because >they are both inside an old metal computer case. Your application would run quite well from unregulated, full wave rectified AC as long as the 'relaxed' voltage is not so high as to place output transistors at risk in their OFF state. Bob . . . ________________________________________________________________________________
Date: Apr 27, 2014
Subject: Re: Amp meter jumps when transmitting
From: Dj Merrill <deej(at)deej.net>
On 4/27/2014 11:21 AM, rv7a.builder wrote: > Thanks for telling me that. I won't try that again. What actually > happens that could damage the transmitter? Thanks. John Hi John, I am sure someone else on here can explain the technical details better than I, but the simple version is that if the transmitter does not have circuitry built in to protect against it, it can blow some of the parts in the final amplifier stage of the transmitter. Hopefully someone else can chime in with more details of exactly how and why. There is probably a good chance that a modern solid state radio has this protection built in, but unless one knows for sure, there is no reason to take the expensive chance that it doesn't. I'd be more concerned with the older radios, but in general it is just good practice to make sure that you have an antenna or a dummy load attached to the antenna port before keying up the transmitter. My personal choice is to make sure there is an antenna attached before even powering a radio on, regardless of whether I am going to key the transmitter part. -Dj -- Dj Merrill - N1JOV - VP EAA Chapter 87 Sportsman 2+2 Builder #7118 N421DJ - http://deej.net/sportsman/ Glastar Flyer N866RH - http://deej.net/glastar/ ________________________________________________________________________________
Subject: Re: Linking 2 coax cables
From: "Eric M. Jones" <emjones(at)charter.net>
Date: Apr 27, 2014
Install a new cable or connectors. No other reasonable solution exists. Read my article attached. -------- Eric M. Jones www.PerihelionDesign.com 113 Brentwood Drive Southbridge, MA 01550 (508) 764-2072 emjones(at)charter.net Read this topic online here: http://forums.matronics.com/viewtopic.php?p=422475#422475 Attachments: http://forums.matronics.com//files/dabbling_with_electricity_179.pdf ________________________________________________________________________________
Date: Apr 27, 2014
From: Henador Titzoff <henador_titzoff(at)yahoo.com>
Subject: Re: Amp meter jumps when transmitting
Dj, John,=0A=0AHere's what happens when a transmitter hurls RF energy down a transmission line.- First, the transmission line has a characteristic i mpedance.- Impedance is almost like a resistance except it's comprised of a mixture of capacitor reactance, inductive reactance and resistance.- R eactances have currents and voltages that are 90 degrees out of phase.- T his means they don't "burn" up any of the energy, whereas resistance has vo ltage and current in phase and does burn up energy.- Check out the attach ed picture from Wiki, which shows how the capacitance, inductance and resis tances are modeled as lumped components. Ideally, the series and parallel r esistances are zero.- In practice they're not but designers try to minimi ze them to decrease attenuation.=0A=0A=0AWhen this energy traveling down th e transmission line reaches the end, what happens?- If there's a resistor there that is equal to the transmission line's characteristic impedance, t hen ideally all of the energy is dissipated in that resistor as heat, since the voltage and current waves are in phase.- If an antenna that is match ed to the line is there, the power is magically transferred to the atmosphe re, because the antenna's job is to match the impedance of the cable to fre e space impedance, which is 376 Ohms.- It isn't really magic, at least ma thematically, but one has to understand Maxwell's and Gaussian's laws to kn ow how this happens and why free space impedance is 376 Ohms.=0A=0A=0ASo le t's say the end of the transmission line is open or doesn't match the imped ance.- In the open case, the energy has nowhere to go except to be transm itted right back to where it came from.- Let's say the transmitter's tran sistors are generating voltage waves that are 10V peak to peak.- This mea ns that the designer has to choose components that can handle this voltage plus some, say another 10V, in order to provide longevity and reliability. - When the reflected energy arrives at the transmitter output, the two wa ves (one going and one coming) can add up to a worst case number of 20V pea k to peak if the phasing is correct. This comes very close to exceeding the design limitation of the transmitter transistors.- If the designer specs the transistors to less than 20V, then the two added waves' peak voltage c ould very well destroy the transistors and/or other components.- The desi gner could add transorbers or some other devices designed to limit the voltage build up at the transmitter to keep it from malfunctioning, but it 's up to the designer and his design specs.=0A=0AAlso, the transmitter has an impedance of its own, and for maximum power transfer it should equal the line's impedance.- When the reflected wave comes back, if there is a sli ght mismatch between the transmitter and line impedances, there will be som e power dissipation at the transmitter but some of the power (wave) will re flect back to the end of the line.- This means there are several waves on the line, but each reflection decreases in amplitude because of losses alo ng the line and at the ends.- All of these waves have to be added or subt racted to find the real voltages at the ends.=0A=0ASo the bottom line is ca n the transmitter handle the reflected waves such that when they add up, th e end voltage doesn't damage the transmitter components, which of course ha ve maximum voltage and power limits.- I add power in there because power causes heat and components can only handle so much maximum temperature.=0A =0AHope this helps.=0A-=0AHenador Titzoff=0A=0A=0A>______________________ __________=0A> From: Dj Merrill <deej(at)deej.net>=0A>To: aeroelectric-list@ma tronics.com =0A>Sent: Sunday, April 27, 2014 6:52 PM=0A>Subject: Re: AeroEl ectric-List: Amp meter jumps when transmitting=0A> =0A>=0A>--> AeroElectric -List message posted by: Dj Merrill =0A>=0A>On 4/27/2014 11: 21 AM, rv7a.builder wrote:=0A>> Thanks for telling me that. I won't try tha t again. What actually =0A>> happens that could damage the transmitter? Tha nks. John=0A>=0A>Hi John,=0A>- - I am sure someone else on here can ex plain the technical details =0A>better than I, but the simple version is th at if the transmitter does =0A>not have circuitry built in to protect again st it, it can blow some of =0A>the parts in the final amplifier stage of th e transmitter. Hopefully =0A>someone else can chime in with more details of exactly how and why.=0A>=0A>- - There is probably a good chance that a modern solid state radio has =0A>this protection built in, but unless one knows for sure, there is no =0A>reason to take the expensive chance that i t doesn't.- I'd be more =0A>concerned with the older radios, but in gener al it is just good practice =0A>to make sure=0A that you have an antenna or a dummy load attached to the =0A>antenna port before keying up the transmi tter.- My personal choice is to =0A>make sure there is an antenna attache d before even powering a radio on, =0A>regardless of whether I am going to key the transmitter part.=0A>=0A>-Dj=0A>=0A>-- =0A>Dj Merrill - N1JOV - VP EAA Chapter 87=0A>Sportsman 2+2 Builder #7118 N421DJ - http://deej.net/spor ========================0A> =0A>=0A>=0A>=0A>=0A> ________________________________________________________________________________
Date: Apr 27, 2014
From: "Robert L. Nuckolls, III" <nuckolls.bob(at)aeroelectric.com>
Subject: Re: Amp meter jumps when transmitting
>So the bottom line is can the transmitter handle the reflected waves >such that when they add up, the end voltage doesn't damage the >transmitter components, which of course have maximum voltage and >power limits. I add power in there because power causes heat and >components can only handle so much maximum temperature. I'm aware of no supplier to aviation that does not consider the rare but possible failure of a coax connection . . . it's common practice to build in protection, robustness or a combination of the two. Bob . . . ________________________________________________________________________________
Date: Apr 27, 2014
From: Henador Titzoff <henador_titzoff(at)yahoo.com>
Subject: Re: Amp meter jumps when transmitting
This is very true, Bob, but I would also like to add that the possibility o f an open in the connectors and coax is more common than "rare."- Most of these failures occur during build, which are accidental, corrosion or fail ure to tighten down connectors correctly.=0A=0A=0AWhile aviation suppliers will consider the rare but possible failure you mention below, there are se veral scenarios where this is not true.- One scenario is the supplier rec eiving counterfeit parts that do not meet specs.- Another scenario is a s hady buyer ordering the wrong parts and a faulty lot is manufactured and sh ipped.- Manufactured lots vary from lot to lot, and some lots may sneak t hrough that do not meet specifications but work well until "something happe ns."- In particular, this business of counterfeit parts has been plaguing manufacturing for at least two decades now.=0A=0A-=0AHenador Titzoff=0A =0A=0A>________________________________=0A> From: "Robert L. Nuckolls, III" =0A>To: aeroelectric-list(at)matronics.com =0A >Sent: Sunday, April 27, 2014 8:44 PM=0A>Subject: Re: AeroElectric-List: Am p meter jumps when transmitting=0A> =0A>=0A>--> AeroElectric-List message p osted by: "Robert L. Nuckolls, III" =0A>=0A> =0A>>So the bottom line is can the transmitter handle the reflected waves =0A>>such that when they add up, the end voltage doesn't damage the =0A>>tr ansmitter components, which of course have maximum voltage and =0A>>power l imits.- I add power in there because power causes heat and =0A>>component s can only handle so much maximum temperature.=0A>=0A>- - I'm aware of no supplier to aviation that does not=0A>- - consider the rare but poss ible failure of a coax=0A>- - connection . . . it's common practice to build=0A>- - in protection, robustness or a combination of the=0A>- ==================0A>=0A>=0A>=0A>=0A>=0A> ________________________________________________________________________________
Date: Apr 27, 2014
From: "Robert L. Nuckolls, III" <nuckolls.bob(at)aeroelectric.com>
Subject: Re: Amp meter jumps when transmitting
> >Manufactured lots vary from lot to lot, and some lots may sneak > through that do not meet specifications but work well until > "something happens." In >particular, this business of counterfeit > parts has been plaguing manufacturing for at least two decades now. Forgive me my friend but we're reaching WAAaaayyyy down into the worry bucket with this line of reasoning. Once you open that door . . . where do the caveats end? Cracked magneto rotors? Contaminated fuel? Bogus rivets? The world is indeed fraught with risks. Nonetheless, the vast majority of misadventures in every venue have nothing to do with parts that fail to meet published requirements. Adding such worries to the knowledge base is potential misdirection of valuable attention from much greater risks. Bob . . . ________________________________________________________________________________
Date: Apr 28, 2014
From: GTH <gilles.thesee(at)free.fr>
Subject: Time delay, flicker filter etc.
Hi Bob and all, It's been a long time since my last post on this knowledgeable list. Our two battery / ABMM / Rotax 914 project has been flying for ten happy years now, except for one little problem that has kept nagging at us from the beginning. The auxiliary battery management module (ABMM) works as advertised, connecting and disconnecting the aux battery contactor following the main bus voltage. But at idle, the voltage undergoes rapid fluctuations, leading to frigthening contactor chattering fits. As a workaround we ended connecting the aux battery manually. I was wondering if an easy way of introducing time delay in the voltage sensing side *or* actuating side of the ABMM could be devised ? A time constant of 2 second would be adequate I suppose. Could some variation of the fuel level flicker filter do ? Thanks in advance for your suggestions, and keep on the good job ! Best regards, -- Gilles http://contrails.free.fr ________________________________________________________________________________
Date: Apr 28, 2014
From: Henador Titzoff <henador_titzoff(at)yahoo.com>
Subject: Re: Amp meter jumps when transmitting
You're right that I'm going down a road fraught with all sorts of relativel y low risk items.- Just trying to explain why an open transmission line m ight kill a transmitter that's otherwise designed to survive it.- I hope my explanation of why the voltage could theoretically double at the transmi tter helped explain to some how this phenomenon happens.=0A=0AHenador Titzo ff=0A=0A=0A>________________________________=0A> From: "Robert L. Nuckolls, III" =0A>To: aeroelectric-list(at)matronics.co m =0A>Sent: Sunday, April 27, 2014 10:50 PM=0A>Subject: Re: AeroElectric-Li st: Amp meter jumps when transmitting=0A> =0A>=0A>--> AeroElectric-List mes sage posted by: "Robert L. Nuckolls, III" =0A>=0A>=0A>> >Manufactured lots vary from lot to lot, and some lots may sn eak =0A>> through that do not meet specifications but work well until =0A>> "something happens."- In >particular, this business of counterfeit =0A>> parts has been plaguing manufacturing for at least two decades now.=0A>=0A >- - Forgive me my friend but we're reaching WAAaaayyyy down into=0A> - - the worry bucket with this line of reasoning. Once=0A>- - you o pen that door . . . where do the caveats end?=0A>- - Cracked magneto ro tors? Contaminated fuel? Bogus rivets?=0A>- - The world is indeed fraug ht with risks. Nonetheless,=0A>- - the vast majority of misadventures i n every=0A>- - venue have nothing to do with parts that fail=0A>- - to meet published requirements. Adding such=0A>- - worries to the know ledge base is potential misdirection=0A>- - of valuable attention from ======================0A>=0A>=0A> =0A>=0A>=0A> ________________________________________________________________________________
Date: Apr 28, 2014
Subject: Re: Amp meter jumps when transmitting
From: Dj Merrill <deej(at)deej.net>
On 04/28/2014 04:22 PM, Henador Titzoff wrote: > You're right that I'm going down a road fraught with all sorts of > relatively low risk items. Just trying to explain why an open > transmission line might kill a transmitter that's otherwise designed to > survive it. I hope my explanation of why the voltage could > theoretically double at the transmitter helped explain to some how this > phenomenon happens. > > Henador Titzoff Your explanation helped me, and matched my limited understanding of how it works, as well as allowing me to learn more. Thank you! :-) -Dj -- Dj Merrill - N1JOV - VP EAA Chapter 87 Sportsman 2+2 Builder #7118 N421DJ - http://deej.net/sportsman/ Glastar Flyer N866RH - http://deej.net/glastar/ ________________________________________________________________________________
Date: Apr 28, 2014
From: "Robert L. Nuckolls, III" <nuckolls.bob(at)aeroelectric.com>
Subject: Re: Time delay, flicker filter etc.
> >A time constant of 2 second would be adequate I suppose. >Could some variation of the fuel level flicker filter do ? > >Thanks in advance for your suggestions, and keep on the good job Referring to the assembly details in . . . http://tinyurl.com/nxmo3us That comparator only has about 0.5% hysteresis . . . try reducing the value of R108 down to 100K which will raise the hysteresis by a factor of ~5x Bob . . . ________________________________________________________________________________
From: "Alvin Voigt" <iamgodlisten(at)hotmail.com>
Subject: ATS Crimper
Date: Apr 29, 2014
ATS has a crimper on sale for $37.50 with additional BNC die sets for $14.95. The assortment of terminals and splices included in the kit are not specifically identified as PDIG, so they my be un-useful for flying machines. I have been using a crimper for many years that I purchased from Bob Knuckles that has been quite satisfactory, but does not have replaceable die sets. http://www.aircraft-tool.com/shop/detail.aspx?PRODUCT_ID=E500-037&utm_s ource=ECLIPSE+APRIL&utm_campaign=ECLIPSE+APRIL&utm_medium=email Alvin Voigt, Ashford, WA ________________________________________________________________________________
From: "Alvin Voigt" <iamgodlisten(at)hotmail.com>
Subject: Fw: ATS Crimper
Date: Apr 29, 2014
Sorry, I meant PIDG terminals and Bob Nuckolls. Past my bed time, I suppose. Alvin From: Alvin Voigt Sent: Tuesday, April 29, 2014 1:09 AM Subject: ATS Crimper ATS has a crimper on sale for $37.50 with additional BNC die sets for $14.95. The assortment of terminals and splices included in the kit are not specifically identified as PDIG, so they my be un-useful for flying machines. I have been using a crimper for many years that I purchased from Bob Knuckles that has been quite satisfactory, but does not have replaceable die sets. http://www.aircraft-tool.com/shop/detail.aspx?PRODUCT_ID=E500-037&utm_s ource=ECLIPSE+APRIL&utm_campaign=ECLIPSE+APRIL&utm_medium=email Alvin Voigt, Ashford, WA ________________________________________________________________________________
From: "Bill S" <docyukon(at)ptcnet.net>
Subject: lm7321 substatute
Date: Apr 29, 2014
Bob Can you recomend a substatute for a lm7321 opamp, in a din package? The sot23 package is just tooooooo small for me to work with. Thanks Bill S. ________________________________________________________________________________
Subject: Battery and master contactor - ok to mount horizontally?
From: Sacha <uuccio(at)gmail.com>
Date: Apr 30, 2014
Quick question about Battery and contactor installation position: I am moving my battery aft and am planning to make an installation similar to the example on Bob's website at http://www.aeroelectric.com/articles/Battery_Grounds/Battery_Grounds.html Except that instead of having the (SVLA) battery upright I want it on its side. Is that ok? And is it ok to mount the master and starter contactors horizontally (I'm planning to use rivnuts and bolt them to a horizontal sheet of stainless steel) instead of using L-shaped tabs to mount them vertically as in the picture? ________________________________________________________________________________
Date: Apr 29, 2014
From: Jeff Luckey <jluckey(at)pacbell.net>
Subject: Re: Battery and master contactor - ok to mount horizontally?
Mounting the battery on its side and the contactors horizontally is just fi ne.- It won't affect the operation of either device.=0A=0A=0A=0A=0A______ __________________________=0A From: Sacha <uuccio(at)gmail.com>=0ATo: aeroelec tric-list =0ASent: Tuesday, April 29, 201 4 6:41 PM=0ASubject: AeroElectric-List: Battery and master contactor - ok t ha =0A=0AQuick question about Battery and contactor insta llation position:=0A=0AI am moving my battery aft and am planning to make a n installation similar to the example on Bob's website at http://www.aeroel ectric.com/articles/Battery_Grounds/Battery_Grounds.html =0AExcept that ins tead of having the (SVLA) battery upright I want it on its side. Is that ok ? And is it ok to mount the master and starter contactors horizontally (I'm planning to use rivnuts and bolt them to a horizontal sheet of stainless s teel) instead of using L-shaped tabs to mount them vertically as in the pic = ________________________________________________________________________________
Date: Apr 30, 2014
From: "Robert L. Nuckolls, III" <nuckolls.bob(at)aeroelectric.com>
Subject: Re: lm7321 substatute
At 12:50 PM 4/29/2014, you wrote: > >Bob Can you recomend a substatute for a lm7321 opamp, in a din >package? The sot23 package is just tooooooo small for me to work >with. Thanks Bill S. What's the application? Can you share a schematic along with a description of function? There are probably 100+ suitable substitutes with rare exception . . . I don't want to offer substitute that stumbles over an exception. The LM7321 is a fine example of the current state of op-amp art. Low voltage performance, rail-to-rail input and outputs, lots of drive capability, etc. http://tinyurl.com/otlg7je However, depending on requirements for the circuit you're crafting, you may NEED few if any of these stellar features. Bob . . . ________________________________________________________________________________
From: "Bill S" <docyukon(at)ptcnet.net>
Subject: Re: lm7321 substatute
Date: Apr 30, 2014
At 10:59 AM 12/22/2011, you wrote: >I am wanting to parllel two Ray Allen RP3 led position indicators useing >only one POS5 position sensor. RAC said that thay wont work just parelling >them and that I need to add another position sensor or a switch which I >would rather not do. Can anyone sudjest an electrical ckt. that would work >for this? Thanks Bill S. > You need a 'buffer-amplifier' between the position feedback potentiometer and ONE of the two indicators. The problem with paralleling the two indicators arises from the fact that they're not a 'high impedance' voltmeter. The system is calibrated for one pot driving one indicator. Adding a second indicator doubles the load on the position signal from the potentiometer. The 'fix' is to convert one of the indicators into a high-impedance voltmeter. You need an operational amplifier with rail-to-rail inputs and outputs. A device like the LM7321 would probably work. http://search.digikey.com/us/en/products/LM7321MF%2FNOPB/LM7321MFCT-ND/1878646 Adding this device to the second indicator prevents it from loading the potentiometer. You need to fabricate something like this . . . http://aeroelectric.com/Pictures/Schematics/Ray-Allen_Dual_Indicators.pdf It could be fabricated on an etched circuit board that would fit inside a d-sub connector back shell. Bob . . . >>Bob Can you recomend a substatute for a lm7321 opamp, in a din package? >>The sot23 package is just tooooooo small for me to work with. Thanks >>Bill S. > > What's the application? Can you share a schematic > along with a description of function? There are probably > 100+ suitable substitutes with rare exception . . . I > don't want to offer substitute that stumbles over > an exception. > > The LM7321 is a fine example of the current state > of op-amp art. Low voltage performance, rail-to-rail > input and outputs, lots of drive capability, etc. > > http://tinyurl.com/otlg7je > > However, depending on requirements for the circuit > you're crafting, you may NEED few if any of these > stellar features. > > > Bob . . . > > > ----- > No virus found in this message. > Checked by AVG - www.avg.com > ________________________________________________________________________________
Date: Apr 30, 2014
From: John <jrevens(at)comcast.net>
Subject: Re: Battery and master contactor - ok to mount horizontally?
Mounting position is definitely a consideration with some contactors. There are units that are designed for horizontal mounting, or are position insensitive, but there are others that are not. The Series 70 White-Rogers units, that have been very commonly used in aircraft for decades are position sensitive. Here is a link to some info: http://www.emersonclimate.com/Documents/White-Rodgers/sell_sheets/R-4003web.pdf <http://www.emersonclimate.com/Documents/White-Rodgers/sell_sheets/R-4003web.pdf> This literature recommends dome, or cap, down. Depending on whether the unit is used as a master or start "solenoid", some recommend mounting dome up These solenoid/contactors will seem to work OK either way, but there is more potential wear and possibility of binding or improper operation if they are operated horizontally. /John / ________________________________________________________________________________
Date: Apr 30, 2014
From: "Robert L. Nuckolls, III" <nuckolls.bob(at)aeroelectric.com>
Subject: Re: Battery and master contactor - ok to mount horizontally?
At 11:44 AM 4/30/2014, you wrote: Mounting position is definitely a consideration with some contactors. There are units that are designed for horizontal mounting, or are position insensitive, but there are others that are not. The Series 70 White-Rogers units, that have been very commonly used in aircraft for decades are position sensitive. Here is a link to some info: http://www.emersonclimate.com/Documents/White-Rodgers/sell_sheets/R-4003web.pdf This literature recommends dome, or cap, down. Depending on whether the unit is used as a master or start "solenoid", some recommend mounting dome up These solenoid/contactors will seem to work OK either way, but there is more potential wear and possibility of binding or improper operation if they are operated horizontally. We've had some discussion about both alleged and demonstrated position sensitivity of these contactors here on the List . . . here is but one of several dialogs . . . http://tinyurl.com/p8uv67n In other discussions we deduced that the W/R Type 70 is not well suited to starter contactor service even in the intermittent version. There are similarly priced contactors optimized for starter contactor service. http://tinyurl.com/n9sql7g http://tinyurl.com/o5turon As a battery contactor mounted in a location certain to be 'drip free', a horizontal mounting doesn't give rise to great concerns. In any case, concerns for in-flight g-loading of contactors has no demonstrable foundation in physics or practice. Bob . . . ________________________________________________________________________________
Date: Apr 30, 2014
From: "Robert L. Nuckolls, III" <nuckolls.bob(at)aeroelectric.com>
Subject: Re: lm7321 substatute
At 10:27 AM 4/30/2014, you wrote: > OOOPS! The 7321 was my selection . . . not sure why I went with SOT23 only except that my head was probably thinking about packaging in the dsub housing . . . Refresh my memory, we ARE talking about a 5v system? This op amp would do the job http://tinyurl.com/ol9jm4v but it's limited to 7.5v max supply voltage. It IS in stock at Digikey at 0.88 each. Bob . . . ________________________________________________________________________________
Date: Apr 30, 2014
From: "Robert L. Nuckolls, III" <nuckolls.bob(at)aeroelectric.com>
Subject: Re: Battery and master contactor - ok to mount
horizontally? At 11:55 AM 4/30/2014, you wrote: > > >Hi, Jeff, >A word of caution for rivnuts: Be sure that you use rivnuts that >are HARDER than the substrate, in your case stainless >steel. Rivnuts have a bad habit of loosening in their holes, making >the bolt nigh onto impossible to unscrew. Don't ask me how I know! > >This might be even worse if you use aluminum rivnuts, as there will >be corrosion between the metals, which will act as a lubricant. Good catch Jim . . . rivnuts are problematic If he cannot get at the back side to install a real 10-32 nutplate, I think I'd fabricate a scab-on with nutplates to mount the contactor. The scab plate needs to have sufficient footprint so that pop-rivets used to fasten it to the aircraft surface are not under the contactor's mounting feet. I would then cut clearance holes in the aircraft surface for the nutplates and pop- rivet the scab with a half dozen or so aluminum rivets. This is the sure-bet prophylactic against that gut wrenching experience of having the rivnut spin in the hole while trying to dismount the contactor . . . just hate it when that happens! Bob . . . ________________________________________________________________________________
Date: Apr 30, 2014
From: "Robert L. Nuckolls, III" <nuckolls.bob(at)aeroelectric.com>
Subject: Re: Was Battery and master contactor - ok to mount
horizontally? Now about Rivnuts At 02:37 PM 4/30/2014, you wrote: >(Please pardon the hijacking of this thread...) > > >Jim, > >I wasn't the original poster on this thread. I think it was >Sacha. However, speaking of Rivnuts... > >I've experienced some of the issues you are talking about. The last >few times I've used Rivnuts, I put an internal-tooth lockwasher on >the back side of the Rivnut before squeezing it, so that it gets >clamped between the work surface and the bulge formed by squeezing. Nice vaccination against spinning rivnut syndrome . . . but if you could get at the back side, why not nutplates? Bob . . . ________________________________________________________________________________
From: "Bill S" <docyukon(at)ptcnet.net>
Subject: Re: lm7321 substatute
Date: Apr 30, 2014
The opamp should be rated at least 15v min. ----- Original Message ----- From: "Robert L. Nuckolls, III" <nuckolls.bob(at)aeroelectric.com> Sent: Wednesday, April 30, 2014 2:19 PM Subject: Re: AeroElectric-List: lm7321 substatute > > > At 10:27 AM 4/30/2014, you wrote: >> > > OOOPS! The 7321 was my selection . . . not sure > why I went with SOT23 only except that my head > was probably thinking about packaging in the > dsub housing . . . > > Refresh my memory, we ARE talking about a 5v > system? This op amp would do the job > > http://tinyurl.com/ol9jm4v > > but it's limited to 7.5v max supply voltage. > It IS in stock at Digikey at 0.88 each. > > > Bob . . . > > > ----- > No virus found in this message. > Checked by AVG - www.avg.com > ________________________________________________________________________________
Date: Apr 30, 2014
From: Jeff Luckey <jluckey(at)pacbell.net>
Subject: Re: Was Battery and master contactor - ok to mount
horizontally? Now about Rivnuts =0A=0A=0A=0A________________________________=0A From: "Robert L. Nuckolls, III" =0ATo: aeroelectric-list(at)matronics.com =0ASent: Wednesday, April 30, 2014 12:42 PM=0ASubject: Re: AeroElectric-Lis t: Was Battery and master contactor - ok to mount horizontally? Now about lls, III" =0A=0AAt 02:37 PM 4/30/2014, you w rote:=0A>(Please pardon the hijacking of this thread...)=0A>=0A>=0A>Jim,=0A >=0A>I wasn't the original poster on this thread.- I think it was =0A>Sac ha.- However, speaking of Rivnuts...=0A>=0A>I've experienced some of the issues you are talking about.- The last =0A>few times I've used Rivnuts, I put an internal-tooth lockwasher on =0A>the back side of the Rivnut befor e squeezing it, so that it gets =0A>clamped between the work surface and th e bulge formed by squeezing.=0A=0A- Nice vaccination against spinning ri vnut syndrome=0A- . . . but if you could get at the back side, why not =0A- nutplates?=0A=0AAgree - but that's the big IF - sometimes you just don't have enough space or access to use nutplates.- (It's just another a rrow in the "How am I gonna fix this GD thing" quiver.)-- -JL=0A=0A=0A = ________________________________________________________________________________
Date: Apr 30, 2014
From: "Robert L. Nuckolls, III" <nuckolls.bob(at)aeroelectric.com>
Subject: Re: lm7321 substatute
At 03:13 PM 4/30/2014, you wrote: > > >The opamp should be rated at least 15v min. Okay, this one should work http://tinyurl.com/llsm37s Let me know how this works for you . . . I'll revise my document on the website. I could also whack out a little ecb that would sit between the solder-cups on a d-sub and make it a whole lot easier to package . . . Bob . . . ________________________________________________________________________________
Subject: Re: Was Battery and master contactor - ok to mount
horizontally? Now about Rivnuts
From: Sacha <uuccio(at)gmail.com>
Date: Apr 30, 2014
> I wasn't the original poster on this thread. I think it was > >Sacha. No need to apologize for "high jacking". I've gathered lots of food for thought which I will now attempt to digest in order to design the battery and contactor mount. Thank you all. I wanted to use rivnuts because they would allow me to permanently fix the steel plate and then bolt things onto it. The rivnuts I was going to use were steel ones. Of course I wasn't aware of the issues with them coming loose. I'm also having second thoughts about using a stainless steel base as I just weighed it and it's 1kg which seems like overkill. Sacha ________________________________________________________________________________
Date: Apr 30, 2014
From: "Robert L. Nuckolls, III" <nuckolls.bob(at)aeroelectric.com>
Subject: Re: Was Battery and master contactor - ok to mount
horizontally? Now about Rivnuts > > >I'm also having second thoughts about using a stainless steel base >as I just weighed it and it's 1kg which seems like overkill. The SVLA battery is essentially leak proof. They cannot exude stuff that eats on your airplane. A tray with 1/2" sides to capture the battery's footprint is quite sufficient for a base. A couple of nylon straps, 1" wide with velcro closures are good for holding it down in the tray. You only need to grab the battery with enough security to withstand a 10g load. For an 18 Ah battery that's 200 pounds or less. A pair of 1" with velcro closures are good for MUCH more than this. Bob . . . ________________________________________________________________________________
Date: Apr 30, 2014
From: Kelly McMullen <kellym(at)aviating.com>
Subject: Re: Was Battery and master contactor - ok to mount
horizontally? Now about Rivnuts If riveting a nut plate isn't possible, a Clickbond nutplate is next best thing, unless you cannot get to backside at all. On 4/30/2014 12:42 PM, Robert L. Nuckolls, III wrote: > > > At 02:37 PM 4/30/2014, you wrote: >> (Please pardon the hijacking of this thread...) >> >> >> Jim, >> >> I wasn't the original poster on this thread. I think it was Sacha. >> However, speaking of Rivnuts... >> >> I've experienced some of the issues you are talking about. The last >> few times I've used Rivnuts, I put an internal-tooth lockwasher on >> the back side of the Rivnut before squeezing it, so that it gets >> clamped between the work surface and the bulge formed by squeezing. > > Nice vaccination against spinning rivnut syndrome > . . . but if you could get at the back side, why not > nutplates? > > > Bob . . . > > ________________________________________________________________________________
Date: Apr 30, 2014
From: "Robert L. Nuckolls, III" <nuckolls.bob(at)aeroelectric.com>
Subject: Re: Was Battery and master contactor - ok to mount
horizontally? Now about Rivnuts At 07:59 PM 4/30/2014, you wrote: > >If riveting a nut plate isn't possible, a Clickbond nutplate is next >best thing, unless you cannot get to backside at all. I would have suggested that . . . did an article in Kitplanes a few months ago that spoke to DIY bond-studs. All versions of this mounting mode disrupt the otherwise smooth mounting surface under the contactor's mounting feet. Bob . . . ________________________________________________________________________________
Date: May 01, 2014
From: "Robert L. Nuckolls, III" <nuckolls.bob(at)aeroelectric.com>
Subject: Re: lm7321 substatute
At 03:13 PM 4/30/2014, you wrote: Have you ordered any op-amps yet? I've got an order going into Digikey this weekend. I could add some to the list if you're in no hurry . . . Bob . . . ________________________________________________________________________________
Subject: Re: lm7321 substatute
From: "Eric M. Jones" <emjones(at)charter.net>
Date: May 01, 2014
> Bob Can you recomend a substatute for a lm7321 opamp, in a din package? The sot23 package is just tooooooo small for me to work with. Thanks Bill S. I want to propose another solution....c'mon bucky! The Swiss could build a watch between the legs of the SOT-23. I have circuits where the SOT-23 parts are the BIG ones. Seriously, soldering tiny parts is a skill that is not hard to master, but you have to start with--ideally, a nice stereo lab microscope of modest power-- but any good magnifier that you don't have to hold with a hand will do. Then use a fine-tip soldering iron, fine tweezers and some fine-gauge solder. This is ultimately pretty easy to do. Many electrowhizzies are not available in bigger packages. Everything is shrinking. To work on tiny stuff requires the right tools. Good luck. -------- Eric M. Jones www.PerihelionDesign.com 113 Brentwood Drive Southbridge, MA 01550 (508) 764-2072 emjones(at)charter.net Read this topic online here: http://forums.matronics.com/viewtopic.php?p=422661#422661 ________________________________________________________________________________
Subject: Ray Allen RP3 led position indicators
From: "Eric M. Jones" <emjones(at)charter.net>
Date: May 01, 2014
> I am wanting to parllel two Ray Allen RP3 led position indicators useing > only one POS5 position sensor. RAC said that thay wont work just parelling > them and that I need to add another position sensor or a switch which I > would rather not do. Can anyone suggest an electrical ckt. that would work > for this? Thanks Bill S. I predict that trying to convert the potentiometers to drive the RP3 position indicators if fraught with peril. I have done it and abandoned the idea in favor of driving the trim box potentiometer with a higher voltage (regulated 12V) and doing the conversion at the RP3 with a simple 1:12 voltage divider (another pot). We have previously mentioned on this site that the problem with the RP3 is that it expects a MAXIMUM input voltage of 1.2V and each digit is therefore 0.12 volts, which is below the noise level for several lower of the LEDs. Years ago. I figured that the real problem was that push buttons had no memory for position, so I built the True Servo Control for Mac/RAC trim boxes, TSCMR. You turn a knob...and that's where the trim box sets the trim. Period. The indicator is the pointer on the knob. You can still use an LED indicator if you like, but it is not necessary since the knob pointer is the indicator. I don't supply the knobs or pots. My "Cessna-Type Trim Wheel" used to drive the TSCMR, but anyone can make a suitable knob and pot. You could even put one inside your control stick. See attached. -------- Eric M. Jones www.PerihelionDesign.com 113 Brentwood Drive Southbridge, MA 01550 (508) 764-2072 emjones(at)charter.net Read this topic online here: http://forums.matronics.com/viewtopic.php?p=422664#422664 Attachments: http://forums.matronics.com//files/tscmr_installation_manual_101.pdf ________________________________________________________________________________
Date: May 01, 2014
From: rayj <raymondj(at)frontiernet.net>
Subject: Re: Was Battery and master contactor - ok to mount
horizontally? Now about Rivnuts Another solution to rivnut rotation I saw suggested somewhere is to file 2 points into the hole, making it kind of a diamond shape. Theoretically, the bulge expands out in to the diamond shape and prevents rotation. Haven't actually tried it, but it seems worth investigating. Raymond Julian Kettle River, MN The things we admire in men, kindness and generosity, openness, honesty, understanding and feeling are the concomitants of failure in our system. And those traits we detest, sharpness, greed, acquisitiveness, meanness, egotism and self-interest are the traits of success. And while men admire the quality of the first they love the produce of the second. -John Steinbeck, novelist, Nobel laureate (1902-1968) On 04/30/2014 03:53 PM, Sacha wrote: > > >> I wasn't the original poster on this thread. I think it was >>> Sacha. > > No need to apologize for "high jacking". I've gathered lots of food for thought which I will now attempt to digest in order to design the battery and contactor mount. Thank you all. > > I wanted to use rivnuts because they would allow me to permanently fix the steel plate and then bolt things onto it. The rivnuts I was going to use were steel ones. Of course I wasn't aware of the issues with them coming loose. > > I'm also having second thoughts about using a stainless steel base as I just weighed it and it's 1kg which seems like overkill. > > Sacha > > ________________________________________________________________________________
Date: May 01, 2014
From: koh leh <kohleh(at)yahoo.com>
Subject: Re: lm7321 substitute
Earlier in the thread I saw a mention of a rail to rail input requirement. If that is still needed, note the common mode input voltage range of Vdd - 1.35 volts maximum for the TLV27x. Seems to indicate the input range is not really rail to rail and I'd expect the output to swing hard to the full on supply voltage whenever the input is within 1.35v of the supply voltage. Ken On 30/04/2014 4:35 PM, Robert L. Nuckolls, III wrote: > > > At 03:13 PM 4/30/2014, you wrote: >> >> >> The opamp should be rated at least 15v min. > > Okay, this one should work > > http://tinyurl.com/llsm37s > > Let me know how this works for you . . . I'll > revise my document on the website. I could > also whack out a little ecb that would sit > between the solder-cups on a d-sub and make > it a whole lot easier to package . . . ________________________________________________________________________________
From: BobsV35B(at)aol.com
Date: May 01, 2014
Subject: Re: Was Battery and master contactor - ok to mount
hori... Good Morning Raymond, There are keyed RivNuts and there is a tool to make the slot for the key. I have had good results just carefully filing a slot in which the key will fit. Works great! Happy Skies, Old Bob In a message dated 5/1/2014 11:13:46 A.M. Central Daylight Time, raymondj(at)frontiernet.net writes: --> AeroElectric-List message posted by: rayj Another solution to rivnut rotation I saw suggested somewhere is to file 2 points into the hole, making it kind of a diamond shape. Theoretically, the bulge expands out in to the diamond shape and prevents rotation. Haven't actually tried it, but it seems worth investigating. Raymond Julian Kettle River, MN The things we admire in men, kindness and generosity, openness, honesty, understanding and feeling are the concomitants of failure in our system. And those traits we detest, sharpness, greed, acquisitiveness, meanness, egotism and self-interest are the traits of success. And while men admire the quality of the first they love the produce of the second. -John Steinbeck, novelist, Nobel laureate (1902-1968) On 04/30/2014 03:53 PM, Sacha wrote: > > >> I wasn't the original poster on this thread. I think it was >>> Sacha. > > No need to apologize for "high jacking". I've gathered lots of food for thought which I will now attempt to digest in order to design the battery and contactor mount. Thank you all. > > I wanted to use rivnuts because they would allow me to permanently fix the steel plate and then bolt things onto it. The rivnuts I was going to use were steel ones. Of course I wasn't aware of the issues with them coming loose. > > I'm also having second thoughts about using a stainless steel base as I just weighed it and it's 1kg which seems like overkill. > > Sacha > > ________________________________________________________________________________
Date: May 02, 2014
From: Alan Barnett <alansbarnett(at)verizon.net>
Subject: Garmin GNS430 wiring
I'm installing a Garmin GNS 430 in my COZY III. I bought the unit from an avionics shop. Since I want to install it myself, I requested all the wires, connectors, etc required. Included were three fuses (2a, 5a and 10a), one DPDT switch, and four SPDT switches. I assume the 10a fuse is for the COM power and the 5a fuse is for the main power, and that the DPDT switch is for both power circuits. Is this reasonable? What are the 2a fuse and the SPDT switches for? Are there really so many switches in a standard installation? Unless there is a compelling reason, I intend to leave out the switches. Thanks for your comments. Alan ________________________________________________________________________________
From: BobsV35B(at)aol.com
Date: May 02, 2014
Subject: Re: Garmin GNS430 wiring
Good Afternoon Alan, You should ask your supplier for a copy of the current install manual. The dealer can either run off the copy himself or give you the address so you can download it.. I do not think he is supposed to give you the address, but there is nothing at all wrong with his supplying you with the most recent install manual available. I think the most current one is from about March of 2013. Is your unit a 430 or a 430W? The install manual will guide you through the various possibilities and help you make decisions based on how you wish to use the unit. Happy Skies, Old Bob In a message dated 5/2/2014 2:52:04 P.M. Central Daylight Time, alansbarnett(at)verizon.net writes: --> AeroElectric-List message posted by: Alan Barnett I'm installing a Garmin GNS 430 in my COZY III. I bought the unit from an avionics shop. Since I want to install it myself, I requested all the wires, connectors, etc required. Included were three fuses (2a, 5a and 10a), one DPDT switch, and four SPDT switches. I assume the 10a fuse is for the COM power and the 5a fuse is for the main power, and that the DPDT switch is for both power circuits. Is this reasonable? What are the 2a fuse and the SPDT switches for? Are there really so many switches in a standard installation? Unless there is a compelling reason, I intend to leave out the switches. Thanks for your comments. Alan ________________________________________________________________________________
Subject: Re: Garmin GNS430 wiring
From: Justin Jones <jmjones2000(at)mindspring.com>
Date: May 02, 2014
I purchased a used one and am in the same boat. I was able to download the G armin GNC 430 manual online by searching google for it. I will try to post t he link when I get back to my computer for those interested. Justin On May 2, 2014, at 15:33, BobsV35B(at)aol.com wrote: > Good Afternoon Alan, > > You should ask your supplier for a copy of the current install manual. The dealer can either run off the copy himself or give you the address so you c an download it.. I do not think he is supposed to give you the address, but t here is nothing at all wrong with his supplying you with the most recent ins tall manual available. I think the most current one is from about March of 2 013. > > Is your unit a 430 or a 430W? The install manual will guide you through t he various possibilities and help you make decisions based on how you wish t o use the unit. > > Happy Skies, > > Old Bob > > In a message dated 5/2/2014 2:52:04 P.M. Central Daylight Time, alansbarne tt(at)verizon.net writes: n.net> > > I'm installing a Garmin GNS 430 in my COZY III. I bought the unit from > an avionics shop. Since I want to install it myself, I requested all > the wires, connectors, etc required. Included were three fuses (2a, 5a > and 10a), one DPDT switch, and four SPDT switches. > > I assume the 10a fuse is for the COM power and the 5a fuse is for the > main power, and that the DPDT switch is for both power circuits. Is thi s > reasonable? > > What are the 2a fuse and the SPDT switches for? > > Are there really so many switches in a standard installation? Unless > there is a compelling reason, I intend to leave out the switches. > > Thanks for your comments. > > Alan > > > ========================== ========= ========================== ========= ========================== ========= ========================== ========= > ________________________________________________________________________________
Subject: Re: Garmin GNS430 wiring
From: Sacha <uuccio(at)gmail.com>
Date: May 02, 2014
I installed mine without switches and I believe this is best unless you are so restricted with battery capacity that you'd need to turn off the GPS during an alternator failure. I've never seen an installation with more than one external switch (usually the avionics switch, but keep in mind this is a useless carryover from the past, not to mention a potentially dangerous single point of failure). > On May 2, 2014, at 21:36, Alan Barnett wrote: > > Unless there is a compelling reason, I intend to leave out the switches. ________________________________________________________________________________
Subject: Re: How do I prevent starter engaged LED from failing?
From: "eschlanser" <eschlanser(at)yahoo.com>
Date: May 02, 2014
Bob et group, In wiring a starter engaged annunciator, I am using an off-the-shelf LED fixture with an appropriate resistor already integrated into it. The resistor is actually soldered directly to the LED lead inside of the fixture. The two attached diagrams from the archives differ in how the diode is added. (I put a 1A inline fuse where both of the attached drawings shows a resistor.) In either drawing, it won't be possible to place the legs of the resistor outside of the legs of a diode in my LED circuit with the integrated resistor. Is that going to be a problem? Would one circuit drawing be preferred over the other? Thank you, Eric Schlanser Z13/8 project Lyc O320 with Skytec 12V HT Wound-Field starter and auto style starter contactor > I think I remember reading if an LED is used instead of a bulb for > starter engagement indication, that you best add a diode to prevent > the LED from getting hurt when power is removed from starter motor. > Where does the diode get placed, in series with the LED or in parallel? > > Either would work . . . but parallel is preferred. > If answer is in parallel where would the dropping resistor for LED > get placed, between the legs of the protection diode, or outside? > > Outside Bob . . .[/quote] > Read this topic online here: http://forums.matronics.com/viewtopic.php?p=422741#422741 Attachments: http://forums.matronics.com//files/starter_engaged_warning_lt_248.pdf http://forums.matronics.com//files/in_line_resistor_6_966.jpg ________________________________________________________________________________
Subject: Re: How do I prevent starter engaged LED from failing?
From: "user9253" <fransew(at)gmail.com>
Date: May 03, 2014
If I understand correctly, you are asking if a protection diode has to be connected inside of the LED fixture at the junction of the LED and integrated resistor. No it does not. It can be external as in the attached drawing. Joe -------- Joe Gores Read this topic online here: http://forums.matronics.com/viewtopic.php?p=422751#422751 Attachments: http://forums.matronics.com//files/led_207.pdf http://forums.matronics.com//files/led_102.jpg ________________________________________________________________________________
Date: May 03, 2014
From: "Robert L. Nuckolls, III" <nuckolls.bob(at)aeroelectric.com>
Subject: Re: How do I prevent starter engaged LED from
failing? At 06:40 PM 5/2/2014, you wrote: > >Bob et group, > > In wiring a starter engaged annunciator, I am using an > off-the-shelf LED fixture with an appropriate resistor already > integrated into it. The resistor is actually soldered directly to > the LED lead inside of the fixture. > > The two attached diagrams from the archives differ in how the > diode is added. (I put a 1A inline fuse where both of the attached > drawings shows a resistor.) In either drawing, it won't be possible > to place the legs of the resistor outside of the legs of a diode in > my LED circuit with the integrated resistor. Is that going to be a problem? >Would one circuit drawing be preferred over the other? They are the same. The protection diode is wired in parallel with the led to clamp off any reverse voltage transient. The resistor is shown upstream and close to the monitored power at the starter. Having this resistor located remotely eliminates the need for fusing the sense lead. Suggest you modify the fixture to move the resistor out to the source end of the wire and add the diode in parallel with the led. Bob . . . ________________________________________________________________________________
Subject: Re: How do I prevent starter engaged LED from failing?
From: "eschlanser" <eschlanser(at)yahoo.com>
Date: May 03, 2014
Joe, You correctly understood my question. Now I must decide whether or not to use your circuit or another fix I came up with after a good night's rest. Since I already have the inline fuse, I can replace the LED in my circuit with a tiny incandescent bulb http://tinyurl.com/kw8usxc I much appreciate your reply and diagram. Eric Schlanser Read this topic online here: http://forums.matronics.com/viewtopic.php?p=422756#422756


March 12, 2014 - May 03, 2014

AeroElectric-Archive.digest.vol-mh